Sunteți pe pagina 1din 227

MRCOG part 1 RECALLS SEPTEMBER 2008

Paper 1 & 2 EMQs In a country wide survey, 10 general practices were picked at random and 5% of patients were selected at random from each practice: Ta) the sample of patients is a true random sample Tb) all practices had an equal chance of selection Tc) all patients had an equal chance of selection Fd) two siblings could not have been selected Te) inferences about all patients in the country may be drawn from the sample A lady is about to deliver and you are about to give her a pudendal block.. a) ant division of ant. remi of S2-4 b) ant division of post.rami of S2-4 b)genitofemoral nerve c) post division of ant. remi of S2-4 d) post division of post. remi of S2-4 e) sensory supply of perineal nerve f) inferior rectal nerve g) ischial tuberosity h)ischial spine Q1 -wht is the root value of pudendal nerve? Q2- wht nerve supplies lower part of vagina (I think)? Q3 why do u give local skin infiltration before episiotomy? If we take the day of fertilization as Day0 then Day2 Day4 Day 8 Day 10 Day12 Day 14

Day 18 Day20 Day 22 Day 24 Day 26 Day 42 Day 56 Day 70 Q4-Which represents 4 cell stage ? Q5- conceptus implants completely? Q6-feable hrt pulse seen on ultrasound? Q7- vertebra form completely? Table: on different values given for deaths due to maternal mortality with direct and indirect causes as well as # of days from delivery to the cause of death Deaths due to placenta previa and hypovolemia x @ e.g 20 days,10 days,8days etc suicide(with previous hx of psychiatric illness) crash victim diabetic neuropathy two more causes given with values

Different numbers were given in options Q8-the number of late maternal deaths due to indirect causes? Q9-number of maternal deaths due to direct causes? Calculate Numbers needed to treat from a drug trial on osteoporosis: (Q frm EMQ book) Options

25 50 100 250 300 400 1000 others Q10-N1=10,n2=5,n3=9990,n4=9995 Q11-N1=50,N2=25,n3=9950,N4=9975 FSH Oxytocin GH Prolactin Rennin ACTH etc Q12-which is a protein with alpha and beta subunits ? Q13 which stimulates mineralocorticoid activity/relsease? Q14- which hormone is produced by posterior pituatory? Folic acid Magnesium Calcium Vit b1-B12 given Q15- which vitamin is required in 400mcgs at preconception? Q16-which vitamins absorption is hampered by oxalic acid & phytates? Insulin & Glucagon excretion Decrease decrease Increase increase Increase decrease Deacrease increase

Nochange no change Nochange increase Nochange decrease Etc Q-17-on taking a large protein meal? Folowing are risk factors for cancer Q-18- HCC ans Hep B Q19- Skin cancer-ans UV radiation Q20- Cervical/uterine cancer-ans HPV Q21- Breast cancer-ans BRCA1 mutation (Repeats from march 2008) Hormone receptors for following are located in a) protein kinase nuclease receptor b) nucleae transcription receptor c) G-linked cell membrane receptor d) G-linked golgi apparatus receptor e) Protein complex intra cytoplasmic receptor f) Multiligand receptor in cell membrane etc g) Protein Tyrosine kinase activity receptor on cell membrane Q22- Insulin Q23-PG E2 Q24-Estrogen Q25-Progesterone

Q26-A family with one kid presumed to have cystic fibrosis however cftr gene mutation was not detected. Family members willing to give samples

if needed.mother wants definite diagnosis as she is 11wks and wants to know diagnosis for her baby.parents are cousins with 1st degree relatives having cystic fibrosis. Invasive diagnostics can be applied to reach final results by? Q27-you have Guthrie spot of their son how will u diagnose him for cystic fibrosis? Q28- families in uk with cystic fibrosis how will u find f508? Options: Pcr Fish test Chromosomal linkage analysis Telomer analysis Sum hereditary crosslinkage chromosomal option (dnt remember) Screen for some known gene mutation Sweat test Saliva test screening test for following Q29-alpha thalessemia Q30-folic acid deficiency Q31-iron deficiency anemia Q32-(one more i think!) options hbelectrophorisis mcv mchc MCh serum iron paul-bennel test etc Q33-which is the commonest cause of non-dysjunction at meiosis? trisomy 21

Q-34 which defect is associated with 60-70% cleft palate and cleft palate? trisomy 13 or 18 or KF syndrome all about cystic fibrosis screening tests for a-thallesemia trait iron deficiency amemia,folic acid anemia and....EMQ RECEPTORS FOR ESTROGEN,PROGESTERONE,PROSTAGLANDIN,INSULIN ....EMQ

NERVE SUPPLYING PERIANAL SKIN,PUDENDAL NERVE ORIGIN,LA DURING EPISIOTOMY IS GIVEN FOR?????......EMQ EMQ receptors for insulin prostaglandins estogen progesteron primary syphilis in early untreated period options syphilis treponemal antibody test syphilis hemaglutination test syphilis immobilization test emq traetment of chlamedia ..................bacterial vaginosis emq pre malignant conditions emq statistics calculate n to be traetad emq diagnosis of iron defecincy anaemia folic acid .................. early untrated syphylis emq congenital adrenal h icreased ? SUBSTANCE

emq embryology 4 cells ? day vertebra formation mcq about DIGITALIS, cyclosporine .mitochondrial dna ,cd8 cells , the uterer(2 mcq) ,3 repeated statistics mcqs, EMQ 36 WS PRIMI GRAVIDA had flu like sympt , the baby was born jaundiced , hepatosplenomegaly ,skIn rash , brady cardia ,,,,choose the cuase (all viruses ,bacteria,listeria,plasmodium )??????

EMQ ...why we do local infiltration + pudendal block EMQ ...cause of post operative infection ??pseudomonas

EMQ .....WHICH vit or mineral absorbtion is affected by phytate (no iron in the choices

MCQ citric acid cycle fatty acid oxidation ???? genetic imprinting ?????stylomastoid foramen spinal cord in smooth muscle cells /regeneration possible /actin &myosin present/lengh of fibers repeated 2 mcqs u/s and radiation effect apoptosis 2 mcqs about hyperplasia pelvic splanchnic nerve Paper 1 & 2 EMQs A lady is about to deliver and you are about to give her a pudendal block.. a) ant division of ant. remi of S2-4 b) ant division of post.rami of S2-4

b)genitofemoral nerve c) post division of ant. remi of S2-4 d) post division of post. remi of S2-4 e) sensory supply of perineal nerve f) inferior rectal nerve g) ischial tuberosity h)ischial spine Q1 -wht is the root value of pudendal nerve? Q2- wht nerve supplies lower part of vagina (I think)? Q3 why do u give local skin infiltration before episiotomy? If we take the day of fertilization as Day0 then Day2 Day4 Day 8 Day 10 Day12 Day 14 Day 18 Day20 Day 22 Day 24 Day 26 Day 42 Day 56 Day 70 Q4-Which represents 4 cell stage ? Q5- conceptus implants completely? Q6-feable hrt pulse seen on ultrasound? Q7- vertebra form completely? Table: on different values given for deaths due to maternal mortality with direct and indirect causes as well as # of days from delivery to the cause of death Deaths due to placenta previa and hypovolemia x @ e.g 20 days,10 days,8days etc suicide(with previous hx of psychiatric illness) crash victim diabetic neuropathy

two more causes given with values

Different numbers were given in options Q8-the number of late maternal deaths due to indirect causes? Q9-number of maternal deaths due to direct causes? Calculate Numbers needed to treat from a drug trial on osteoporosis: (Q frm EMQ book) Options 25 50 100 250 300 400 1000 others Q10-N1=10,n2=5,n3=9990,n4=9995 Q11-N1=50,N2=25,n3=9950,N4=9975 FSH Oxytocin GH Prolactin Rennin ACTH etc Q12-which is a protein with alpha and beta subunits ? Q13 which stimulates mineralocorticoid activity/relsease? Q14- which hormone is produced by posterior pituatory? Folic acid Magnesium Calcium Vit b1-B12 given

Q15- which vitamin is required in 400mcgs at preconception? Q16-which vitamins absorption is hampered by oxalic acid & phytates? Insulin & Glucagon excretion Decrease decrease Increase increase Increase decrease Deacrease increase Nochange no change Nochange increase Nochange decrease Etc Q-17-on taking a large protein meal? Folowing are risk factors for cancer Q-18- HCC ans Hep B Q19- Skin cancer-ans UV radiation Q20- Cervical/uterine cancer-ans HPV Q21- Breast cancer-ans BRCA1 mutation (Repeats from march 2008) Hormone receptors for following are located in a) protein kinase nuclease receptor b) nucleae transcription receptor c) G-linked cell membrane receptor d) G-linked golgi apparatus receptor e) Protein complex intra cytoplasmic receptor f) Multiligand receptor in cell membrane etc g) Protein Tyrosine kinase activity receptor on cell membrane Q22- Insulin Q23-PG E2 Q24-Estrogen Q25-Progesterone

Q26-A family with one kid presumed to have cystic fibrosis however cftr gene mutation was not detected. Family members willing to give samples if needed.mother wants definite diagnosis as she is 11wks and wants to know diagnosis for her baby.parents are cousins with 1st degree relatives having cystic fibrosis. Invasive diagnostics can be applied to reach final results by? Q27-you have Guthrie spot of their son how will u diagnose him for cystic fibrosis? Q28- families in uk with cystic fibrosis how will u find f508? Options: Pcr Fish test Chromosomal linkage analysis Telomer analysis Sum hereditary crosslinkage chromosomal option (dnt remember) Screen for some known gene mutation Sweat test Saliva test EMQ 1 ST PAPER WHICH NERVE SUPPLIES PERIANAL SKIN EMQ 2 CYSTIC FIBROSIS a - invasive testing will be done ( she is preg 11 ws) which test b- their child blood is ready for guathri test ,which test you will do c- the relatives are willing to be tested , which test will reveal the 405 mutation for families in great braiten any one have suggetions more options were given for these qs karyotyping comparative genomic hybridization flourecent ???? MCQ

propranolol smooth endoplasmic reticulum anal canal smooth muscles (2 mcq ,paper1 ,2 ) primitive streak fetal circulation ??microtubules digitalis mcq ; the following are bacteria cryptococcus ,candida,histoplasmosis, mcq : the embryological origin of peritoneum,.... mcq mitosis miosis mcq : the follwing are structral abnormalities of chromosomes mcq : ?..................increase with preg mcq : drug interaction methotrexate , sulphonamides

mcq :arterial and mean blood pressure mcq : causes of alkalosis mcq : the folloing are types of cells with their secretion ? endothelial cells heparin ?mast cells igE ? plasma celle (i am not sure of the exact coupling) mcq at 20 ws gestation rbcs are made in liver lymph nodes spleen bone marrow mcq about pudendal nerve mcq ischiorectal fossa mcq ureter in abdomen mcq ureter in pelvis mcq foramen of winslow (in detailes)

mcq kallman syndrome in men mcq hcg mcq trophoplasts mcq ? all types of cells in islets of langerhans and their products mcq the following are premalignant conditions: paget disease ...(repeat mcq endotoxins or exotoxins ( i am not sure) MCQ CORPUS LUTEUM mianly endothelial clls secerets inhibin function decrease after 12 ws MCQ INHIBIN

MARCH 2008
cystic fibrosis guthic spot test receptors in apoptosis caspa arias stella reaction blood picture in PET and in preg is inflam like 1.EMQ paclitaxel carboplatin side effects 2.EMQ complex partial seiizure gum hypertrophy,acne ,facial coarsening,vit K TO BE given..?drug used 3.EMQ oestrogen progesterone receptor site 4.EMQ on vitamins-xerophthalmia..morning sickness..macrocytic anaemia. 5.MCQ rubella incubation period,specific immunity within 15 days.. 6.EMQ HEPATOCELLULAR carcinoma,..?? 7.EMQ lining epithelium of ureter,..?? 8. EMQ montoux rection.. 9.MCQ ovulatory DUB 10.MCQ concerning aneurysm- 10% due 2 inflam.,syphilitic aneu,thoracic vessels,marfan recessive

if a mother has a child with cystic fibrosis ,she is preg and will do amniocentesis a. which test to be done (every possibility mentioned) b. all relatives will volunteer to do gutic spot test ,whichtest you will carry on the blood -graph with hormones of the endometrial cycle -citric acid cycle (graph) microsatellites alleles - cd4 -nk cells -down syndrome -pulmonary embolism anal canal -vagina -ovary

phenylketonuria inhibin passive transport erythropoietin renin hcg levels syphylis rubella chlamydia hormone dependent malignancies(testicular carc._sertoli leydig-clear cellcarc.ofkidney-thyroid carc.) pigd missed abortion MCQ congenital heart emq crown \heel lenghth at birth crown/rump length 6 cm =? weeks

about 10 questions statistics plrese does anyone remember the q about ring Y chromosome q about sattellite alleles ?? passive diffussion not depend on . concenteration gradiant molecular size

2following tissue are capable of regeneration spinal cord liver epedermis myocardium bone marrow

3 double blind trial delliosion of HCG in the urine pregnancy test about the q APOPTOSIS and capsase ,q methods of disinfection and BOWIE DICK test where is source to study these subjects? q progesterone receptors q about progesterone receptors syphilis , toxoplasma , hpv , immune responses EMQ type of hpv causing benign wart type of hpv causing cancer cx MCQ down syndrome associated with duodenal atresia true associated with ambig genetalia false only maternal chromosome F

only paternal chromosome F MCQ oogonia miosis starts at puberty miosis before mitosis during S phase chromosomes are doubled the most diagnostic test to be done on amniocentesis gautic spot test what test you do with the blood sample emq about vitamins (hyperemesis) i wrote vit B6 and (xerophthalmia) q about endometrial hyperplasia q about side effect of drug (carboplatin)-(taxal) q about arias stella if its specific in pregnancy q about chlamydia q about varicella q about hpv ,wart answers 16 or 18 or 6,11 q about hcg conc in pregnancy citric acid cycle diagram hormones diagram fsh,lh,estrogen,testosteron q about:by transvaginal u/s yolk sac apear in which week many questions in genetic about pcr,fish test q about laser q in pathology about totipotent cell,tumour like mass q about voiding presure q about ca requirement in pregnancy q about test used to diagnose folic acid deficency q about thalasthmia q about cervical ectropion cells i choose columner cell - mri contraindicated in early pregnancy q about stallete instability in dna - transverse abdominus ms attached to lumbar transverse process -cut s1 lead to autonomic bladder

q about cancer which are hormone dependent -breast -prostate -thyroid q about guthre test q about cystic fibrosis and diagnosis -swet test -karyotyping-fish test q about disinfection and sterelization q about complication of contraceptive pills emqs 1lining of ureter 2.erythropoeitin and renin q from john duthie 3complications of cisplatin 4 paclitaxel 5statistics -std error calculation 7 mean in a normal distribution 8occipito frontal diametr 9hcg titre at detection by tvs 10 at 3 days aftr fertilization 11crl in scan is 60mm what is gestational age 12crl at term ? 13 parietal sutur is between 14kuffer cells are -phagocytic 15 estradiol receptors 16 progestrone receptors -we know that thes are intra nuclear but there were 2 confusin options with intranuclear plz refer this in depth , one option had intranuclear kinase or somethin 16anticonvulsant with description of phenytoin 17vitamin deficiency causin hyperemesis 18 - macrocytic anemia 19 -xerophtalmia 20 cystic fibrosis 3 emqs 24 citric acid cyle diagrm , enzymes were options complement causes 26 b thalasemia detection 27 folate def detection

29 basal cell ca - local malignant i think 30tumor like hamaroma

March 2007
Copper IUD mode of action: sperm motility microthrombi MRI: ionising radiation? no adverse effect on fetus? detect fetal cardiac abn? USG: probe: array of magnets? doppler to detect fetal heart movement? facial hair increased by: oestrogen? testerone? spirolactone? characterics: pass thro' origin? slope = 0? pass thro the mean? unaffected by changing scale? unaffected by changing dependent variable? Copper containing IUDs: F should be changed every year F have a high incidence of actinomycoces colonization than plastic devices T Cause a relative increased in ectopic pregnancy T have been implicated as a cause of fatal infection in pregnancies F do not cause menorrhagia Comments: Inert devices can be left in place until the menopause, but copper devices need renewal every 3-5 years, depending on the make, because of the gradual absorption of copper. Copper IUCDs produce local

concentrations of copper salts which apparently give some protection against bacterial contamination. Pelvic infection with actinomyces organisms is most likely with a plastic device that has been in situ for some years. While the rate of intrauterine pregnancy is reduced, that of ectopics is not. Hence, there is a relative increase in ectopic pregnancy after IUCD insertion. If an IUCD is left in place there is a slight risk of intrauterine infection, preterm labour and antepartum haemorrhage, but most pregnancies are uncomplicated and the device is delivered with the placenta. Increased menstrual loss may be caused by increased fibrinolytic activity which occurs round the IUCD. The progestogen intrauterine system (IUS) reduces menstrual flow and often dysmenorrhoea. Which of the following is/are true concerning MRI? A It involves ionizing radiation (False) B Has no recognised side effects on the foetus (True) C The pregnant mother should be turned to her left side during scanning (True) D Tissue with high hydrogen concentrations are difficult to distinguish False) E Blood vessels appear white on scanning (False) Comments: Water is a molecule composed of hydrogen and oxygen atoms. The nuclei of the hydrogen atoms are able to act as microscopic compass needles. When the body is exposed to a strong magnetic field, the nuclei of the hydrogen atoms are directed into order - stand "to attention". When submitted to pulses of radio waves, the energy content of the nuclei changes. After the pulse, a resonance wave is emitted when the nuclei return to their previous state. The small differences in the oscillations of the nuclei are detected. By advanced computer processing, it is possible to build up a three-dimensional image that reflects the chemical structure of the tissue, including differences in the water content and in movements of the water molecules. This results in a very detailed image of tissues and organs in the investigated area of the body. In this manner, pathological changes can be documented.[/color:febed2ebf4][/b:febed2ebf4]--------------------------------------------------------------------Many stats about sensitivity, positive predictive value, accuracy, even the linear regression Acute UTI in pregnancy

rarely symptomatic ?F Copper IUD mode of action: ?T sperm motility ?T microthrombi ?T intervent ovary movement MRI: ionising radiation F no adverse effect on fetus T detect fetal cardiac abn ?F ( I think it will be very small) USG: probe: array of magnets? ?F doppler to detect fetal heart movement T facial hair increased by: oestrogen?F testerone?T spirolactone F ( I think it is used for treating hirsutism) Mycobacteria ?T strict ?? *(nor sure which word they use ) aerobic ------------------------------------------------------------------48---if a distribution of results is markedly skewed to the left(sep2000) a-the mean is same as 50th centile f b-the same no of values lie on either side of the median- t c-the mode is equal to the median- f d-the students test should be used to compare this distribution with another- f e-logrithmic transformation of the results will produce a distribution close to normal-t -concerning the ablity of a test to predict disease a-sensitivity is the ablity to predict those with disease correctly T b--sensitivity is the same as positive predictive value F c-the confidence interval must cross 1 to prove significance t (NOT A MUST as CI at 1 is perfect ) d-an odds ratio of 1;3 implies the risk of 33% f e--an odds ratio of 2 indicates a halving of risk F

-----------------------------------------------------------------------------------1-SUCCESSFUL LACTATION IS a-maintained by estrogen -b-maintained by progesterone -c-initiated by prolactin surge-d-maintained by human placental lactogen e-inhibited by dopamine 2-THE RELEASE OF CATECHOLAMINES FROM THE ADRENAL MEDULLA INCREASES a-during sleep b-when the nerves to adrenal glands are stimulated c-when the blood sugar rises d- immediately following a myocardial infarction e- in the presence of pheochromocytomaT F-DURING ACUTE HAEMORRHAGE 3- In the pathogenesis of thrombosis a-prostacyclin induces platelet aggregation b-plateletes synthesis thromboxane A2-c-thromboxane A2 induces vasoconstriction-d-contact with subendothelial collagen causes platelete aggregation-e-thrombin inhibits platelete aggregation --4- CONVERSION OF GLUCOSE TO LACTIC ACID a-occurs in single enzymatic reaction b-is the only pathway for synthesis of ATP in red blood cells C-is a reversible process in skeletal muscle d- is inhibited by high cellular conc of ATPe-occurs in skeletal muscle when availability of oxygen is limited5-GLUCOCORTICOIDS a- promotes hepatic gluconeogenesisb- suppress uptake of glucose by musclecpromotes protein breakdowndpromotes fat breakdowne-increase glycolysis in adipose tissue6-Actinomycete israelii

1-Is a fungus 2- forms yellow granules in pus 3-is a commensal in mouth-4- is a commensal in vagina-5- is usually resistant to penicillin 7-Following substances increase the serum uric acid concentration( a-colchicine-b-chlorothiazide-c-allopurinol-d-probenecid-e-phenylbutazone 7--THE FOLLOWING ARE INHERITED AS AUTOSOMAL RECESSIVE CONDITIONS A. tuberous sclerosis. b-phenylketonuria-C. ahondroplasia.-D. sickle cell anaemiaE. Von Giek l disease. 8-. Genes on sex chromosomes are responsible for the inheritance of ( A. glucose-&pbosphate dehydrogenase defiency. B. defective colour vision. C. hairy ear rims. D. homocystinuria. E. Hurler syndrome. 9-in human a haploid no of chromosomes is found in) a-red blood cellsb-blastocystsc-primary oocytes-. d-the first polar bodye-spermatozoa10-osteoprosis is associated with a-an increase in uncalcified bone matrix(osteoid tissue b-prolonged oestrogen therapy

c-a normal histological bone structured-bone fracture e-irregularity of epiphyseal plates 11-THE FOLLOWING HAVE an anti-emetic effect a-hyoscine bromide b-morphine sulphate c-chlorpropamide-d-promethazine hydrochloride e-perphenazine 12CLOMIFENE CITRATE a- is an antiandrogen b-does not stimulate ovulation directly c-can produce visual disturbances-d-is generally prescribed throughout the proliferative phase of the menstrual cycle e-in the treatment of anovulation increases the risk of multiple pregnancy 13--if a distribution of results is markedly skewed to the left a-the mean is same as 50th centile b-the same no of values lie on either siDe of the median c-the mode is equal to the mediand-the students test should be used to compare this distribution with anothere-logrithmic transformation of the results will produce a distribution close to normal15. when a man has hemophilia a. 50% of his daughters would not expected to be carriers b. 25% of his sons expected to be carriers c. Good medical control of blood definition reduce the risk of this condition in his children d. His new born child is likely to require an urgent blood transfusion e. His sister has 50% probability of being a carrier 16- THE INTERSTITIAL CELL (LEYDIG) OF THE TESTIS a--secrete seminal fluid b-are stimulated by LH c-secrete androgen binding protein

d-secrete fructose e-produce testosterone T in response to LH -17--CONCERNING VIRUSES a-the core of every virus contain RNA b-they usually produce intracellelur toxin causing cell death c-antibodies are directed against capsular protein d- they can be grown in intact cells-e-interferone are synthetic antiviral substances 18-THE FOLLOWING ARE CYTOTOXIC ALKYLATING AGENTS( a-cyclophosphamide b-mercaptopurine-c-chlorambucil-d-fluorouracil-e-methotrexate19-2,3 DIPHOSPHOGLYCERATE a-is present in higher conc in maternal erythrocytes than fetal erythrocytes-b-binds more avidly to haemoglobin A than to haemoglobin E c-increases the affinity of haemoglobin for oxygen d-is a phospholipid e- is synthesised by the pentose phosphate pathway 18-HYPERKALEMIA IS A CHARACTERSTIC FINDING IN TINDALL a-primary aldosteronism b-treatment with Spironolactonec-hyperparathyroidismd-ACTH secreting tumours of bronchuse-renal failure19- CONCERNING INHERITABLE DISEASES A-huntigdon ,s chorea is transmitted by a dominant gene b-phenylketonuria is transmitted by a recessive gene c-haemophillia ia an autosomal dominant condition-d- Von Willebrand disease is a sex linked condition e-cystic fibrosis is transmitted by an x linked recessive gene--

20-Early blood borne dissemination is a characteristic feature of: a) carcinoma of the endometrium b) osteosarcoma c) basal cell carcinoma d) carcinoma of the cervixe) choriocarcinoma21- in tissue pigmentation the following are associated a-kernicterus and conjugated billirubin b-addison disease and increased cutaneous melanin c-melanosis coli and bile pigmentsdwilson disease and copper deposition in the corneae-corpus leuteum and carotenoids22-acquired diverticular disease of the colon a-is present in at least 15% of Caucasian over the age of 50 yearsb-is due to congenital abnormality of the bowel wall c-is associated with increased intraluminal pressure d-is associated with muscular thickening e-may result in intestinal obstruction 23 stored blood which is to be used for transfusion a-kept at -4 degree b-must be used in 1 week c-is tested for compliment content before transfusion -d-may be used for platelet replacement e-contains an acid anticoagulant 24- in uncomplicated homozygous beta thalasemia there is a-hypochromasia-b-a reduction in haemoglobin A c-an increase in haemoglobin F d- RED CELL SICKLING e-presence of megaloblasts in bone marrow32-The following are capable of cellular regeneration a)spinal cord b)liver parenchyma c)gut epithelium d)kidney ???

e)bone marrow 33-during the development of female reproductive system a-primordial germ cells arise in yolk sac-b-ovarian development is dependent upon oestrogen activity-c-the paramesonephric duct give rise to the cervix-d-the greater vestibular glands arise from the urogenital sinus-e-differentiation of external genitalia is dependent upon ovarian activity-36- IN CONGENITAL ADRENAL CORTICAL HYPERPLASIA a-commonest deficiency is C21 b- plasma cortisol conc is raised c-there may be excessive secretion of 17 hydroxy progesterone-d- sodium retension is characterstic e-blood catecholamine conc are increased 37-- SCHISTOSOMA HAEMATOBIUM a- is a snail b- is prevalent in china c- infestation may affect the uterine cervix d- give rise to chronic granulomatous lesions-e- infestation predisposes to carcinoma 38- KETONE BODIES . a-can be utilised by theadult brain. B-Include acetone c-are water soluble -d- are synthesised in skeletal muscles e-can be utilised during starvation 39-IRON IONS a-diffuse passively into erythropoitic cellsb-bind to transferinc-are taken up by hepatocytesd- are necessary for cytochrome synthesise-are absorbed predominantly by the ileum40-- --the conjugation of billirubin

a-takes place in hepatocytesb-is catalysed by UDP glucuronyl transferasec-is inhibited by phenobarbitoned-renders it water solublee-is impaired in acute billiary obstruction41- in radiotherapy( a-1Gray is eq to 1joule per kg b- the skin usually recieve a greater dose of radiation than underlying tissuec-the major effect of radiation energy is to damage the cytoplasm of the celld-cells in tissue which are hypoxic are more vulnerable to radiatione-radiation induced changes in tissues may take 6 weeks to develop-42--osteomalacia is characterised by a-mineralization of the periostium b-deposition of uncalcifiedbone matrix -c-normal osteobastic activity -d-increase capillary fragility e-normal calcification of bone 43-progesterone a-is a C21 compound b-synthesised by the ovary before ovulationc-increases ventilationd- raises BMRe-binds to corticotrophin binding globulin44-EPIDERMAL GROWTH FACTOR A- is mitogenic-. b- synthesis is stimulated by oestradiolc-is a steroid molecule-F ... d- is found in endometrium-e- binds to receptor on the nuclear membrane45-the following are structural aberrations of chromosomes: a. deletions -b. inversions

c. aneuploidy d. . polyploidye translocations 46-- Messenger RNA a-Synthesis is dependent on RNA polymerase b-is an exact copy of sense DNA c-Contains exons d-Is measured by Western blotting e-translation occurs in the nucleus 47-- BETA SYMPATHOMIMETIC DRUGS MAY A- caue bronchospasm b-reduce frequency of uterine contraction c-cause heart block d-reduce diastolic blood pressure e- increase blood glucose conc 48- The following statistical statements are correcta-in the normal distribution,the value of mode is 1.73 times the median b-in a distribution skew to the right,the mean lies to the left of medianc-in the series- 2;7;5;2;3;2;5;8, the mode is 2 d-students t test is designed to correct for skew distribution e-the chi squared testmay be used when data are not normally distributed50-Contraindicated in breast feeding A-POPs B-BromocriptineC-chloroqunined-?? e-warfarineF 51-Contraindicated with kidney impairment A-Dopamine B-Gentamycine C-Cephahexin D-Cis platin 52- the following are characters of hypoxic cell death:

A-Apoptosis -B-Phagocytosis C-Pyknosis -D-Poikilocytosis E-Release of phospholipids 52- the following are characters of hypoxic cell death: A-Apoptosis -B-endocytosis C-Pyknosis -D-Poikilocytosis E-koliosytosis prostagladinsa-lipooxygenase pathway b-thromboxane causes vasoconstriction c-increased in the myometruim d-? e-? linear regression: a-starts from the origin b-passes thru the meanc-values vary d-ranking is done??? following test r used to compare 2 drugs a and b a-student t test b-meta analysis c-wiloxone paired test withney u paired test e? toxic shock syndrome associated with the use of tampoons b-due to toxigenic starinsof strep c-infrequently reported outside n america d-a consequence of previous antibioitic therapy e-confined to seualy active women the following can regenerate

a-kidneyepitheluim? b-pheriheral axonsc-liver parenchyme d-bone marrow e-bone radiation sensitive a-bone b-bone marrow c-epith of gut d-skin e-? Mycobacteruim a- are alchochol acid fast b-does not form spores c-d-a facultative anaerobes d-responsible for leprosy e-pathogenic inhumans all strains the germination of tetanus spores in a wound is inhibited by a-tissue trauma b-oxygen c-inject of toxoid d-injection of antitoxin e-removal of devitalised tissue Aldosterone a-reduces Na resorption in PCT b-reduces Na absorption in descen loop of henle c-Increase Na absortion in DCT Increase K loss from the tubule e increases Na absortion in collecting tubules the following r conclusive evidence of pregnancy in uterine cureetings???? can, t remmber a-decidua compactab-Arias -stella changes in endometrail gladc-spiral arterioles d-plasma cell infiltration e chorionic villi

White cell migration from the bld vessels in areas of inflammation involves--- march 1997/2 21 a-call migration between the endothelial cells b-a pssive loss of fluid bld elements c-cell migration independeant of endothelial cell motion d-initail emigration of polymorhs neutrophils e-more polymorphs than monocytes after 2 day= the vulva is supplied by a-ant cutaneu erve of the thigh b-femoral br/o genito femoral nerve c-? d? e? glucocorticoids a-promote hepatic gluconeogenesis b-suppress uptake of glucose by muscles c-promote protein break down d-promote fat reakdown e-increase glycolysis halothane(refer sept 1997)42 a-cardic arrythmias b-explosive mixtures with air c-liver damage 53- U/S: A-Pulstile B-Increase body temp 1 degree after 15 min scanning.. C-High frequency penetrates deeper TISSUE,D-Can distinguish between 2 points closer than 0.5 mm E-Best echoes are produded by beam at right angle to the structures. 54-Disinfectant solutions may become contaminated with: 1-Enterobacter species 2-Streptococcus species 3-Escherichia coli 4-Pseudomonas aeruginosa 5-Staphyloccus pyogenes-

55-Nitric oxide a) is synthesised in the endothelium b) has a short half life -c) causes smooth muscle contraction d) increases during pregnancy e) combines with oxygen to produce L-arginine but few question got modifications eg; nitrous oxide synthesised by macrophages naloxone antiemitic effect drugs contraindicated in breast feeding drugs causing renal impairment ultasound mri statistics quit difficult all definitions rna pre implantation diagnosis FIBRINOGEN (1 2) HEPATOGLOGIN 3) FERRITIN 4) ... predominantly produced by UG system - is a kind of lukotriene - is phospholipid which reactant protein will increase in injury -------------------------------------------------------------------------------1) FIBRINOGEN 2) HEPATOGLOGIN 3) FERRITIN 4) albumin 5) ? Cell cycle -prophase I and II

-mitosis I and II Vulva nerve supply Urinary bladder - lining/embrology vagina - embrology Fetal testis IL I MCH I Function of Neutrophil polymorphs Fetal HbF PG Umblical cord embrology prostaglandins half lifa 30 min f il1 produced by macrophages whartons jelly originate from extra embryonic endodem detrusor muscle is of mesodermal origin acute phase proteins are transferrin alfa feto protein preimplantation diagnosis

possible 2 detect the sex of the embryo uses pcr 4 chromosomal deletions can be used 4 detection of single gene disoderes facial hair causes testosterone mestranol spironolactone real time bias length time bias negative predictive value fibrocystic disease of thr breast...in paper 2 , breast anatomy and breast from endocrine function (in paper 1..)...ie 3 qs we about the breast levator ani its attachnment and its coverings endometrial carcinoma adenomyosis about measuring the arterial blood pressure and its methods interleukin 2 prenatal implantation genetic diagnosis menstural cycle in paper 1 and 2

MARCH 2006
The germination of tetanus spores in a wound is inhibited by a)tissue trauma b)oxygen c)injection of anti-toxin d)injection of toxoid e)removal of devitalised tissue The following values fall within the normal range for the adult female bladder a)residual volume of 100ml b)voiding capacity of 250ml c)bladder capacity of 900ml d)intravesical pressure rise of less than 10cm H2O during early filling e)maximum urine flow rate of 60ml per second

In the small intestine, the following substances are absorbed by active processes a)water b)sodium c)vitamin K d)amino acids e)chloride White cell migration from blood vessels in areas of inflammation involves a)cell migration occurring between endothelial cells b)a passive loss of fluid blood elements c)cell migration independent of endothelial cell motion d)initial emigration of polymorphonuclear neutrophils e)more polymorphs that monocytes after 2 days Antibodies (!!) a)are soluble proteins b)are formed in the fetus before 12 weeks of intrauterine life c)have an average molecular weight of around 10000 daltons d)of the rhesus type are genetically transmitted e)are produced by the ribosomes of plasma cells In DNA a)a codon is a sequence of three bases b)all codons have an identified function c)there is a greater variety of amino acids than there are different codons d)replication can be initiated at several different points along a chromosome e)complementary pairing precedes messenger mRNA synthesis The inferior vena cava a)is formed at the level of the fifth lumbar vertebra b)commences posterior to the right external iliac artery c)receives the left ovarian vein d)receives the right ovarian vein e)pierces the central tendon of the diaphragm In the normal human pelvis a)the promontory of the sacrum is in the upper anterior border of the first sacral vertebra b)the anterior surface of the sacrum has five paired foramina

c)the joint between the two pubic bones is a synovial joint d)the acetabular fossa is wholly formed from parts of the pubic and ischial bones e)the transverse diameter of the brim is greater than the anteroposterior diameter Arginine vasopressin a)reduces the glomerular filtration rate b)controls water loss in the proximal renal tubule c)is synthesised by the posterior pituitary gland d)is released in response to a rise in plasma osmolality e)is released in response to a fall in circulating plasma volume Concerning ovarian function a)progesterone is the major steroid of the developing follicle b)granulosa cells secrete oestradiol c)oestradiol is derived from androgen precursors d)insulin-like growth factor (IGF-1) is not secreted by the ovary e)circulating inhibin concentrations are a marker of granulosa cell function Actinomyces israelii a)is a rickettsia b)forms yellow granules in pus c)is a commensal in the mouth d)is a commensal in the vagina e)is usually resistant to penicillin Halothane produces a)cardiac arrhythmias b)explosive mixtures with air c)liver damage if given repeatedly d)myometrial relaxation e)bronchial irritation The therapeutic effect of the first drug is enhanced by the second drug a)phenytoin: ethinyloestradiol b)bromocryptine: metoclopramide c)penicillin: probenicid d)ritodrine: dexamethasone e)warfarin: phenobarbitone Potassium

a)is mainly intracellular b)plasma levels vary in proportion to intracellular levels c)plasma levels are decreased in Addisons disease d)plasma levels are increased in diabetic ketoacidosis e)deficiency occurs with prolonged vomiting Concerning carbohydrates a)sucrose is a disaccharide of glucose and fructose b)cereal grains contain less than 40% starch c)cellulose is a fructose polysaccharide d)a normal diet contains less than 60g of carbohydrate daily e)dietary carbohydrate is oxidised in the body to carbon dioxide and water Steroid hormones a)all contain 20 carbon atoms b)can be produced by structures of urogenital ridge origin c)are mostly activated in the liver d)are predominantly excreted unchanged in the urine e)mainly circulate unbound to carrier proteins Ventilation is increased due to stimulation of central receptors by a)nikethamide b)hypoxia c)doxapram d)phenobarbitone e)salbutamol Tetrahydrofolic acid a)is involved in purine synthesis b)is a precursor of folic acid c)is a coenzyme in amino acid synthesis d)catalyses the conversion of glucose to glucose-6-phosphate e)activity is inhibited by Methotrexate Mitochondrial DNA a)is located in the nucleus b)inheritance is patrilineal c)is present in two copies per cell d)mutation causes cystic fibrosis e)is involved in the control of oxidative phosphorylation The following conditions may lead to hydronephrosis

a)mercury poisoning b)cervical carcinoma c)renal calculi d)renal vein thrombosis e)posterior urethral valves In uncomplicated homozygous beta thalassaemia there is a)hypochromasia b)a reduction in haemoglobin A2 c)an increase in haemoglobin F d)no depletion of iron stores e)the presence of megaloblasts in bone marrow The following statements relate to lung function in normal pregnancy a)vital capacity is increased by about 50% b)tidal volume is increased c)the subcostal angle increases d)the residual volume is reduced e)the respiratory rate is increased In normal pregnancy, uterine blood flow a)is about 50ml/minute at term b)is maintained throughout the cardiac cycle to the choriodecidual space c)is reduced by prostacyclin d)is increased during uterine contractions e)represents about 10% of the cardiac output by the end of the first trimester During normal pregnancy a)arterial pCO2 decreases b)the blood hydrogen ion concentration decreases c)plasma bicarbonate concentrations decrease d)urine pH falls e)lactic acid production is increased The anal canal a)has an upper part which is innervated by the inferior hypogastric plexus b)has a lower part which is supplied by the superior rectal artery c)drains lymph to the superficial inguinal nodes from its upper part d)has its internal sphincter innervated by the inferior rectal nerve e)has a superficial part of its external sphincter attached to the coccyx. Human placental lactogen

a)is a single chain polypeptide b)reaches the same concentration in fetal and maternal blood at term c)may be secreted by the decidua d)is detectable only after the 25th week of pregnancy e)is an insulin antagonist Human chorionic gonadotrophin !! a)is a glycoprotein b)is detectable 48 hours after fertilisation occurs c)secretion peaks at 20 weeks of gestation d)is synthesised by the corpus luteum of pregnancy e)binds to luteinizing hormone receptors Cholecalciferol (vitamin D) a)promotes the absorption b)is 25-hydroxylated in the liver c)is synthesised in the SKIN d)is 1-hydroxylated in the kidney e)is most active in the 1,25-dihydroxyl form Unconjugated bilirubin !! a)is normally present in the plasma in lower concentration than conjugated bilirubin b)circulates in the plasma bound to albumin c)is not excreted in the urine d)does not cross the blood-brain barrier e)crosses the placenta The following are capable of cellular regeneration: a)spinal cord b)liver c)epidermis d)myocardium e)bone marrow The Leydig cells of the testis !! a)secrete seminal fluid b)are stimulated by luteinising hormone c)are active in intrauterine life d)secrete fructose e)produce androstenedione Angiotensin II !!

a)is the most potent vasoconstrictor b)reduces aldosterone production c)is mainly found in the lungs d)is a decapeptide e)is produced when the extracellular fluid volume is reduced Cyproterone acetate a)is an oestrogen b)is used for the treatment of amenorrhoea c)binds to androgen receptors d)increases libido e)inhibits spermatogenesis Lignocaine used as a local anaesthetic !! a)causes tachycardia if given as a systemic injection b)has a longer lasting action than bupivicaine c)is used in combination with adrenaline for ring block d)causes vasoconstriction e)is a weak base Plasma osmolarity in the human a)is normally about 290 milliosmoles per kg in the nonpregnant state b)increases during the first trimester of pregnancy c)is closely controlled by plasma protein concentration d)is regulated by arginine vasopressin e)regulates the sensation of thirst In the statistical analysis of any group of numerical observations a)the mean is always less than the mode b)the median value always lies at the mid-point of the range c)standard deviation is always greater than the standard error of the mean d)the standard error of the mean is independent of the total number of observations e)there are the same number of observations greater than and less than the median value The pelvic splanchnic nerves a)are derived from the posterior rami of the sacral spinal nerves b)supply afferent fibres c)mix with branches of the sympathetic pelvic plexus d)supply the ascending colon with motor fibres e)supply the uterus with parasympathetic fibres

Exotoxins a)are derived from gram-negative bacteria b)have specific action c)are more toxic than endotoxins d)are neutralised by their homologous antitoxin e)can be converted to toxoid Candida albicans a)is gram positive b)is an anaerobic organism c)is associated with diabetes mellitus d)is motile e)is inhibited by oral tetracycline therapy

Transferrin a)is one third saturated with iron b)is increased in pregnancy c)binds to 10mg of iron per gram d)levels in the neonate are low e)is actively transported In Crohns disease there is a) non casesous granuloma formation b) formation of deep fissures c) formation of crypt abscesses d) the presence of mucosal polyps e) a recognised association with occurence in the vulva The rectum a) usually commences at the level of S3 b) has a mesentery in its proximal third c) drains lymph to the pre-aortic nodes d) has a parasympathetic nerve supply derived entirely form the S3 spinal segment e) is about 25cm in length The female urethra (!!) a) traverses the perineal membrane b) is lined throughout by urothelium c) has a muscle layer continuous with that of the bladder d) has an external sphincter supplied by the obturator nerve

e) corresponds developmentally to the membranous urethra in the male During menstruation, endometrial haemostasis depends upon a) a platelet and fibrin plug formation after the first 20 hours of bleeding b) oestrogen mediated vasoconstriction c) prostaglandin mediated spiral vessel constriction d) production of plasminogen activating factor by the glandular epithelial cells In HIV, seroconversion illness after a needlestick injury a)occurs within 1 to 4 weeks following exposure b)infectivity decreases after seroconversion c)the CD4 count is a useful adjunct in diagnosis d)measurement of HIV RNA viral load is most useful in diagnosis The following tumours arise in the ovary: a) nephroblastoma b) cystadenoma c) granulosa cell tumour d) neuroblastoma e) teratoma Early blood borne dissemination is a characteristic feature of: a) carcinoma of the endometrium b) osteosarcoma c) basal cell carcinoma d) carcinoma of the cervix e) choriocarcinoma Breast milk compared to cow's milk (!!) a) has more protein b) has more sodium c) contains more carbohydrate d) has more casein e) contains more calories The following are consequences of pulmonary embolism: a) pulmonary infarction b) fibrinous pleurisy c) right ventricular hypertrophy d) sudden death e) haemoptysis

In a country wide survey, 10 general practices were picked at random and 5% of patients were selected at random from each practice: a) the sample of patients is a true random sample b) all practices had an equal chance of selection c) all patients had an equal chance of selection d) two siblings could not have been selected e) inferences about all patients in the country may be drawn from the sample Nitric oxide a) is synthesised in the endothelium b) has a short half life c) causes smooth muscle contraction d) increases during pregnancy e) combines with oxygen to produce L-arginine Carbohydrates a) yield 17kj/g of energy b) are a major source of energy for the brain c) are a a major component of the diet d) give a higher energy yield compared to fats Head mesenchyme gives rise to the following skull bones a)Parietal b)Mandible c)Maxillary d)Frontal e)Ethmoid Pre-implantation genetic diagnosis a) can be used to identify single gene defects b) is used in ICSI only c) testing for trisomy 21 should be offered in a case of ovum donation with a 45 year old recipient and 23 year old donor d) is routinely used in IVF in selection of embryos e) is governed by the regulations of the Human, Embryology and Fertilisation Authority The following are gram positive organisms a)Brucella b)Listeria c)Staphylococcus d)Vibrio cholera

Endometriotic deposits a) only occur in the pelvis b) consist of deposits of endometrial stromal tissue without glands c) do not occur in postmenopausal women d) are hormone sensitive Amyloidosis a)Can be inherited b)Is associated with bronchiectasis c)Is a cause of cardiomyopathy Phenylketonuria a)can only be diagnosed in the adult b)is diagnosed by high levels of phenylpyruvate in the urine c)treatment is with diet restriction DUB (Dysfunctional uterine bleeding) a) is common at the extremes of life b) may be caused by coagulation disorders Chylomicrons a) are not normally present in the fasting state b) after hydrolysis of lipoproteins, mainly consist of phospholipid Other questions that I cannot remember in such detail were about: Insulin (*half life of 30min) Pituitary gland Anatomy (*contains pars tuberalis) Many Pathology questions (equivalents ie *ovarian dysgerminoma = seminoma?, *endodermal sinus tumour = yolk sac tumour?) Embryology (notochord, primitive streak, mesonephros derivatives) Ionising radiation (radiosensitivity ie *liver /intestinal epithelium - which is more radiosensitive; *is radiation exposure of dental x-ray equivalent to 1/5 that from a transatlantic flight) difference b/w human milk and cows milk casien lactose sodium ascorbic acid water phenyl ketoneuria

inherited disorder cause by defficiency of phenyl alanine dietry restriction of tyrosine helpful diagnosed by measuring phenyl ketone level in urine essential amino acids are not synthesized by body sufficiently all ketogenic can produce energy dont remember exact wording radiation mostly harmful in 8-12 wks gestation xrays r ionizing radiation contraindicated in pregnancy transatlantic flight hv higher dose of radiation than xray MRI repeat q from sept paper chylomicrins responsible for turbulence of plsma?dont remem metabolized in adipose tissue -true 80% 20% in liver after hydrolysis by lipoprotien mainly phospholipids r absorbed form intestinal luminal cells to ----? ovarian tumors dysgerminoma and endodermal sinus tumor hv same origin choriocharcinoma can arise cocp increase risk of ovarian cancers -this qs repeated again in other qs with different wording b cells arise from plasma cells in pregnancy humoral immunity is depressed early in pregnany tumor necrotic factor alpha raised in parturition pulmonary embolism cause s sudden death fibrinous plurisy rt ventricular hypertrohy haemoptysis repeated from sept 05 transferrin endo metrium hiv seroconversion carbohydrate preimplantation diagnosis repeat from march 2005 progesteron following rise in leuteal phase

basal body temp progesterone LH--PLZ SOME ONE ADD I DONT REMEMBER NOTOCHORD R SOLID CELLS GIVE RISE TO brain --? SKULL MESENCHYME GIVE RISE TEMPORAL BONE FRONTAL PARIETAL Q-Disinfectant solutions may become contaminated with: 1-Enterobacter species 2-Streptococcus species 3-Escherichia coli 4-Pseudomonas aeruginosa 5-Staphyloccus pyogenes Q-Examples of active transport across a membrane include the passage of: 1-Potassium ions into the neurons 2-Water into the proximal renal tubular cell 3-Glucose from the proximal renal tubular cell 4-Fatty acids into the mucosal cells of the intestine 5-Hydrogen ions from the gastric oxyntic cells. ENDOMETRIOSIS has endometrial tissue with no glands not occurs after menopause The following values fall within the normal range for the adult female bladder Fa)residual volume of 100ml Tb)voiding capacity of 250ml Fc)bladder capacity of 900ml Td)intravesical pressure rise of less than 10cm H2O during early filling Fe)maximum urine flow rate of 60ml per second

The germination of tetanus spores in a wound is inhibited by Fa)tissue trauma Tb)oxygen Fc)injection of anti-toxin Td)injection of toxoid Te)removal of devitalised tissue In DNA Ta)a codon is a sequence of three bases Tb)all codons have an identified function Fc)there is a greater variety of amino acids than there are different codons Td)replication can be initiated at several different points along a chromosome Fe)complementary pairing precedes messenger mRNA synthesis Concerning ovarian function Fa)progesterone is the major steroid of the developing follicle Tb)granulosa cells secrete oestradiol Tc)oestradiol is derived from androgen precursors Td)insulin-like growth factor (IGF-1) is not secreted by the ovary Fe)circulating inhibin concentrations are a marker of granulosa cell function Actinomyces israelii Fa)is a rickettsia Tb)forms yellow granules in pus Tc)is a commensal in the mouth Fd)is a commensal in the vagina Fe)is usually resistant to penicillin In the small intestine, the following substances are absorbed by active processes F a)water T b)sodium F c)vitamin K T d)amino acids T e)chloride White cell migration from blood vessels in areas of inflammation involves T a)cell migration occurring between endothelial cells F b)a passive loss of fluid blood elements

T c)cell migration independent of endothelial cell motion T d)initial emigration of polymorphonuclear neutrophils F e)more polymorphs that monocytes after 2 days Which are formed from intramembranous ossification? T Temporal , occipital, parietal F. Sphenoid, Ethmoid Antibodies (!!) Ta)are soluble proteins -- Ab are glycoprotein. Are they soluble? not sure [color=red:c2901df9c1][b:c2901df9c1]F[/b:c2901df9c1][/color:c2901df9 c1]b)are formed in the fetus before 12 weeks of intrauterine life Fc)have an average molecular weight of around 10000 daltons Fd)of the rhesus type are genetically transmitted Te)are produced by the ribosomes of plasma cells Correction The germination of tetanus spores in a wound is inhibited by Fa)tissue trauma Tb)oxygen [color=red:1e03073b8c][b:1e03073b8c]T[/b:1e03073b8c][/color:1e0307 3b8c]c)injection of anti-toxin Td)injection of toxoid Te)removal of devitalised tissue Insulin (*half life of 30min) F Pituitary gland Anatomy (*contains pars tuberalis)F Q-Disinfectant solutions may become contaminated with: T1-Enterobacter species F2-Streptococcus species F3-Escherichia coli T4-Pseudomonas aeruginosa F5-Staphyloccus pyogenes The inferior vena cava Ta)is formed at the level of the fifth lumbar vertebra Fb)commences posterior to the right external iliac artery Fc)receives the left ovarian vein

Td)receives the right ovarian vein Te)pierces the central tendon of the diaphragm In the normal human pelvis Ta)the promontory of the sacrum is in the upper anterior border of the first sacral vertebra Fb)the anterior surface of the sacrum has five paired foramina Fc)the joint between the two pubic bones is a synovial joint [b:715c81bcd5]d)the acetabular fossa is wholly formed from parts of the pubic and ischial bones [/b:715c81bcd5] Te)the transverse diameter of the brim is greater than the anteroposterior diameter Arginine vasopressin Fa)reduces the glomerular filtration rate Fb)controls water loss in the proximal renal tubule Fc)is synthesised by the posterior pituitary gland Td)is released in response to a rise in plasma osmolality Te)is released in response to a fall in circulating plasma volume Concerning ovarian function Fa)progesterone is the major steroid of the developing follicle Tb)granulosa cells secrete oestradiol Tc)oestradiol is derived from androgen precursors Td)insulin-like growth factor (IGF-1) is not secreted by the ovary Fe)circulating inhibin concentrations are a marker of granulosa cell function Actinomyces israelii Fa)is a rickettsia Tb)forms yellow granules in pus Tc)is a commensal in the mouth Fd)is a commensal in the vagina Fe)is usually resistant to penicillin Halothane produces Ta)cardiac arrhythmias

Fb)explosive mixtures with air Tc)liver damage if given repeatedly Td)myometrial relaxation Fe)bronchial irritation The therapeutic effect of the first drug is enhanced by the second drug Fa)phenytoin: ethinyloestradiol Fb)bromocryptine: metoclopramide Tc)penicillin: probenicid Td)ritodrine: dexamethasone Fe)warfarin: phenobarbitone Potassium Ta)is mainly intracellular Fb)plasma levels vary in proportion to intracellular levels Fc)plasma levels are decreased in Addisons disease Td)plasma levels are increased in diabetic ketoacidosis Te)deficiency occurs with prolonged vomiting Concerning carbohydrates Ta)sucrose is a disaccharide of glucose and fructose [color=red:715c81bcd5][b:715c81bcd5]F[/b:715c81bcd5][/color:715c81b cd5]b)cereal grains contain less than 40% starch [color=red:715c81bcd5][b:715c81bcd5]F[/b:715c81bcd5][/color:715c81b cd5]c)cellulose is a fructose polysaccharide Fd)a normal diet contains less than 60g of carbohydrate daily Te)dietary carbohydrate is oxidised in the body to carbon dioxide and water Steroid hormones Fa)all contain 20 carbon atoms Tb)can be produced by structures of urogenital ridge origin Fc)are mostly activated in the liver Fd)are predominantly excreted unchanged in the urine Fe)mainly circulate unbound to carrier proteins Ventilation is increased due to stimulation of central receptors by

Ta)nikethamide Tb)hypoxia Tc)doxapram Fd)phenobarbitone Fe)salbutamol Tetrahydrofolic acid Ta)is involved in purine synthesis Fb)is a precursor of folic acid Fc)is a coenzyme in amino acid synthesis Fd)catalyses the conversion of glucose to glucose-6-phosphate Te)activity is inhibited by Methotrexate Mitochondrial DNA Ta)is located in the nucleus Fb)inheritance is patrilineal Fc)is present in two copies per cell Fd)mutation causes cystic fibrosis Te)is involved in the control of oxidative phosphorylation The following conditions may lead to hydronephrosis ?Fa)mercury poisoning Tb)cervical carcinoma Tc)renal calculi ?Fd)renal vein thrombosis Te)posterior urethral valves In uncomplicated homozygous beta thalassaemia there is Ta)hypochromasia Fb)a reduction in haemoglobin A2 Tc)an increase in haemoglobin F Td)no depletion of iron stores Fe)the presence of megaloblasts in bone marrow The following statements relate to lung function in normal pregnancy Fa)vital capacity is increased by about 50% (ans from busyspr) Tb)tidal volume is increased

Tc)the subcostal angle increases Td)the residual volume is reduced Fe)the respiratory rate is increased [color=darkblue:715c81bcd5]In normal pregnancy, uterine blood flow Fa)is about 50ml/minute at term Tb)is maintained throughout the cardiac cycle to the choriodecidual space Fc)is reduced by prostacyclin Fd)is increased during uterine contractions Te)represents about 10% of the cardiac output by the end of the first trimester [/color:715c81bcd5] P.200 sep 2001 paper 2 Q 46 During normal pregnancy Ta)arterial pCO2 decreases Fb)the blood hydrogen ion concentration decreases Tc)plasma bicarbonate concentrations decrease Fd)urine pH falls ?Te)lactic acid production is increased The anal canal Ta)has an upper part which is innervated by the inferior hypogastric plexus Fb)has a lower part which is supplied by the superior rectal artery Fc)drains lymph to the superficial inguinal nodes from its upper part Fd)has its internal sphincter innervated by the inferior rectal nerve Te)has a superficial part of its external sphincter attached to the coccyx. Human placental lactogen ?T[b:715c81bcd5]a)is a single chain polypeptide [/b:715c81bcd5] Fb)reaches the same concentration in fetal and maternal blood at term Tc)may be secreted by the decidua ?F[b:715c81bcd5]d)is detectable only after the 25th week of pregnancy[/b:715c81bcd5] ?T[b:715c81bcd5]Te)is an insulin antagonist [/b:715c81bcd5]

Human chorionic gonadotrophin !! Ta)is a glycoprotein ?T[b:715c81bcd5]b)is detectable 48 hours after fertilisation occurs [/b:715c81bcd5] Fc)secretion peaks at 20 weeks of gestation Fd)is synthesised by the corpus luteum of pregnancy Te)binds to luteinizing hormone receptors Cholecalciferol (vitamin D) Ta)promotes the absorption Tb)is 25-hydroxylated in the liver Tc)is synthesised in the SKIN Td)is 1-hydroxylated in the kidney Te)is most active in the 1,25-dihydroxyl form Unconjugated bilirubin !! ?Fa)is normally present in the plasma in lower concentration than conjugated bilirubin ?Tb)circulates in the plasma bound to albumin Fc)is not excreted in the urine Fd)does not cross the blood-brain barrier ?Te)crosses the placenta The following are capable of cellular regeneration: Fa)spinal cord Tb)liver Tc)epidermis Fd)myocardium Te)bone marrow The Leydig cells of the testis !! Fa)secrete seminal fluid Tb)are stimulated by luteinising hormone Tc)are active in intrauterine life Fd)secrete fructose ?Fe)produce androstenedione

Angiotensin II !! Ta)is the most potent vasoconstrictor Fb)reduces aldosterone production Tc)is mainly found in the lungs Fd)is a decapeptide -- angiotensin I is a decapeptide Te)is produced when the extracellular fluid volume is reduced Cyproterone acetate Fa)is an oestrogen [color=blue:715c81bcd5]--is an antiandrogenic progesterone[/color:715c81bcd5] Fb)is used for the treatment of amenorrhoea [color=blue:715c81bcd5]-treat hirsutism [/color:715c81bcd5] Tc)binds to androgen receptors Fd)increases libido Te)inhibits spermatogenesis Lignocaine used as a local anaesthetic Ta)causes tachycardia if given as a systemic injection Fb)has a longer lasting action than bupivicaine Fc)is used in combination with adrenaline for ring block Fd)causes vasoconstriction Te)is a weak base Plasma osmolarity in the human Ta)is normally about 290 milliosmoles per kg in the nonpregnant state Fb)increases during the first trimester of pregnancy ?Fc)is closely controlled by plasma protein concentration Td)is regulated by arginine vasopressin Te)regulates the sensation of thirst In the statistical analysis of any group of numerical observations Fa)the mean is always less than the mode Tb)the median value always lies at the mid-point of the range Tc)standard deviation is always greater than the standard error of the mean Fd)the standard error of the mean is independent of the total number of observations

Te)there are the same number of observations greater than and less than the median value The pelvic splanchnic nerves Fa)are derived from the posterior rami of the sacral spinal nerves Fb)supply afferent fibres Tc)mix with branches of the sympathetic pelvic plexus Fd)supply the ascending colon with motor fibres Te)supply the uterus with parasympathetic fibres Exotoxins Ta)are derived from gram-negative bacteria [color=red:715c81bcd5][b:715c81bcd5]T[/b:715c81bcd5][/color:715c81 bcd5]b)have specific action Tc)are more toxic than endotoxins Td)are neutralised by their homologous antitoxin Te)can be converted to toxoid Candida albicans Ta)is gram positive Fb)is an anaerobic organism Tc)is associated with diabetes mellitus Fd)is motile Fe)is inhibited by oral tetracycline therapy T treat with miconazole T commonsal in intestine

Transferrin Ta)is one third saturated with iron c)binds to 10mg of iron per gram d)levels in the neonate are low e)is actively transported Increased in pregnancy t

In Crohns disease there is ?Ta) non casesous granuloma formation Tb) formation of deep fissures Tc) formation of crypt abscesses Td) the presence of mucosal polyps e) a recognised association with occurence in the vulva The rectum Ta) usually commences at the level of S3 Fb) has a mesentery in its proximal third [b:715c81bcd5][color=red:715c81bcd5]T[/color:715c81bcd5][/b:715c81 bcd5]c) drains lymph to the pre-aortic nodes [color=red:715c81bcd5][b:715c81bcd5]T[/b:715c81bcd5][/color:715c81 bcd5]d) has a parasympathetic nerve supply derived entirely form the S3 spinal segment [color=red:715c81bcd5][b:715c81bcd5]F[/b:715c81bcd5][/color:715c81b cd5]e) is about 25cm in length [b:715c81bcd5]The female urethra (!!) a) traverses the perineal membrane b) is lined throughout by urothelium c) has a muscle layer continuous with that of the bladder d) has an external sphincter supplied by the obturator nerve e) corresponds developmentally to the membranous urethra in the male [/b:715c81bcd5] [b:715c81bcd5]During menstruation, endometrial haemostasis depends upon a) a platelet and fibrin plug formation after the first 20 hours of bleeding b) oestrogen mediated vasoconstriction Tc) prostaglandin mediated spiral vessel constriction d) production of plasminogen activating factor by the glandular epithelial cells [/b:715c81bcd5] During menstruation endometrial hemostasis depends upon Platelet and fibrin plug formation after first 20 hrs of bleeding Production of PAF by glandular epithelial cells

Estrogen mediated hemostasis PG mediated hemostasis

In HIV, seroconversion illness after a needlestick injury Ta)occurs within 1 to 4 weeks following exposure [color=darkblue:715c81bcd5][b:715c81bcd5]?T[/b:715c81bcd5][/color:7 15c81bcd5]b)infectivity decreases after seroconversion Tc)the CD4 count is a useful adjunct in diagnosis Td)measurement of HIV RNA viral load is most useful in diagnosis HIV seroconversion Illness occurs witin 1-4 wks......T...........SEROCONVERSION OCCURS IN FIRST FEW WEEKS......FOLLOWED BY SIGNS OF ILLNESS After illness infectivity decreases .........[b:715c81bcd5][color=darkblue:715c81bcd5].F.........[/color:715c81 bcd5][/b:715c81bcd5]I THINK SO BECAUSE THOUGH ITS A LATENT PHASE AND NO OVERT SYMPTOMS OCCUR WHILE THE VIRUS MULTIPLIES, I THINK INFECTIVITY REMAINS JUST THE SAME......... The best available diagnostic test is HIV viral load assay........[color=red:715c81bcd5][b:715c81bcd5]T[/b:715c81bcd5][/col or:715c81bcd5].........FROM AN OBSTETRIC POINT OF VIEW.....VIRAL LOAD IS THE ONLY USEFUL WAY OF PREDICTING DISEASE......BECAUSE SEROLOGY IS UNRELIABLE IN ITS RESULTS DUE TO MATERNAL ANTIBODIES......WHICH MIGHT TAKE UPTO 18MONTHS TO DIE DOWN... Without any intervention during pregnancy the mother to child transmission rate is 80% .........[color=red:715c81bcd5][b:715c81bcd5]F[/b:715c81bcd5][/color:71 5c81bcd5].....ITS 15-45%......LATTER IN UNTREATED CASES...... Mode of delivery by LSCS decreases the transmission to baby......[color=red:715c81bcd5][b:715c81bcd5]T[/b:715c81bcd5][/color :715c81bcd5].....CAN BRING THE TRANSMISS DOWN TO 2% Short course of anti viral Rx during labor decreases transmission to baby.........[b:715c81bcd5][color=red:715c81bcd5]T[/color:715c81bcd5][/ b:715c81bcd5]

BF is safe ........[b:715c81bcd5][color=red:715c81bcd5]F[/color:715c81bcd5][/b:71 5c81bcd5].......MOST IMP METHOD OF TRASMISS IF OTHER PREVENTIVE MEASURES HAVE BEEN UNDERTAKEN....

The following tumours arise in the ovary: Fa) nephroblastoma Tb) cystadenoma Tc) granulosa cell tumour Fd) neuroblastoma Te) teratoma Early blood borne dissemination is a characteristic feature of: Fa) carcinoma of the endometrium [color=red:715c81bcd5]T[/color:715c81bcd5]b) osteosarcoma Fc) basal cell carcinoma Fd) carcinoma of the cervix Te) choriocarcinoma Breast milk compared to cow's milk (!!) a) has more protein Tb) has more sodium c) contains more carbohydrate d) has more casein e) contains more calories difference b/w human milk and cows milk Tlactose Tsodium ascorbic acid Twater PS:human milk contains more water The following are consequences of pulmonary embolism: Ta) pulmonary infarction Tb) fibrinous pleurisy

?Fc) right ventricular hypertrophy Td) sudden death Te) haemoptysis [color=blue:715c81bcd5]In a country wide survey, 10 general practices were picked at random and 5% of patients were selected at random from each practice: Ta) the sample of patients is a true random sample Tb) all practices had an equal chance of selection Tc) all patients had an equal chance of selection Fd) two siblings could not have been selected Te) inferences about all patients in the country may be drawn from the sample [/color:715c81bcd5] ans from onexamination Nitric oxide Ta) is synthesised in the endothelium Tb) has a short half life Fc) causes smooth muscle contraction Td)production is increases during pregnancy [b:715c81bcd5]e) combines with oxygen to produce L-arginine [/b:715c81bcd5] T generated by arginine (from onexamination) Carbohydrates a) yield 17kj/g of energy b) are a major source of energy for the brain ?Tc) are a a major component of the diet Fd) give a higher energy yield compared to fats Head mesenchyme gives rise to the following skull bones a)Parietal b)Mandible c)Maxillary d)Frontal e)Ethmoid Pre-implantation genetic diagnosis

Ta) can be used to identify single gene defects Fb) is used in ICSI only Fc) testing for trisomy 21 should be offered in a case of ovum donation with a 45 year old recipient and 23 year old donor Fd) is routinely used in IVF in selection of embryos ?T[b:715c81bcd5]e) is governed by the regulations of the Human, Embryology and Fertilisation Authority[/b:715c81bcd5] The following are gram positive organisms ?Fa)Brucella Tb)Listeria Tc)Staphylococcus ?Fd)Vibrio cholera Endometriotic deposits Fa) only occur in the pelvis ?Tb) consist of deposits of endometrial stromal tissue without glands ?Fc) do not occur in postmenopausal women ?Td) are hormone sensitive Amyloidosis Ta)Can be inherited Tb)Is associated with bronchiectasis Tc)Is a cause of cardiomyopathy (ref: Familial Amyloidosis Familial amyloidosis, or ATTR, is a rare form of inherited amyloidosis. The amyloid deposits in familial amyloidosis are composed of the protein transthyretin, or TTR, which is made in the liver. Familial amyloidosis is inherited an [color=darkblue:715c81bcd5]autosomal dominant [/color:715c81bcd5]in genetics terminology) http://www.ncbi.nlm.nih.gov/entrez/query.fcgi?cmd=Retrieve&db=PubM ed&list_uids=12472203&dopt=Abstract http://www.emedicine.com/med/topic3365.htm When the heart is involved heavily but the nerves are not, the disease is called familial amyloid cardiomyopathy (FAC).

Phenylketonuria Fa)can only be diagnosed in the adult Tb)is diagnosed by high levels of phenylpyruvate in the urine ?Tc)treatment is with diet restriction ?Tinherited disorder Fcause by defficiency of phenyl alanine ( defficiency of phenylalanine hydroxylase) Fdietry restriction of tyrosine helpful (restriction of phenylalanine) ?Fdiagnosed by measuring phenyl ketone level in urine DUB (Dysfunctional uterine bleeding) Ta) is common at the extremes of life Tb) may be caused by coagulation disorders

Insulin (*half life of 30min) T Pituitary gland Anatomy (*contains pars tuberalis) F Many Pathology questions (equivalents ie *ovarian dysgerminoma = seminoma?, *endodermal sinus tumour = yolk sac tumour?) Embryology (notochord, primitive streak, mesonephros derivatives) NOTOCHORD R SOLID structure as it forms --T GIVE RISE TO brain -F F is formed from primitive streak (ans from mrcogexam) [color=blue:715c81bcd5][b:715c81bcd5]F fuses temporally with the endoblast of yolk sac[/b:715c81bcd5][/color:715c81bcd5] Ionising radiation (?T radiosensitivity ie *liver /intestinal epithelium - which is more radiosensitive; *is radiation exposure of dental x-ray equivalent to 1/5 that from a transatlantic flight)

essential amino acids Fare not synthesized by body sufficiently ?Tall ketogenic ?Tcan produce energy dont remember exact wording [b:715c81bcd5]Are ONLY used for the synthesis or important body proteins. Canot be catabolised for energy.[/b:715c81bcd5] radiation ?Tmostly harmful in 8-12 wks gestation Txrays r ionizing radiation Tcontraindicated in pregnancy [b:715c81bcd5]transatlantic flight hv higher dose of radiation than xray [/b:715c81bcd5] Biophysics: Radiation effect more in hypoxic tissue..false Bone, peripheral nerve and muscle are senstive to radiation..false Intestine and bone marrow are sestive to radiation..true

chylomicrins Ta) are not normally present in the fasting state ?T[b:715c81bcd5]responsible for turbulence of plsma?[/b:715c81bcd5]dont remem [color=darkblue:715c81bcd5]metabolized in adipose tissue [b:715c81bcd5]true [/b:715c81bcd5]80% 20% in liver [/color:715c81bcd5] Fafter hydrolysis by lipoprotien mainly phospholipids ?F r absorbed form intestinal luminal cells to ----? (ref: Monoglycerides and free fatty acids associate with bile salts and lecithin to form micelles * The core of the micelle is composed of cholesterol and fat soluble vitamins * Monoglycerides, cholesterol and free fatty acids are absorbed by passive diffusion across the duodenal and jejunal mucosa while the bile salts

remain in the lumen and are absorbed in the terminal ileum Chylomicrons have a core of triglycerides and cholesterol and a coat of protein and phospholipids

ovarian tumors Tchoriocharcinoma can arise cocp F increase risk of ovarian cancers Pill failure: tetracyclineT Rifmpicintrue digoxin....F phenobarbitone....T Contraindicated in breast feeding: cascara.....T phenophthalien...F norethisterone.....F chloramphenicol...T sulphonamides......T

b cells Farise from plasma cells in pregnancy humoral immunity is depressed early in pregnany Ftumor necrotic factor alpha raised in parturition pulmonary embolism cause s : Tsudden death Tfibrinous plurisy

Frt ventricular hypertrohy Thaemoptysis following rise in leuteal phase Tbasal body temp Tprogesterone

SKULL MESENCHYME GIVE RISE TEMPORAL BONE FRONTAL PARIETAL Q-Examples of active transport across a membrane include the passage of: ?T1-Potassium ions into the neurons F2-Water into the proximal renal tubular cell --F ?T3-Glucose from the proximal renal tubular cell [color=red:715c81bcd5][b:715c81bcd5]F[/b:715c81bcd5][/color:715c81b cd5]4-Fatty acids into the mucosal cells of the intestine 5-Hydrogen ions from the gastric oxyntic cells. (ref: glucose absorption in small intestine n renal tubules is active,all other places is facilitated using GLUT. also Na/K pump is active. Na absorption is active in renal tubules. Monoglycerides, cholesterol and free fatty acids are absorbed by passive diffusion across the duodenal and jejunal mucosa while the bile salts remain in the lumen and are absorbed in the terminal ileum )

ENDOMETRIOSIS has endometrial tissue with no glands ?Tnot occurs after menopause[b:715c81bcd5][/b:715c81bcd5] Regarding data

a The coefficient of variation expresses the Standard deviation as a proportion of the mean b Standard error of mean indicates how close a sample is to the population mean c A probability of one means that an event has one in ten chance of happening d Logarithmictransfoormation is a method of normalizing data that are not normally distributed e ANOVA is used for normally distributed data

Indomethacin can block ovulaion Pulsatile release of LH occurs in first weel of life RBCs possess the antigens of ABO system in fetus MRI involves the administration of radio labelled compound Doppler effect is used to detect movement Proton is injected to improve contrast during MRI

ventilation and stimulation of central chemoreceptors...someone said true for hypoxia. From my knowledge, the central chemoreceptors respond to pH and CO2, not O2, rather the PERIPHERAL aortic and carotid bodies respond to O2 and thus hypoxia.

sept.2005

Sigmoid colon

Attached to pelvic wall Lies lateral to psoas major Ends at S3 Rectum Starts at S3 No appendices epiploae Middle rectal artery is the major artery suuplying it 12 cm long T pallidum Spiral shape Causes yaws in children Stained with silver impregnation HIV seroconversion Illness occurs witin 1-4 wks After illness infectivity decreases The best available diagnostic test is HIV viral load assay Without any intervention during pregnancy the mother to child transmission rate is 80% Mode of delivery by LSCS decreases the transmission to baby Short course of anti viral Rx during labor decreases transmission to baby BF is safe Percutaneous mode of infection H simplex Rabies Hepatitis B Epstein barr virus Hep A mode of infection is through Blood transfusion Shared needles Faeco oral route Insect bites Air borne

Match the disease with their respective causative org Bone marrow suppression myco bacterium avium Red cell aplasia parvovirus

The foll are pre malignant CIN 3 Vulval lichen sclerosis Endometrioses Fibro epithelial polyp Atypical endometrial hyperplasia Bacterial vaginosis Microscopic finding clue cells pH less than 4.5 best Rx with ampicillin sensitive to clindamycin may be due to gardnerella vaginalis vaginal infecton incidence of n gonorrhoea resistance to penicillin is 1% incidence of n gonorrhoea resistance to 3 rd gen ceph 5% azithromycin DOC for trichomoniasis crytococcus neoformans causes meningitis C trachomatis Strains L123 cause LGV Reticulate body is the infectious form Transferrin Increased in pregnancy One third saturated with Fe Binds to 10 mg of iron per gm Levels in neonate are low Antibodies

Are soluble proteins Formed before 12 wks in fetus ABO antigens are present in fetus Foll Hb chains are present in intrauterine life Alpha 2 beta2 Alpha 2 gamma 2 Gamma 4 Alpha 2 epsilon 2 Zeta 2 epsilon2 Pulm embolism Spiral CT scan is indicated in suspected cases DVT always causes pulm embolism X ray angiography is diagnostic Spiral CT involves dignoses based on the perfusion copared to air in lungs ECG is reliable for diagnosis MRI Involves administerind of radio contrast agents Protons give better imaging Damage to fetus is due to tissue heating Xray Contraindicated in pregnancy Low rays are used for breast imaging Conventional USG Increased freq gives higher resolution Increased freq gives better attenuation TVS uses higher freq compared to conventional USG Doppler USG measures movement Gives accurate measurement of bld flow in fetal vessels Doppler effect based on blood flow within vessels

Calcification occurs in Renal calculi Hyaline degeneration of fibroid Parathyroid adenomas Secondary bone deposits in prostratic CA Foll originate from endothelial cells Nitric oxide Protein C Fibronectin Plasminogen activator Thyroid 100-200 micro gms are trapped daily increases in size in pregnancy trapsinorganic iodine colloid is stored outside epithelial cells TBG is increased n pregnancy 5% is bound to CBG ketone bodies formed from acetone formed from acetyl CoA not formed during starvation not utilised by brain Mgso4 Potentiates action of non depolarising agents Check respiratory status Causes brisk patellar reflexes In toxicity Rx IV calcium sulphate

CIN CIN 3 does not involve breach in basement epithelial layer Arias stella reaction mimicks clear cell CA Tissue biopsy is required for diagnosis

Incidence is decreased in renal transplant patients GH Stmulated by glucose infusion Increased in sleep Opposes insulin action Causes positive nitrogen balance Decreased in pregnancy Foll form boundaries of ovarian fossa Ureter Ext iliac vn Int iliac art Int pudendal art Obliterated umb art Penem antibiotics Active in beta lactam ring Drug of choice for ps aeroginoa Vagina Derived from mullerian ducts Seperated from anal canal by anal body Bladder Pelvic splanchnic stimulation contracts the trigone Ext urethral sphincter innervated by pudendal nv Mucosa of urethra is innervated by pudendal nv Transaction of spinal cord above S2 causes autonomous bladder Afferent innervation of glans of clitoris is by ilioinguinal nv Hypo k+ is found in Prolonged vomiting In diabetis ketoacidosis Addisons disease Foll are below inguinal ligament

Ilio inguinal nv Lateral branch of genito femoral nv Subcostal nv Femoral art Inf hypogastric (pelvic) plexus Supplies pain fibres to body of ut Continues as vesical plexus Lies at base of braod lig Sciatic nv S3&4 contribute towards it Lies behind quadratus femoris Tibial branch supplies short head of biceps femoris Injection on the outer & upper quadrant of the buttock may cause damage to the nv Undescended testis More common in premature than term infants Commonly assoc with gonadal neoplasia Asoc with 1% of absent testis More common on left side Embryo Parameso duct lies medial to mesonephros Mesonephros forms urine TVS detects fetal heart beat 21 days after implantation Erthropoiesis is entirely medullary

UG sinus Divides the cloaca coronally Urachus connects bladder to umbilicus Corpus luteum Is major source of relaxin Contains endothelial cells

Hysterectomy prolonges the life period 17 hydroxy prog is the main prog secreted steroids prevent recruitment of new follicles fertilisation acrosome fuses with zona pellucida polar body has same karyotype as ovum fetilised egg moves slower than unfertilised egg in fallopian tube inner ell mass lysis deciduas during implantation histo pathologic examination of tissue of conception can diagnose CIN 3 Cx incompetence Ectopic preg Gest Trophblastic Di Ring inversion of Y chromosome found the foll will be done Maternal chromosome is tested Paternal chromosome is tested Scan to look for male genitalia Anomaly scan and amniocentesis done for fetus Do karyotype of all previous children of couple Right ureter Below sigmoid coon Supplied by vaginal art Lined throughout by urothelium Contain glands Oral absorption of drug depends More availablibility in ionised form than non ionised Increased lipid solubility increases penetration in CNS Enters intestinal cells by simle diffusion Protein binding increases excretion Lipid form increases uptake Fetal circulation

Portal vn drains in IVC Ductus arteriosus takes blood to pulm art R atria and L atria communicate thro Foramen Ovale Voiding in females Occurs above rate of 20 ml/sec When vesical press is more than 45 mm H2O Initial contraction of trigone Enzyme activity is modified by Dephosphorylation Allosteric esters Binding to plasma prot Proteolytic cleaving of inactive precursors Gene transcription Functional cyst occurs in Stein levinthal synd PID Multiple sclerosis Gest tropho di Rx with clomiphene Amniotic fluid Hypotonic compared to fetal plasma Absored in lungs Unrelated to swallowing Increased until term in fetus Contains more HCO3 compared to fetal blood Action of complement Increase in permeability Migration of polymorphs Genetics South blot detects DNA DNA is exact copy of RNA

DNA is transcribed by RNA polymerase Ribosomes contain RNA Transcription occurs at any direction along the chain DNA replication is called transformation DNA has transcription restriction enzyme Histones do not contain DNA Insulin Binding to tyrosine receptor kinase increases activity Causes glucose transporter movement to plasma membrane Increases glycogen synthetase activity Increases fat deposition Increase glucose utilisation by CNS Heparin T is 1.5 hrs LMW heparin is administered 4 hrly Given in pregnancy causes intracranial haemorrhage in fetus Longer duration of action in S/C compared to IV administration Foll have imp natural reservoir other than human Listeria Saolmonella typhimurium Brucella Crohns disease Non caseous granulomatous infection Forms deep fissures Forms crypt abscesses Assoc with mucosal polyps Recognised to occur in vulva Assoc with intestinal wall thickening Absorption in kdny Glucose occurs in loop of henle Water is actively absorbed in proximal conv tubules

Estrogen 17 beta estradiol is formed by aromatisation of testos increases LH receptors secreted by theca cells granulosa cells secrete inhibin humoral immunity is depressed in pregnancy T & B cells are derived from bone marrow teratoma usually benign can occur is assoc with germ cell tumors secretes hormones

during menstruation platelet and fibrin plug is formed after 1st 20 hrs of bleeding hemostasis is thro estr mediated vaso constriction hemostasis is thro PG mediated spiral vessel constriction During ovulation Basal vacuoles occur in mid secretory phase Gland stromal mitosis maximum in mid secretory phase PG in follicle fluid during ovulation Increase of estr and prog receptors in proliferaive phase LDL receptors are increased in proliferative phase Indomethacin inhibits ovulation Matrixmetalloproteinases(MMC) is inhibited by progestrones Cloiphene can cause deficient secretory phase Glycogen Synthesis occurs in muscle Is stored along with water Carbohydrates Yield energy ?17 J/Kg Major source of energy for brain Major component of diet

Higher energy compared with fats Foll decrease the efficacy of OCPs Isoniazid Carbamaepine Phenytoin Rifampicin Female pelvis compared to male Sacrum more curved Greater sc notch is larger Sub pubic angle is more Distance betw pubic sym to acetabulum is less than the diam of acetabulum Vascular derived epithelial growth factor Causes vasodilatation of placental vessels Causes angiogenesis in placenta Is an angigogenic factor Increases permeability of endothelial cells And always save the best for last STATISTICS Definition LBW less than 1,500 gms PMR deaths in first 1 week per 1,000 births Perinatal mortality deaths after 22wks ( more than 154 days) until 1 wk after birth Maternal mortality rate Tests used ANOVA Std error of mean means diffce betw sample mean from population mean Probablity of 1 means 1 in 10 chance for event to occur Examples of ?continuous variables are Bld glucose

Haemoglobin Gender Height Blood pressure

Sigmoid colon Attached to pelvic wall......T Lies lateral to psoas major ........F Ends at S3 .........T SIGMOID COLON...STARTS AT PELVIC BRIM.....ENDS AT S3.......IS 45 CMS LONG.....LIES ON THE PERITONEAL SURFACE OF BLADDER AND UTERUS.....SO THE PSOAS IS A DEEPER STRUCTURE.....

Rectum Starts at S3 ........T No appendices epiploae.......T Middle rectal artery is the major artery suuplying it........F......ITS THE SUPERIOR RECTAL ARTERY 12 cm long .............T

T pallidum Spiral shape........T Causes yaws in children..........T Stained with silver impregnation.........F....... THE ONLY TESTS FOR MICROSCOPY ARE DARK GROUND

IMMUNOFLOURSCENCE................OTHERWISE SEROLOGY IS THE OTHER DIAGNOSTIC AID....

HIV seroconversion Illness occurs witin 1-4 wks......T...........SEROCONVERSION OCCURS IN FIRST FEW WEEKS......FOLLOWED BY SIGNS OF ILLNESS After illness infectivity decreases ..........F.........I THINK SO BECAUSE THOUGH ITS A LATENT PHASE AND NO OVERT SYMPTOMS OCCUR WHILE THE VIRUS MULTIPLIES, I THINK INFECTIVITY REMAINS JUST THE SAME......... The best available diagnostic test is HIV viral load assay........T.........FROM AN OBSTETRIC POINT OF VIEW.....VIRAL LOAD IS THE ONLY USEFUL WAY OF PREDICTING DISEASE......BECAUSE SEROLOGY IS UNRELIABLE IN ITS RESULTS DUE TO MATERNAL ANTIBODIES......WHICH MIGHT TAKE UPTO 18MONTHS TO DIE DOWN... Without any intervention during pregnancy the mother to child transmission rate is 80% .........F.....ITS 15-45%......LATTER IN UNTREATED CASES...... Mode of delivery by LSCS decreases the transmission to baby......T.....CAN BRING THE TRANSMISS DOWN TO 2% Short course of anti viral Rx during labor decreases transmission to baby.........T BF is safe ........F.......MOST IMP METHOD OF TRASMISS IF OTHER PREVENTIVE MEASURES HAVE BEEN UNDERTAKEN....

Percutaneous mode of infection H simplex........T Rabies .......F.....BITE WITH SALIVA Hepatitis B ........F...... Epstein barr virus .......T I THINK.......ECAUSE IT SAYS YOU CAN

GET IT FROM PATIENTS THROUGH "CASUAL CONTACT"......

Hep A mode of infection is through Blood transfusion.....F Shared needles .....F Faeco oral route ......T Insect bites ......F Air borne ......F Match the disease with their respective causative org Bone marrow suppression myco bacterium avium Red cell aplasia parvovirus

The foll are pre malignant CIN 3 ........T Vulval lichen sclerosis.......F BUT NOT SURE... Endometrioses .......F Fibro epithelial polyp ............T I THINK... Atypical endometrial hyperplasia ??T

Bacterial vaginosis Microscopic finding clue cells.........T pH less than 4.5 .......F........HAS TO BE MORE THAN 5 FOR BV TO OCCUR best Rx with ampicillin ............F sensitive to clindamycin ...........F.......I DIDNT FIND IT ANYWHERE.....SO I WILL GO FOR F

may be due to gardnerella vaginalis........T.....

vaginal infecton incidence of n gonorrhoea resistance to penicillin is 1% ......T BUT NOT SURE 100% incidence of n gonorrhoea resistance to 3 rd gen ceph 5% .....? FALSE azithromycin DOC for trichomoniasis ,.......F....METRO... crytococcus neoformans causes meningitis .........T.....ESP IN AIDS PTS

C trachomatis Strains L123 cause LGV.........T Reticulate body is the infectious form .......F.....RETICULATE I THE VEGETATIVE NON INF INTRACELL. FORM.... Transferrin Increased in pregnancy ........T One third saturated with Fe .........T Binds to 10 mg of iron per gm ???.....DUNNO....DIDNT FIND THIS KIND OF DATA ANYWHERE.... Levels in neonate are low........FALSE.....

Antibodies Are soluble proteins........I THINK ITS TRUE....THEY HAVE TO BE.....BUT NO BOOK CONFIRMATION YET..... Formed before 12 wks in fetus .........TRUE.....IGM FORMS AT 11 WEEKS.... ABO antigens are present in fetus........TRUE

Foll Hb chains are present in intrauterine life Alpha 2 beta2 .......TRUE......HB A......PRESENT 8TH MONTH ONWARS Alpha 2 gamma 2 ........T......HB F......ALL THROUGH PREG AND LATER TOO... Gamma 4 ......F Alpha 2 epsilon 2........F Zeta 2 epsilon2 ........T.....HBGOWER 1.......PRESENT I THE FIRST FEW WEEKS..... ALSO THERE IS HBA2....ALPHA2 AND DELTA 2 UNITS...

Pulm embolism Spiral CT scan is indicated in suspected cases.........F DVT always causes pulm embolism ............F X ray angiography is diagnostic ...............T Spiral CT involves dignoses based on the perfusion copared to air in lungs ...............F ECG is reliable for diagnosis............F IS SPIRAL CT AND VQ SCAN THE SAME THING????SORRY FOR ASKING SUCH A BASICALLY DUMB QUESTION....

MRI Involves administerind of radio contrast agents........T I THINK.... Protons give better imaging ?????T I THINK.........BECAUSE IT DEENDS ON THE OSCILLATION OF CHARGED PARTICLES......EPS THE HYDROGEN ATOM.... Damage to fetus is due to tissue heating.......F I THINK.....BECAUSE MRI IS NOT AN IONISING RADIATION.....SO I DONT THINK THERE IS ANY HEATING.....

Xray Contraindicated in pregnancy...............F Low rays are used for breast imaging ............????......LOW COMPARED TO WHAT??7 TO 10 MRADS ARE USED..........WHICH IS MORE THAN EG THE 2-3 MRADS FOR CHEST X RAY...........I DONT THINK THATS LOW.......I WOULD SAY FALSE.....

Conventional USG Increased freq gives higher resolution Increased freq gives better attenuation TVS uses higher freq compared to conventional USG Doppler USG measures movement ........T Gives accurate measurement of bld flow in fetal vessels........T Doppler effect based on blood flow within vessels ...........T......THE SOUND OF BLOOD FLOW WITHIN VESSELS....TO BE EXACT.... FOR THE FIRST THREE STEMS......I COULDNT FIND THE ANSWER......BUT IT WAS SOMEWHERE IN THIS FORUM.........I WILL POST IT AS SOON AS I COME ACROSS IT AGAIN.......

Calcification occurs in Renal calculi ...........T Hyaline degeneration of fibroid.........T.....CALCIFICATION OCURS IN FIBROID........I DONT WHAT THE SIGNIFICANCE OF HYALINE DEGENERATION IS IN TH QUESTION Parathyroid adenomas ............T Secondary bone deposits in prostratic CA ..........T

Foll originate from endothelial cells Nitric oxide......T Protein C ............F Fibronectin ........T Plasminogen activator................T

Thyroid 100-200 micro gms are trapped daily.......T increases in size in pregnancy .........T I THINK.....ACTOVITY INCREASES........SO DOES THE SIZE......BUT IM NOT SURE.... trapsinorganic iodine .......T colloid is stored outside epithelial cells.........T TBG is increased n pregnancy ...........T. 5% is bound to CBG ...........................TO TBG I THINK WAS MEANT....F......T3 AND T4 ARE BOUND 75 AND 85 %

ketone bodies formed from acetone..............T formed from acetyl CoA ..........T ormed during starvation .....T not utilised by <a href="http://www.[obscene]a.com/tutorial/?q=brain &s=0">brain </a> T

Mgso4 Potentiates action of non depolarising agents T I THINK Check respiratory status T......PARALYSIS OF RESP MUSCLE IS THE

FIRST SIGN OF OVERDOSE Causes brisk patellar reflexes T.........ANOTHER SIGN OF MG EXCESS.... In toxicity Rx IV calcium sulphate .....F.....CA GLUCONATE...

CIN CIN 3 does not involve breach in basement epithelial layer F....THATS EXACTLY TH DISTINCTION Arias stella reaction mimicks clear cell CA..........THIS ONE IS JUT SOMEWHERE IN MY brain CELLS......DONT KNOW NOW.... Tissue biopsy is required for diagnosis ......FALSE....SMEAR CAN ALSO SHOW CIN III CHNGES....... Incidence is decreased in renal transplant patients?????......F I THINK.....COULDNT FIND INFO ANYWHERE.......BUT MY GUESS IS THAT IMMUNOSUPRESSION DURING RENAL TRANSPLANT WOULD EXACERBATE MALIGNANCY RATHER THAN SUPPRESS IT.....

GH Stmulated by glucose infusion.........FALSE.... Increased in sleep ......T Opposes insulin action F Causes positive nitrogen balance F Decreased in pregnancy F GH STIM BY LOW ENERGY CONDN HYPOGLYCaenia EXCERCISE FASTING

HIGH AMINO ACIDS HIGH PROT MEAL ARGININE INFUSION STRESS STIM GLUCAGO SLEEP GH INHIB BY GLUCOSE CORTISOL FFA MEDROXYPROGEST GH ANABOLIC HORMONE......WANTS TO BULD AND CONSTRUCT.....SO I THINK N2 BALANCE WOULD BE NEGATIVE...... AND SHOULD BE HIGH IN PREG I THINK......

Foll form boundaries of ovarian fossa Ureter T LATERAL TO OF Ext iliac vn T ABOVE OF Int iliac art T ABOVE THEN POST TO OF Int pudendal art F.....WAY OFF BELOW THE LEVATOR ANI....NOWHERE CLOSE... Obliterated umb art..T LAT TO OVARY..

Penem antibiotics Active in beta lactam ring T Drug of choice for ps aeroginoa T

Vagina Derived from mullerian ducts T Seperated from anal canal by anal body....PINEAL BODY-T Bladder Pelvic splanchnic stimulation contracts the trigone F......ALL TRUE....BUT NOTHING CONTRACTS THE TRIGONE! Ext urethral sphincter innervated by pudendal nv....F....NO PUDENDAL INVLVMT IN URETHERA......SUPP BY SOMATIC FIBS FROM S2-4 Mucosa of urethra is innervated by pudendal nv F Transaction of spinal cord above S2 causes autonomous bladder T I THINK........BECAUSE IT WOULD STILL WORK AUTONOMOUSLY THROUGH REFLEX.......BUT NO SOMATIC ACTION BECAUSE CEREBRAL CONTROL IS LOST.... Afferent innervation of glans of clitoris is by ilioinguinal nv ........F AFFERRENT OF GLANS CLIT IS THROUGH PELVIC SPLANCH......EFF IS THRU PUDENDAL N......AND BLOOD SUPPLY IS THROUGH INT PUD ARE PUD N ALSO SUPPS INF RECTAL NERVE AND A PERINEAL NERVE....

Hypo k+ is found in Prolonged vomiting T In diabetis ketoacidosis T Addisons disease F HYPERKALEMIA HERE........CORTISOL IS HYPOKALEMIC.......AND NO CORTISOL IN ADDISONS....

Foll are below inguinal ligament Ilio inguinal nv T Lateral branch of genito femoral nv T Subcostal nv F Femoral art T

Inf hypogastric (pelvic) plexus Supplies pain fibres to body of ut T Continues as vesical plexus T Lies at base of braod lig T

Sciatic nv S3&4 contribute towards it F L4 TO S3 Lies behind quadratus femoris F......IT LIES ON IRIFORMIS.....WIT NO MORE USCLES ON TOP....QF IS A VERY DEEP MUSCLE Tibial branch supplies short head of biceps femoris ......FASE......SHORT HEAD IS BY COMMON PERONEAL.....LONG HEAD ISBY TIBIAL..... Injection on the outer & upper quadrant of the buttock may cause damage to the nv ....FLASE........THIS IS THE SAFEST AREA

Undescended testis More common in premature than term infants T Commonly assoc with gonadal neoplasia T Asoc with 1% of absent testis More common on left side

Corpus luteum Is major source of relaxin T RELAXIN, INHIBIN AND ACTIVIN Contains endothelial cells T I THINK, IT HAS RICH BLOOD SUPPLY.......SO ENDOTHELIAL CELLS TOO I SUPPOSE... Hysterectomy prolonges the life period F I THINK.....HYSTERECTOMY WOULD HAVE NO EFFECT.......OR IT WOULD DECREASE THE LIFE......NO BOOK CONFIRMATION.... 17 hydroxy prog is the main prog secreted F I THINK.......PROGESTERONE IS THE MAIN steroids prevent recruitment of new follicles T STEROIDS LOWER THE LEVEL OF ESTROGEN RECEPTORS..

fertilisation acrosome fuses with zona pellucida T polar body has same karyotype as ovum T fetilised egg moves slower than unfertilised egg in fallopian tube F UNSURE.....BUT WHY SHOULD IT? inner ell mass lysis deciduas during implantation F INNER CELL MASS DOES NOTHING OUTSIDE.....IT BECOMES THE FETUS.....

histo pathologic examination of tissue of conception can diagnose CIN 3 T

Cx incompetence F Ectopic preg T Gest Trophblastic Di T

Ring inversion of Y chromosome found the foll will be done Maternal chromosome is tested F Paternal chromosome is tested T Scan to look for male genitalia T Anomaly scan and amniocentesis done for fetus ?T Do karyotype of all previous children of couple F ALL ARE DEDUCTIONS......BECAUSE Y IS RELATED WITH PATERNAL.....AND ASSOCIATED WITH EXPRESSION OF MALE PHENOTYPE.......PLEASE DISCUSS

Right ureter Below sigmoid coon F LEFT URETER Supplied by vaginal art T Lined throughout by urothelium T IF UROTHELIUM IS ANOTHER NAME FOR TRANSITIONAL EPITH Contain glands F.......BLADDER EPITH IS DEVOID OF GLANDS......I THINK URETERS' IS ALSO THE SAME....

Oral absorption of drug depends More availablibility in ionised form than non ionised F.....BECAUSE I DIDNT FIND IT ANYWHERE THAT IONISED FORM INFLUENCED ORAL ABS.....

Increased lipid solubility increases penetration in CNS T Enters intestinal cells by simle diffusion T Protein binding increases excretion F Lipid form increases uptake T

Fetal circulation Portal vn drains in IVC T Ductus arteriosus takes blood to pulm art F AWAY FROM IT R atria and L atria communicate thro Foramen Ovale T

Voiding in females Occurs above rate of 20 ml/sec T MAX URINE FLOW RATE =20-40 ML/SEC When vesical press is more than 45 mm H2O T VOID PRESS=45-70 MM Initial contraction of trigone F........TRIGONE DOES NOTHING!!!

Enzyme activity is modified by Dephosphorylation T Allosteric esters T Binding to plasma prot T Proteolytic cleaving of inactive precursors T Gene transcription ?F ALL GUESSES......ANYONE KNOWS THE ANSWERS??

Functional cyst occurs in Stein levinthal synd T PID F Multiple sclerosis F Gest tropho di F Rx with clomiphene T

Amniotic fluid Hypotonic compared to fetal plasma ?T Absored in lungs ?T Unrelated to swallowing F Increased until term in fetus F DECS TO TERM Contains more HCO3 compared to fetal blood ?T

Action of complement Increase in permeability T Migration of polymorphs T

Genetics

South blot detects DNA F ITS ONLE SEROLOGICAL TEST DNA is exact copy of RNA F DNA is transcribed by RNA polymerase T Ribosomes contain RNA T Transcription occurs at any direction along the chain F ONLY 5' TO 3' DNA replication is called transformation F DNA has transcription restriction enzyme ?F DIDNT FIND ANYTHING LIKE THAT ANYWHERE Histones do not contain DNA T

Insulin Binding to tyrosine receptor kinase increases activity T Causes glucose transporter movement to plasma membrane T Increases glycogen synthetase activity T Increases fat deposition T Increase glucose utilisation by CNS ?T

Heparin T is 1.5 hrs T LMW heparin is administered 4 hrly F 8 TO 12 Given in pregnancy causes intracranial haemorrhage in fetus F DOESNT CROSS PLAC COS ITS TOO BIG Longer duration of action in S/C compared to IV administration T

Foll have imp natural reservoir other than human

Listeria F Saolmonella typhimurium F Brucella T FROM DAIRY AIMALS

Crohns disease Non caseous granulomatous infection T Forms deep fissures T Forms crypt abscesses T Assoc with mucosal polyps F......PSEUDOPOLYPS Recognised to occur in vulva F......AT THE MOST....A FISTULA MIGHT OPEN I THE VULVA......BUT THATS NOT CROHNS OCCURING IN THE VULVA I THINK.... Assoc with intestinal wall thickening ? T......NOT A PROPER STATEMENT......THERE IS INFLAMMATION.....THICKENING IS A VERY LAYMAN TERM I THINK....

Absorption in kdny Glucose occurs in loop of henle F ACTIVE ABSPN IN PCT WITH AA Water is actively absorbed in proximal conv tubules F OPASSIVE DIFFSN IN PCT WITH NA AND CL

Estrogen 17 beta estradiol is formed by aromatisation of testos T increases LH receptors T secreted by theca cells F GRANULOSA CELLS

granulosa cells secrete inhibin F CORP LUT DOES humoral immunity is depressed in pregnancy F I THINK.....IF IT WERE TRUE, RHESUS CONFLICT WOULDNT BE AN ISSUE T & B cells are derived from bone marrow T

teratoma usually benign T can occur is assoc with germ cell tumors T secretes hormones F....NOT USUALLY I THINK

during menstruation platelet and fibrin plug is formed after 1st 20 hrs of bleeding F I THINK.....PLATELET PLUG FORMN TAKES PLACE VERY QUICK AND IN VERY SMALL TRAUMAS..... hemostasis is thro estr mediated vaso constriction F hemostasis is thro PG mediated spiral vessel constriction F PG ARE CONSIDERED TO BE THE CAUSE.....BUT I DONT KNOW IF ITS AT THE SPIRAL ARTS OR BELOW THEM.......BECAUSE THE APIRAL ARTS ARE ACTUALLY SHED....ISNT IT...

During ovulation Basal vacuoles occur in mid secretory phase T Gland stromal mitosis maximum in mid secretory phase T PG in follicle fluid during ovulation F DIDNT FIND THAT ANYWHERE

Increase of estr and prog receptors in proliferaive phase T I THINK LDL receptors are increased in proliferative phase ?F Indomethacin inhibits ovulation F Matrixmetalloproteinases(MMC) is inhibited by progestrones ???!!! WOW NO CLUE Cloiphene can cause deficient secretory phase F ITS ANTI ESTROGEN......SO IT MAY SHORTEN PROLIF PHASE....

Glycogen Synthesis occurs in muscle T Is stored along with water F ITS POLYMERISED......AND TURNS TO CRYSTALS ON STORING.....THIS IS THE REASON THERE IS NO OSMOTIC HAVOC IN CELLS WITH IT

Carbohydrates Yield energy ?17 J/Kg T Major source of energy q=brain &s=0">brain </a> F I THINK ITS GLUCOSE ACTULLY..... Major component of diet T Higher energy compared with fats F FATS HAVE MAX ENERGY

Foll decrease the efficacy of OCPs Isoniazid F Carbamaepine T Phenytoin T

Rifampicin T ALSO PHENOBARB

Female pelvis compared to male Sacrum more curved F LESS CURVED AND MORE MORE BACK TILT Greater sc notch is larger T Sub pubic angle is more T Distance betw pubic sym to acetabulum is less than the diam of acetabulum F......ACETABULUM IS VERY SMALL...AND THE DISTANCE IS GREATER

MRCOG part 1 RECALLS


Some past MCQs and EMQs and parts of questions or items of questions as remembered by my students (these are actual questions appeared in march 2009 exams)
LINING OF: ECTOCX,ENDOCX,VULVA,ANAL CANAL, TRACHEA MESTROLONE:TERATOGENIC:T FIMBRIAL CYST:ORIGIN ATLANTIC FLIGHT=1/10 OF DENTAL XRAY EXPOSURE:F DNA MICROARRAY USES RFLP OCCIPITAL SKULL DEFECTS IN SYNDROME DEVELOPMENT OF TESTIS AFTER DAYS OF FERTILIZATION,DEF YOLK SAC RECEPTORS FOR ESTROGEN,PROGESTERONE,PROSTAGLANDIN,INSULIN OBTURATOR N URETER,PELVIS:M/F DIFFERENCE, BREAST CERVICAL SMEAR NORMAL,CONTAINS,

PITUITARY:REPEAT Q emqz........1.a glycoprotien synth by placenta pek 9 weeks. 2.peptide raised in down synd synth by placenta 3.peptide synth by placenta starting 5th week till b4 term 4.maculopap rash ,fever,myalgia,teratoganic 5.maculopap rash ,cerebral calcifications 6.vescicular painful rash not teretoganic options for 1st 3 emqz 1.hpl 2.hcg 3.inhibinA 4.inhibin B 5.acth 6.estrogen 7progestron options for next 3 1.cmv 2.rubella 3.herpez1 4.herpez2

5.herpez 3 6. hsv 1,2 7.hiv

vitamin causing hyperemasis vit that synthesized in human body vitamin destroyd by heating,boling calcitonin gh hiv virus

recurrent abortion human and pork insulin tv ultrasound ,frequency cobalt molecular wt vq scan, xray chest in pregnancy physiologcal changes after surgry respiratory phy in preg acute phase reactanta dentritic cells and hiv formalin fixed rpocs sample may be used to diagnose?karyotyping half life of renin teratoganic drugs blood preserved for transfusion? diaphragm ,structures passing from

heriditory cancers?where those all true?burkits lymphoma,xerodermapigmentosum messanger rna

rectus abdominus..attachment dystrodhic calcification in fibroid following can regenrate 1.liver paranchyma 2.nurons in anterior horn 3.glomerulli

ovulatory dub.. 1.extrems of life 2.5%cases of menorhagia 3.due to excess prostaglandin secretion two more options

following r correct pairs 1.progestron and molar pregnancy 2.DES and vaginal adenosis 3...... FISH anterior horn is sensory
pineal gland derivatives of nural crest extent of cauda equina endotoxins, release of tnf,stimulate coagulation cascade bacteroids r aerobics,motile mycoplasma,causes arthritis tricomnas vaginalis is intracellular,anerob meningo and ghonococci which r true ? truncus arterisus asss with abs of septum in aorto pulm channel ......t meckel divert-persistent vitelloint duct ......t..... it hpv 1,2 or vzv mycoplasma,causes arthritis ??? f pulse pressure indpndnt of stroke volume bp of neonateis less thn adult oxy hbcurve shft to left breast lymphatic drainage 75% in axill ln 75% blood volume in veins

sensitivity,specificit,ppv,normal distribution

beta oxidation of lipids

rlfp fish

kufer cells of liver.macrofages or apcz?

HIV Has reverse transcriptase that synthesise host Rna Attach to dendritic cells 1st if transferd by vaginal rout Envelope protein no 120p is needed for viral attachement HIV 2 is prevalent in Europe

EMQs: Ectocervix/Endocervix/Bladder/Trachea/Labia Majora/Bowel/Fallopian tube histology. Also cervical histology when near menstruation - ? Vacuoles etc???? Day of Yolk sac and commencement of testicular differentiation? Drugs in pregnancy - ? Tetracycline (bones) and Enoxaparin (anticoagulation in pregnany) Receptors for Oestrogen/Progesterone/Prostaglandins and Insulin Cell type/ Responce to kuppfer cells and HPV vaccine - ? cytotoxic/type 1 or type 2 responce? Hormones in Pregnancy - Inhibin/?HPL? and HCG Infections - ? CMV/Varicella/?Rubella Risk factors for ovarian/cervical and endometrial ca. Vitamins associated with hyperemesis/production yb body and heat resistant - ? Thiamine/Vitamin D and Vitamin C? Chromosome anomolies associated with Occipital skull defects/overlapping fingers and telosomerism? Not sure about the answers for these?

Vitamins associated with hyperemesis/production yb body and heat resistant - ? Thiamine/Vitamin D and Vitamin C?

deficiency of vit b6 causes hyper emesis,vit synth by human body vit k,may b flexed fingere.....trisomy 18 hypertelorism.......trisomy 18 not find ans of scalp defect

hpv vaccin moa is production of antibodies nutralizing the virus

MW of cobolt (?60) and frequency for transvaginal uss. histology of thorax / bladder / ectocervix with bleeding / bowel and skin (anal biopsy). Also receptors for oestrogen/progesterone/insulin and prostaglandins. prostatic utricle is drived from mesonephric duct ,f its a deriv of parameso preovulatory follicle is lined by theca intrna cells true in CBD obstruction serm cholestrol will be dec ?? 1.paranchymal cells of liver cn convert fruc into glu false 2.an ovarian teratoma is usually melignan false 3.nephroblastoma may b benign false 4.duplication,anuploidy,polyploidy r structral abberations of chromosome false 5.translation of dna occur inside nuclus true I am not sure 6.cong billi is constituant of amnitic fluid from 2nd tri after 20-22 weeks yess so should the answer be yes???//clarify 7.normal level is 500md/l in blood.what is actual value of both cong and uncon billi its lipid soluble ???????///cant find 8.plasma albumin binds wt triglesrides 9.metabolic acidosis occur after admin of ammuni chloride yes 10.within 48hrs of maj surgical operation there is inc in potassium excretion yes 11.cholestrol is a major constit of hdl yes 12.it is synth from acetate yes 13.histamin inhbit pepsin sec false 14.surfactant is pre in fetal lung in 24w gest noNot sure 15.pseudomonas is motile yes 16.sub cut inj of atropin produces hyonotic effect false 17.estrogn inc serum conc of albumin false binds maximally to estrogens..so conc in serum should be less 18.angiotensinogen is a globulin yes alpha 2 globulin 19.serum conc of all releasing horm of hypoth inc in preg false 20.halothn prod expulsive mix with air yes

Metabolic acidosis: With repeated vomiting With ingestion of ammonium chloride With hypokalemia..

PG stimulates alpha adrenergic receptors Pg stimulates the isolated myometrial muscle fibres in vitro

Cervix !!!!!!!!!!!!!!! Microglandular hyperplasia is premalignant. Microglandular hyperplasia is caused by COC Microglandular hyperplaisia can co exist with CIN Cervical polyp is premalignant.

All types of dysplasia may be present in microscopi examination..

The following are characters of hypoxic cell death: Apoptosistrue Phagocytosisfalse Pyknosisfalse Poikilocytosisfalse Release of phospholipidstrue

1.HNPCC syndrome.- is endometrial ca associated with it? 2.HCG IS DETECTABLE WITHIN 48 HRS OF FERTILISATION 3.THF IS PRECURSOR OF FOLIC ACID 4.hpl reaches the same concentration in fetal and maternal blood at term

1.BILE NORMALLY CONATAI HI CONC OF FREE BILLI 2.in CBD obstruction serm cholestrol will be dec 3.obturator nerve innervates obt intrnus muscle 4.preovulatory follicle is lined by theca intrna cells 5.pelvic splanshnic nerves inervates cervix nd ut. 6.fetal testis r intrabdominal 12w afr conception 7.prostatic utri is drived from mesonephric duct With regard to the chi-squared test: a. it is used to test the difference between frequencies t b. it is used as an alternative to the t-test to determine the difference between two means f c. the number of degrees of freedom is the number of independent comparisons. ? d. the greater the value of the chi-squared test, the less likely it is to be significant f e. the null hypothesis is not required t

.The prevalence rate of a disease has the following features: a. it is dependent on the incidence of the disease b. it is dependent on the duration of illness c. it measures all the current cases in the community d. it can be estimated from a cross-sectional study e. it can be used to determine the health needs of a community 10.An aetiological factor must satisfy the following before one can say that it is causally related to a disease: a. exposure to the factor must precede the development of the disease b. elimination of the factor decreases the risk of the disease c. the factor is found more frequently among the diseased than non-diseased

d. the factor is not found among persons without the disease e. the factor is found in all cases with the disease

11.A recently discovered treatment for retinoblastoma significantly extends the lifespan of the patient, but does not prevent the disease or lead to its cure. Given this scenario, which of the following statements about retinoblastoma is (are) true? a. its incidence will increase. b. its prevalence will increase. c. its incidence will decrease. d. its prevalence will decrease. e. both the incidence and prevalence will increase

12.A random selection of 1200 adults agree to participate in a study of the possible effects of drug X. They are followed prospectively for a period of five years to see if there is an association between the incidence of cataract and the use of drug X. This type of study is a: a. case-control study. b. randomized controlled clinical trial. c. cross-sectional study. d. cohort study. e. cross-over study.

13.'Validity' as applied in statistics refers to: a. expresses the degree to which two things are related. b. implies that the results of a test can be reproduced. c. describes how well a study measures what it purports to measure. d. measures the strength of relationship between cause and effect. e. indicates the probability of obtaining a given result by chance alone.

14.For the data series: 2, 1, 6, 4, 2 a. the mode is 4 b. the median is 3 c. the standard deviation is 4 d. the mean is 3 e. the mean is always identical to the median

statistical analysis of any group of numerical observationa.the mean is always less than the mode.F b.the median value always lies to the midpoint of range.T .its false bz the range of no may contain even as well as odd numbers. c.SD always >SEM -?? d.SEM is independant of the total no.of the observation.F e.there r same no of observation greater and less than the median value.f

Transferrin a)is one third saturated with iron b)is increased in pregnancy c)binds to 10mg of iron per gram d)levels in the neonate are low e)is actively transported lignocain is used in combination with adrenaline for ring block

The rectum a) usually commences at the level of S3 b) has a mesentery in its proximal third c) drains lymph to the pre-aortic nodes d) has a parasympathetic nerve supply derived entirely form the S3 spinal segment e) is about 25cm in length

During menstruation, endometrial haemostasis depends upon a) a platelet and fibrin plug formation after the first 20 hours of bleeding b) oestrogen mediated vasoconstriction c) prostaglandin mediated spiral vessel constriction d) production of plasminogen activating factor by the glandular epithelial cells

Copper IUD mode of action: ?T sperm motility ?T microthrombi ?T intervent ovary movement

stored blood which is to be used for transfusion a-kept at -4 degree f b-must be used in 1 week f c-is tested for compliment content before transfusion -- f d-may be used for platelet replacement f e-contains an acid anticoagulant t

linear regression: a-starts from the origin b-passes thru the meanc-values vary d-ranking is done???

following test r used to compare 2 drugs a and b a-student t test b-meta analysis c-wiloxone paired test withney u paired test

d)levels in the neonate are low ??? yes t

following test r used to compare 2 drugs a and b a-student t test f b-meta analysis f c-wiloxone paired test t withney u paired test t

The rectum a) usually commences at the level of S3 ..t b) has a mesentery in its proximal third .....t c) drains lymph to the pre-aortic nodes ...f d) has a parasympathetic nerve supply derived entirely form the S3 spinal segment f e) is about 25cm in length f

During menstruation, endometrial haemostasis depends upon a) a platelet and fibrin plug formation after the first 20 hours of bleeding ....t b) oestrogen mediated vasoconstriction ...f c) prostaglandin mediated spiral vessel constriction ...t d) production of plasminogen activating factor by the glandular epithelial cells ...f Copper IUD mode of action: T sperm motility ...f T microthrombi ..t T intervent ovary movement ...f facial hair increased by: oestrogen..f testerone...t spirolactone...f

stored blood which is to be used for transfusion a-kept at -4 degree f b-must be used in 1 week f c-is tested for compliment content before transfusion -- f? d-may be used for platelet replacement f e-contains an acid anticoagulant

prostaglandin

predominantly produced by UG system ...f - is a kind of lukotriene ..f - is phospholipid ..f?

following incr in injury ) FIBRINOGEN t 2) HEPATOGLOGIN t 3) FERRITIN..f

by transvaginal u/s yolk sac apear in which week....5-6 weeks and trans abdominal,7-8 weeks

Enzyme activity is modified by Allosteric esters ?F Binding to plasma prot T Proteolytic cleaving of inactive precursors F Gene transcription t

incidence of n gonorrhoea resistance to penicillin is 1% ...........?

incidence of n gonorrhoea resistance to 3 rd gen ceph 5%

genetics South blot detects DNA ...t DNA is exact copy of RNA ....f opposite z true DNA is transcribed by RNA polymerase t Ribosomes contain RNA t Transcription occurs at any direction along the chain t DNA replication is called transformation f DNA has transcription restriction enzyme f? Histones do not contain DNa...f

Histones do not contain DNa...f :T Histones dont containDNA. DNA is wound around histone. 1.coeliac ganglia contain chromafin cells? 2.pelvic sd by plan n supp ascen cilon wd motor fibers? 3.estrog prog and fsh lh having diurnal variation? 4.blood from inf vena cava is largely directd 2 foramen ovale? 5.mete nephric duct arises frommeso neph duc? 6germination of tet spors cn b inhbit by antitoxin nd toxoid both?

1.t pallidm immobilization test is pos in yaws and lime disease 2.hep b coretes ftes in vivo? 3. antigenimea indicates infectivity? 4.peniclin potentiat action of lmwheprin? 5.normal human seminal fluid ciagula 6.ethanol in excess stimulats fatty acid oxidation? 7hbf is <5%at 8w aftr birth

Calcitonin a. Is synthesised in the parathyroid glands False b. Is a decapeptide False ,32aa c. Secretion is increased at serum calcium levels below 1.5 mmol/l (6.1mg/100ml) False d. Increases renal calcium excretion True Question 2: The following cross the placenta a. TRH False b. TSH True c. Thyroxine binding globulin False d. Thyroglobulin f m not sure plz correct Question 3: The following would cause a rise in plasma cortisol concentrations a. Intense cold True b. Dexamethasone administration True c. Anti-diuretic hormone administration False d. Exercise True Question 4: The following factors cause a rise in plasma aldosterone concentration a. Hypovolaemia True b. Hyperkalaemia False c. Decreased total body sodium content True d. Raised ACTH levels False

Monozygotic TWINS Are always identical True Are always of the same sex True Have an increased risk of congenital abnormalities compared to singlet

True Have an increased risk of Down syndrome false

Question 2: Dizygotic twins May be identical False May be monochorionic False May be monoamniotic True Occur more frequently in older women False

Monozygotic TWINS Are always identical True Are always of the same sex True Have an increased risk of congenital abnormalities compared to singlet True Have an increased risk of Down syndrome false ///are u sure.. I cant find any reference

Question 2: Dizygotic twins May be identical False May be monochorionic False always dichorionic and diamniotic May be monoamniotic True ////false Occur more frequently in older women False true

1.obturator nr inner obtur intrns muscle? Nerve to obturator internus from sacral plexus. 2.glycosylation takes place in SER? 3.rubella incub per 7 to 10 dz? ..14-21 days

4.rubell specific ab appear in 14 dz? ????????????????/ 5.actinomycets is comonsl in vag nd mouth? yes 6.core antignemia of hep b virus sugg infktivity? yes

1.listeriosis is best rx with benzyl penicill yes 2.prednisolone is broncodilator? false 3.theraputic eff of ritodrin enhancd by dxamethason false 4.median value alwaz lie in midpoint of range false 5.blood ph is regulatd predom by hco3 ions? false 6.passive diff is diff for sterioisomers ??????????/ 7.lactose may b detectd in urine of preg women false

1.muscle glycoge is releasd in res to glucocorticoids ???????????????/ 2.igm fixes compl by altrnate pathway false 3.osteoprosis is asso with abnorm histo,irregular epiphy. Cant understand the question.. 4.iron may b red by vit c nd again becoms feric aftr absorption. yes 5.renin may b producd by decidua yes(expressed by chorion amnion and placentaso T/F?///???) 6.estro stimulats angioten production ???/////////cant find any reference 7.adult spinal cord is packe within single layer of dura( two layer of meneinges dura and pia so the answer should be False)

.tetrahydrofolate is inhbtd by methotrexat 2.it is a co enzyme in aminoacid metabolsm 3.dna is cleaved by restriction enzyme,its irreversbly damage in vitro at 75deg 4.hla codes for 3 clases of antigen 5.endothelila cells release factor8 related antigen 6.medulloblastomA and melignant melanoma spreads via lymphatics. 7.renal vein thrombosis is asso with hydronephrosis n fetal circulation the out put of rt vent>lt vent? false 2.mast cells control melanin frmationin dermis falseits controlled by melanin 3.myometrial cont is mediate by somatic nr false 4.it depnds on myomet gapjunctions yes 5.reducd arterial pco2 dec ph false 6.arissa stella reaction in endometrial curetting provide definit diag of preg false 7.woundhealing is delayd by ultra violt lite false

1.chromosomz can b analysed more quickly by blood sample aas compare to amnotic fluid true 2.lactose is non reducing sugar and cn b found in preg urine yes 3.glucose is trans via placenta by simple diffusion no facilated transfer 4.E2is most abundant estrogen in late pregnancy 5.vitk def causes hypothrombonemia essential for coagulation factors & prothrombinso ans should be true 6.ethanol supp adh secretion yes 7.2,3DPG IS abundnt in meternal thn fetal blood false .b mimetics red dias bp,thy incr blood glucose

2.asprin causes bronchoconstr yes???? Not sure 3.clindamycin is cell wall synth inhbitor false

4.herpes simplex caus cx warts false..molluscum contagsioum 5.germination of tetnus spors inhb by anti toxin an TT,both yes 6.in premenopausal women,95%circula testost drivd from per conversion.b.ovaries and adren contrib equallyto circula androgen.c.DHEAS driv almost xclusiv from adrena.d.testo promo SHBG synth.

: 50% DHEA + androstenidione from adrenals.50% androstenidone from ovary 25% ?ovarian 25% peripheral conversionTesterone 25% ovarian,25%adrenal 50% peripheral conversion manily from androstendione yes they decrease Shbg

.insulin like growth fac is not secr by ovary true 2.insulin sec is stimulatd bygastrin,noradrenali,somatostatin,glucagon,argnin??? .3.thyroxin is a steroid false 4.female uri bladder is in contact with supravaginal cx yes 5.obturatr internus form the roof of ischioractal fosa.false.it forms the lateral wall

1.The cells a.fetal epithelill cells yes b.amniotic cells yes c.dermal fibroblast. yes

2.Nitrogenous wastes a.urea ,creatnin,uric acid....increases in conc from 1st trimester till term. yes

3.amino acids.....same conc as maternal plasma yes

4.plasma proteins mailny albumin and globulin with ratio 6:4....inc till 30w yes

5.oncofetal protein......alpha feto proteins.....only in early pregnancy yes

6.lipids 400mg/lit at term 50%FFA,phospholipid cholestrol,lecithin. yes

7.carbohydrats mainly glucose......half of maternal serum yes

8.lactate,pyrvate,alpha ketoglutrate......same as mat plasma no Idea yes

9.anti bacterial factors....lysosomes,peroxidase,alpha interferons yes

10.Inorganic salts.....same as mat ECF yes

11.hormones....mainly E2 conjugated form,progestron and pregnanidiol,cortisone. ???estrogens mainly estradiol ,progesterone and pregnendiol,17 hydroxycortisone found in trace amounts..chamberlain pg 46

12.insulin......raises towards term

13.Billirubin.....dec towards term yes

14.pco2......55-60mmhg yes/no its 50-60 mm hg

15.po2...2-15mmhg lower then that of mat arterial po2 .yes

16.PH.....7.0

1.glomerulonephritis is ty3 hypersenstivity post streptococcal Glomerulonephritis 2.lymphocytse can change into plasma cells yes 3.serotonin causes platlet aggregation true 4.osteomalacia is charact by normal mirelization of bone false 5.minimun daily req of ess amino acid is 0.3 to 1.o g 6.protein poor but energy suff diet results dec creatnin excretion yes 7.injestion of complex prot is essential for maintanance of nitrogen balance yes

8.nor conc of plasma protin is 7g/dl 9.nerve impulses req energy ????Cant find any reference 10.in preg plasma folate level dec 11.phenytoin dec rbc folate conc yes 12.vessopresin stimulate myometrium similar structure to oxytocin so the answer should be YES.What do u think 13.extra dural space lies bet pia and dura yes 14.neonate dlverd at term has head circum of 40 to 50 cm yes 15.pineal gland activity is relatd to day length it maintains the biological clockso the anser is NO..what do u think 16.crh is released from median eminance of hypothalams yes 17.inhibin is released in pulses yes 18.t3 redily crosses placenta does not cross placenta in significant amount..so whats the answer 19.tsh in in preg ???falls in first trimester and increases in 3rd trimester so whats the answer 20.hcg primerly metabolised in kidney yes 21.insulin is metabolised in kidney..liver and kidney so yes/no whats the answer 22.bacteroids r motile some motile others non motileso whats the answer???? 23.primary oocyte has ben completed 1st mitoticdiv by birth yes 24.urethra is surrounded by smooth muscle in middle third. its covered by fibrous muscular and mucosa throughoutanswer should be false 25.pelvic surfece of secrum is in contact with anal canal false 6.glucagon promot breakdwn of protein 27.estradiol 17 can b adminis orally false 28.pressor effect of ang2 is suppresd in normal preg yes 29.cephalothin should b avoided in normal preg 30.leptospirosis produces positive wesserman reaction false 31.can b transmitted via skin yes 32.warfirin is >80%protein bound yes 33.odds ratio 1.3 applies risk of 33% yes 34.odds ratio 2 indicate halving the risk false

Options for Questions 1-2 A Epidermolysis [bleep] B Hereditary spherocytosis C Huntington?s disease D Achondroplasia E Hypertrophic cardiomyopathy F Marfan?s syndrome G Myotonic dystrophy H Von Willebrand disease I Alpha-1 anti-trypsin deficiency J Congenital adrenal hyperplasia

Instructions: A 20 year old woman has been referred for pre-natal diagnosis at 11 weeks gestation. For each of the scenarios below, choose the single most appropriate disorder from the above list. Each option may be used once, more than once or not at all. Question 1 The fetus is found to be affected by an autosomal dominant disorder caused by a point mutation in the gene for fibroblast growth factor receptor III Your answer: A Correct answer: D

Question 2 The fetus is at risk of a condition which requires fetal skin biopsy and electron microscopy for diagnosis Your answer: A Correct answer: A

Options for Questions 3-3 A Vitamin A B Vitamin D C Vitamin E D Vitamin K E Vitamin B12 F Vitamin C G Vitamin B6 H Thiamine

I Riboflavine J Folic acid Instructions: For each of the scenarios below, choose the single most appropriate option from the above list. Each option may be used once, more than once or not at all. Question 3 Maternal intake of this vitamin around the time of conception has been shown to reduce the incidence of neural tube defects Your answer: J Correct answer: J

Options for Questions 4-5 A Haemochromatosis B Down?s syndrome C Haemophilia A D Patau syndrome E Coeliac disease F Red-green colour blindness G Myotonic dystrophy H Multiple sclerosis I Prader-Willi syndrome J Cerebral palsy Instructions: For each of the modes of inheritance below, choose the single most appropriate disorder from the above list. Each option may be used once, more than once or not at all. Question 4 Autosomal recessive Your answer: C Correct answer: A

Question 5 Autosomal dominant Your answer: G Correct answer: G

Options for Questions 6-7 A Vitamin A B Vitamin D

C Vitamin E D Vitamin K E Vitamin B12 F Vitamin C G Vitamin B6 H Thiamine I Riboflavine J Folic acid Instructions: For each of the scenarios below, choose the single most appropriate option from the above list. Each option may be used once, more than once or not at all. Question 6 Fat soluble vitamin. Deficiency occurs within a few days of cessation of bile secretion Your answer: D Correct answer: D

Question 7 This vitamin is a cofactor in the synthesis of prothrombin Your answer: D Correct answer: D

Options for Questions 8-9 A DNA polymerase alpha B DNA polymerase beta C DNA polymerase delta D DNA polymerase gamma E DNA helicase F DNA ligase G RNA polymerase H RNAase I Telomerase J Restriction endonucleases Instructions: Nucleic acid replication is vital for cell division. For each of the questions below, choose the single most appropriate option from the above list. Each option may be used once, more than once or not at all. Question 8 Which enzyme replicates DNA at the ends of chromosomes? Your answer: A Correct answer: I

Question 9 Which enzyme cuts DNA at palindromic sequences? Your answer: A Correct answer: J

Options for Questions 10-11 A Vitamin A B Vitamin D C Vitamin E D Vitamin K E Vitamin B12 F Vitamin C G Vitamin B6 H Thiamine I Riboflavine J Folic acid Instructions: For each of the scenarios below, choose the single most appropriate option from the above list. Each option may be used once, more than once or not at all. Question 10 Deficiency of this vitamin typically occurs in women with hyperemesis gravidarum Your answer: H Correct answer: H

Question 11 Water-soluble vitamin with anti-oxidant effects Your answer: G Correct answer: F

Options for Questions 12-13 A RNA polymerase I B RNA polymerase II C RNA polymerase III D DNA polymerase gamma E DNA helicase F DNA ligase G Translation H RNAase I Telomerase J Restriction endonucleases Instructions: RNA and protein synthesis are vital for cell growth and differentiation. For each of the questions below, choose the single most appropriate option from the above list. Each option may be used once, more than once or not at all. Question 12 Which enzyme synthesizes transfer RNA? Your answer: H Correct answer: C

Question 13 Which enzyme degrades RNA? Your answer: A Correct answer: H

Options for Questions 14-15 A Epidermolysis [bleep] B Hereditary spherocytosis C Huntington?s disease D Achondroplasia E Hypertrophic cardiomyopathy F Galactosaemia G Myotonic dystrophy H Von Willebrand disease I Alpha-1 anti-trypsin deficiency J Congenital adrenal hyperplasia Instructions: A 20 year old woman has been referred for pre-natal diagnosis at 11 weeks gestation. For each of the scenarios below, choose the single most appropriate disorder from the above list. Each option may be used once, more than once or not at all.

Question 14 The fetus?s father died suddenly from an inherited autosomal dominant condition Your answer: G Correct answer: E

Question 15 The fetus is at risk of an autosomal dominant condition. Neonatal disease does not occur if the condition is inherited from the father but there is a 20% risk of neonatal disease if the condition is inherited from the mother. Your answer: G Correct answer: G

Options for Questions 16-17 A Vitamin A B Vitamin D C Vitamin E D Vitamin K E Vitamin B12 F Vitamin C G Vitamin B6 H Thiamine I Riboflavine J Folic acid Instructions: For each of the scenarios below, choose the single most appropriate option from the above list. Each option may be used once, more than once or not at all. Question 16 Fat soluble vitamin produced in the skin by photo-activation of 7-dehydrocholesterol Your answer: B

Correct answer: B

Question 17 Fat soluble vitamin with anti-oxidant effects Your answer: C Correct answer: C

Options for Questions 18-19 A Marfan?s syndrome B Hereditary spherocytosis C Haemophilia B D Haemophilia A E Hypertrophic cardiomyopathy F Galactosaemia G Myotonic dystrophy H Von Willebrand disease I Alpha-1 anti-trypsin deficiency J Congenital adrenal hyperplasia Instructions: A 20 year old woman has been referred for pre-natal diagnosis at 11 weeks gestation. For each of the scenarios below, choose the single most appropriate disorder from the above list. Each option may be used once, more than once or not at all. Question 18 The woman and her partner are known to be carriers of an autosomal disorder associated with juvenile cirrhosis and emphysema Your answer: A Correct answer: I

Question 19 There is a risk of ambiguous genitalia and neonatal vomiting if the child inherits this condition Your answer: A Correct answer: J

Options for Questions 20-20 A Phenylketonuria B Duchenne muscular dystrophy C Acute intermittent porphyria D Patau syndrome E Angelman syndrome F Achondroplasia G Androgen insensitivity syndrome H Congenital adrenal hyperplasia I Prader-Willi syndrome J Cerebral palsy Instructions: For each of the mechanisms of disease listed below, choose the single most appropriate disorder from the above list. Each option may be used once, more than once or not at all.

Question 20 Non-dysjunction during meiosis Your answer: A Correct answer: D

Options for Questions 21-22 A Edward syndrome B Down?s syndrome C Haemophilia A D Patau syndrome E Coeliac disease F Achondroplasia G Myotonic dystrophy H Sickle cell disease I Prader-Willi syndrome J Cerebral palsy Instructions: For each of the modes of inheritance below, choose the single most appropriate disorder from the above list. Each option may be used once, more than once or not at all. Question 21 X-linked recessive Your answer: A

Correct answer: C Question 22 Autosomal recessive Your answer: A Correct answer: H

Options for Questions 23-24 A Epidermolysis [bleep] B Hereditary spherocytosis C Huntington?s disease D Achondroplasia E Hypertrophic cardiomyopathy F Marfan?s syndrome G Myotonic dystrophy H Von Willebrand disease I Alpha-1 anti-trypsin deficiency J Congenital adrenal hyperplasia Instructions: A 20 year old woman has been referred for pre-natal diagnosis at 11 weeks gestation. For each of the scenarios below, choose the single most appropriate disorder from the above list. Each option may be used once, more than once or not at all. Question 23 The fetus is at risk from a haemolytic disorder caused by a mutation in the gene for spectrin Your answer: A Correct answer: B

Question 24 The woman?s father is affected by an autosomal dominant neuro-psychiatric disorder that typically presents in the 3rd -4th decades of life Your answer: A Correct answer: C

Options for Questions 25-26 A Vitamin A B Vitamin D C Vitamin E D Vitamin K E Vitamin B12 F Vitamin C G Vitamin B6 H Thiamine I Riboflavine J Folic acid Instructions: For each of the scenarios below, choose the single most appropriate option from the above list. Each option may be used once, more than once or not at all. Question 25 Fat soluble vitamin synthesized in the intestinal wall from beta carotene Your answer: A Correct answer: A

Question 26 Fat soluble vitamin synthesized by large intestinal bacteria Your answer: A Correct answer: D

Options for Questions 27-28 A Edward syndrome B Down?s syndrome C Acute intermittent porphyria D Patau syndrome E Angelman syndrome F Achondroplasia G Androgen insensitivity syndrome H Congenital adrenal hyperplasia I Prader-Willi syndrome J Cerebral palsy Instructions: For each of the mechanisms of disease listed below, choose the single most appropriate disorder from the above list. Each option may be used once, more than once or not at

all.

Question 27 Deletion in a paternally derived chromosome Your answer: A Correct answer: I

Question 28 Deletion in a maternally derived chromosome Your answer: A Correct answer: E

Options for Questions 29-30 A Marfan?s syndrome B Hereditary spherocytosis C Haemophilia B D Haemophilia A E Hypertrophic cardiomyopathy F Galactosaemia G Myotonic dystrophy H Von Willebrand disease I Alpha-1 anti-trypsin deficiency J Congenital adrenal hyperplasia Instructions: A 20 year old woman has been referred for pre-natal diagnosis at 11 weeks gestation. For each of the scenarios below, choose the single most appropriate disorder from the above list. Each option may be used once, more than once or not at all. Question 29 The woman is affected by an autosomal dominant condition associated with aortic aneurysms and lens subluxation Your answer: A Correct answer: A

Question 30 The woman has an autosomal dominant bleeding disorder Your answer: A Correct answer: H

Options for Questions 31-32 A DNA polymerase alpha B DNA polymerase beta C DNA polymerase delta D DNA polymerase gamma E DNA helicase F DNA ligase G RNA polymerase H RNAase I Telomerase J Okazaki fragments Instructions: Nucleic acid replication is vital for cell division. For each of the questions below, choose the single most appropriate option from the above list. Each option may be used once, more than once or not at all. Question 31 Which enzyme synthesizes the lagging strand of DNA? Your answer: A Correct answer: A

Question 32 Which enzyme joins Okazaki fragments forming a continuous strand of DNA? Your answer: A Correct answer: F

Options for Questions 33-34 A Vitamin A B Vitamin D C Vitamin E D Vitamin K E Vitamin B12 F Vitamin C G Vitamin B6 H Thiamine I Riboflavine J Folic acid Instructions: For each of the scenarios below, choose the single most appropriate option from the above list. Each option may be used once, more than once or not at all. Question 33 Deficiency of this vitamin in childhood causes rickets Your answer: A Correct answer: B

Question 34 Fat soluble vitamin. Synthesis by the kidneys is regulated by parathyroid hormone Your answer: A Correct answer: B

Options for Questions 35-36 A Marfan?s syndrome B Hereditary spherocytosis C cystinuria D phenylketonuria E Coeliac disease F Galactosaemia G Myotonic dystrophy H Gaucher disease I Alpha-1 anti-trypsin deficiency J Congenital adrenal hyperplasia Instructions: A 20 year old woman has been referred for pre-natal diagnosis at 11 weeks gestation. For each of the scenarios below, choose the single most appropriate disorder from the above list. Each option may be used once, more than once or not at all. Question 35 Pre-natal diagnosis can be undertaken by measuring the concentration of 17-hydroxyprogesterone in amniotic fluid Your answer: A Correct answer: J

Question 36 The fetus is at risk from an autosomal recessive disorder which can be treated by exclusion of milk products from the diet Your answer: A Correct answer: F

Options for Questions 37-38 A RNA polymerase I B RNA polymerase II C RNA polymerase III D DNA polymerase gamma E DNA helicase F DNA ligase G Translation H RNAase I Telomerase J Restriction endonucleases Instructions: RNA and protein synthesis are vital for cell growth and differentiation. For each of the questions below, choose the single most appropriate option from the above list. Each option may be used once, more than once or not at all. Question 37 Which enzyme synthesizes ribosomal RNA? Your answer: A

Correct answer: A Question 38 Which enzyme synthesizes messenger RNA? Your answer: A Correct answer: B

Options for Questions 39-40 A Haemochromatosis B Hereditary spherocytosis C cystinuria D phenylketonuria E Coeliac disease F Galactosaemia G Myotonic dystrophy H Gaucher disease I Alpha-1 anti-trypsin deficiency J Congenital adrenal hyperplasia Instructions: A 20 year old woman has been referred for pre-natal diagnosis at 11 weeks gestation. For each of the scenarios below, choose the single most appropriate disorder from the above list. Each option may be used once, more than once or not at all. Question 39 The fetus is at risk from an autosomal recessive disorder caused by a mutation in the beta-galactosidase gene Your answer: A Correct answer: H

Question 40 The fetus is at risk from an autosomal recessive disorder characterised by diabetes mellitus, cardiomyopathy, arthropathy and cirrhosis Your answer: A

increased during prgnancy :..... *renal excretion of HCO3 *plasma volume 2600_____ 3800ml.(until 32wks.) it is related to the size of the fetus . *Red cell mass(1400ml _____1650-1800) until term . *COP (4.5l___6.1 l/min) = 40% early and reach plateau at 24 __ 30wks *HR (80 ___ 90/min) = 10 % * O2 consumption (250 __ 300ml ) increased by 30 __ 50 ml at term . *stroke volume increased by 64 __ 71 ml ... this inc in cardiac out put 1500 ml ( skin 500--- uterus 400 at term 700..... kidney 300 others (git , breast 300). *Renin inc in pregnancy and estrogen containing pills but the action of angiotensinogen 2 is blocked so decreased PR *Fluid transfer across the endothelium so Oedema. *EDHF (endothelial derived hyperpolarizing factor ) * Throboxane ( 3-5 times) until term *EDrwF *Endothelin . *clotting factors 1,5, 8 . * Increased PAI (plasminogen activating inhibitor===> decreased fibrinolysis) *Increased PTA (tissue plasmin activator)==> inc fibrinolysis *Generalised inflamatory response *erythropoiten (3times ) *Ventliation due to progesterone(40% more in the first trimester). * The inc in O2 is 50ml ( 20 fetus , 18 metabolic activity ,6 COP , 6 inc inrenal work. *ADH. *the sensitivity of the respiratoratory to Co2( like does hypoxia ) *Diaphragm 4cm ( transverse diameter of the chest 2cm , subcostal angle 38 - 103 degree ) * Pulmonary blood flow 40% * pulmonary arterial pressure (12--19%) *Tidal volume 200ml *Alveular ventilation 30% *Vital capacity (MAY).

*Minute volume (3L) *Kidney length (1cm ) due to inc cell size. *Ureter dilated due to ( proges cause smooth muscle reaxation+obstruction by the uterus and collectig system) *Renal blood flow 1.2L/min___1.5L/min early and decreased in the 3rd trimest ) *GFR(120 ml/min. *filtered load of glucose ====> glucosuria. *Renal excretion of sugars, water and soluble vitamins ,proteins(albumin) and transferin , amino acids. *Aldosterone and deoxycorticosterone , cortisol ( total and free ) max in 3rd trimest. *Inc filtered load of sodium (balanced by reabsorption) *Apettite and thirst . *Motilia and somatostatin *Gums.swell and bled easily , caries. *Gastric emptying time (50 to 100 min ) *Inc osteolytic activity=====> inc alkaline phosphatase *Serum bile acids ===>cholyglycin *Fatty acids *Enhanced insulin response to glucose early in the 2nd trimest so inc in 2hr plasma glucose *Binding proteins , transferin binding capacity *B globin * SHBG. * IgM , IgD. *lipids (TG , cholesterol ,phospho lipids , FFAs , lipoproteins ,. *Pro vit A ,E ,D . Parathrmone *copper (2--3 times ) due to inc ceruplasmin. *susceptability to infection (polio , infuensa , malaria ). *Erythropoiesis due to ( erythroid hyperplasia and inc transferin ) *TWBC (neutrophils) *MCV *ESR *Red cell fragility (RC) *red cell size . *Fibrinogen. FAP *Factor 7,8,9,10,12 from the third trimest *Fetal arrterial presure through out prgnanacy. *Water reabsorption in large itestine . Iron absorbtion *Size of the thyroid gland . *Protein bound iodine . *total T3 *Androstenadione , Testerone. *The proportion of B to T cells AND WE HAVE DECREASED DURING PREGNANCY SOON ( URS nagm) Also there is increase in ACTH CRH

How To Prepare For the folic acid plasma test, do not eat or drink (other than water) for 8 to 10 hours before the test. If you take any medicines regularly, your doctor will talk to you about how to take these before the test. You do not need to do anything before having a folic acid red blood cell test Results A folic acid test measures the amount of folic acid in the blood. Normal values vary from lab to lab. Folate in liquid portion (plasma) of blood

Adult 220 nanograms per milliliter (ng/mL) 445 nanomoles per liter (nmol/L) (SI units) Children 521 ng/mL 1147 nmol/L

Folate in red blood cells Adult 140628 ng/mL 3171422 nmol/L (SI units) Children More than 160 ng/mL More than 362 nmol/L High values High levels of folic acid in the blood may mean that you eat a diet rich in folic acid, take vitamins, or take folic acid pills. Consuming more folic acid than the body needs does not cause problems. High folic acid levels can also mean a vitamin B12 deficiency. Body cells need vitamin B12 to use folic acid. Therefore, if vitamin B12 levels are very low, folic acid cannot be used by the cells, and high levels of it may build up in the blood. However, a folic acid test is not a reliable way to test for a vitamin B12 deficiency. Low values Low folic acid levels can mean you have a problem with your diet, alcohol dependence, or an eating disorder such as anorexia nervosa.

some past EMQ1.tumour like growth - hamartoma 2.locally mg ca - basal cell ca 3.due to which vit defxeropthalmia - vit A hyperemesis graviderum - thimine macrocytic anemia - vit B12 4.screening test for following alpha thalassaemia - hb electrophpresis beta thalassaemia -same folate def - mcv iron def - low level of serum iron?? untreated syphilis - flurosent Treponemal antibody test 5.hormone receptors for following r located in Insulin - protien tyrosine kinase activity receptor on cell membrane PGE2 - G linked cell membrane receptor. Estrogen - nuclear transcription receptor? progesteron - protien kinase nuclear receptor? 6.on taking a large protien meal insulin glucagon both secretion increased. 7.cervical ca - hpv 16,18 8.woman after cervical surgery with drains left in place for some weeks has slow recov bacilli dont understand.is it actinomycete ? 9.immune response in malaria with pg - b lymphocyte? 10.in a woman who recvd vaccine for Hpv - cytotxic t cell? This suggests the occurrence of an immune response more specific for the RP5 line in women having had multiple pregnancies, and who are likely to develop immunity to pregnancy-associated parasites. Both humoral and cellular mechanisms may account for the lower susceptibility of multigravidae to malaria. 1.a gray(100rads)is a unit of energy absorption 2.conjugatd billi is constituants of amniotic flud 3.theraputic dose of megnisium trisillicate causes alkalosis 4.cholestrol is maj cons of hdl 5.it can b synthesisd from acetate 6.muscle glycoge metebol cannot yield free glu 7.it is independant of phosphorylase 8.it only gener atp undre arobic condition 9.it is entirely intracellular 10.it is released in circulation in resp to glucocorticoid 11.adenocarcinoma of ovary involv mtaplastic process 12.coal dust causes ca bronchus

13.stored blood b4 transfusion checkd for complment contant 14.t nd b lymphocytes can do phagocytosis 15.macrophages provides protecton against intracellular pathogen 16.the peak pressure in pulmonry arterial sys is <onetenth of that in systemic circulation 17.transitional epith of bladder has mesodermal orign 18.amnion has meso dermal origion 19.peritoneum has meso drmal orign 1.randomisation is best achieve by alternate allocation of subjects in each group 2.creatnin has plasma clearance rate equal to plas flow to kidney 3.glycolysis is a mobilization of stored glucose units 4.glucose to pyruate pathway is present in all tissues 5.putez jager synd causes intestinal ca. 6.testicular ca is hormone depndent 7.bile normally contain hi conc of free cholestrol 8.inspiration is brought abt by relaxation of intercostal muscle 2.creatnin has plasma clearance rate equal to plas flow to kidney T,it may not be equal to it ????125ml/hr both i dont think so inspiration is brought abt by relaxation of intercostal muscle true randomisation is best achieve by alternate allocation of subjects in each group ????what is the best way of randomization then? 1iron diffus passively in erythropoietic cells 2.n gonnorhae culture is inhbitid by low temp 3.anti biotic rx in septic shock aggriv hypotension 4.death is characterize by lucocytosis 1.a gray(100rads)is a unit of energy absorption 2.conjugatd billi is constituants of amniotic flud 3.theraputic dose of megnisium trisillicate causes alkalosis 4.cholestrol is maj cons of hdl 5.it can b synthesisd from acetate 6.muscle glycoge metebol cannot yield free glu 7.it is independant of phosphorylase 8.it only gener atp undre arobic condition 9.it is entirely intracellular 10.it is released in circulation in resp to glucocorticoid 1.iron diffusespassively in erythropoietic cells 2.timoxifin incr the action of warferin 3.asprin dec the action of ritodrin 4.relative ris reduction of 60%is significant irrespective of value of p 5.alcohol is transported actively across placenta 6.folic acid is destroyed by boliing water 7.natural killer cells have MHC 1 receptors 8.ovary develop in medulla of primitive gonade 9.aldosteron is a steroide 10.vincent angina and billhazaria is caused by?organisms. 11.asprin causes bronchoconstriction 12.in randomised double blind trail pt will b taking either of 2 drugs 13.thiopenton na is predominately excreted via kidney 14.amniotic fluid normally contain fetal or maternal cells 1iron diffus passively in erythropoietic cells false 2.n gonnorhae culture is inhbitid by low temp yes 3.anti biotic rx in septic shock aggriv hypotension 4.death is characterize by lucocytosis .a gray(100rads)is a unit of energy absorption Its unit of Xrays absorbed/radioactive energt absorbed..so it should be yes...comments invited 2.conjugatd billi is constituants of amniotic flud yes

3.theraputic dose of megnisium trisillicate causes alkalosis false 4.cholestrol is maj cons of hdl yes 5.it can b synthesisd from acetate yes 6.muscle glycoge metebol cannot yield free glu yes 7.it is independant of phosphorylase false 8.it only gener atp undre arobic condition yes..not sure 9.it is entirely intracellular yes 10.it is released in circulation in resp to glucocorticoid yes??????????

2.timoxifin incr the action of warferin false 3.asprin dec the action of ritodrin should be falseno reference 4.relative ris reduction of 60%is significant irrespective of value of p 5.alcohol is transported actively across placenta yes 6.folic acid is destroyed by boliing water yes 7.natural killer cells have MHC 1 receptors yes 8.ovary develop in medulla of primitive gonad false.it develops in cortex 9.aldosteron is a steroid yes 10.vincent angina and billhazaria is caused by?organisms. vincents angina BORRELIA VINCENTI billhazaria is SCHISTOSOMIASIS.caused by S hematobium,japonicum,mansoni species 11.asprin causes bronchoconstriction yes thats how it is involved in aspirin mediated bronchial asthama 12.in randomised double blind trail pt will b taking either of 2 drugs yes 13.thiopenton na is predominately excreted via kidney false metabolised by the liver 14.amniotic fluid normally contain fetal or maternal cells yes *After injection of the D antigen into Rhesus negative individuals, the peak concentration of anti-D antibodies is attained within 3-6 weeks false rbc *Have an average diameter of 80 microns false 7 microns *Have a mean cell volume of 60 fl yes 76-96 *passive diff is diff for sterioisomers ??????????/cant understand *billirubin facilitates absorption of fat from the gut yes *lactose-may be detected in the urine of pregnant woman yes *un conjugated billii bound to specific proteins in the liver cells *Glycosylation takes place in the smooth endoplasmic reticulum yes *nuclear heterochromatin is genetically inactive ???false * Tetany may occur as a complication of September 1998/P2/Q27, March 2001/P2/Q30 A-osteoporosis B-hypercapnia (hypocapnia)????? C-respiratory acidosis??? D-peripheral neuropathy false E-untreated hyperparathyroidism false *liver parenchyma is more radiosensitive than intestinal epithelium false

*Chromosomes1-are usually studied after 12-24 hrs. culture of peripheral blood yes *30-year-old female undergoes a Chest and abdominal X-ray arranged by her GP and does not realise that she is 7 weeks pregnant. Increased risk to which of the following are found in those children exposed in utero to such X irradiation *Acute lymphoblastic Leukaemia *Intra-uterine growth retardation *Mental retardation *Diabetes *Cerebral gliomas 1The rate of DNA replication is directly under the control of enhancer sequences. 2Mitochondrial genes are inherited from the mother. false 3Transcription factors are mainly made of RNA. ??? 4Introns are the portions of a gene which code for protein. false 5Most of the human genome encodes polypeptide. 6The following are true regarding cyclic AMP: 1.produced from ATP yes 2.degraded by phosphodiestrase yes 3.activates protein kinase C yes 4.activates STAT 3 whats that cant understand 5.produced in response to Glucagon. Bowie Dick Test The Bowie-Dick (BD) test was developed to detect air leaks and to evaluate the ability of the air removal system to reduce air residuals within the sterilization chamber to an accept able level. (1) It is not an absolute air or vacuum measurement system. The Bowie-Dick test sheet is a visual indicator test that may be used as a predictor of future air removal problems in that any unsatisfactory outcome may be anticipated to get worse with continued use of the sterilizer. (2) Fluorescent in-situ hybridization 1.Can be used in the pre-natal diagnosis of Down syndrome yes 2.Typically takes 2-3 weeks for the results to be available flase 3.Requires cells in the metaphase stage of cell division false 4.Can be applied to chorionic villi but not to amniocytes false but not sure

Question 5: With respect to the structure of chromosomes *Nucleosomes are secondary coiling of DNA around spherical histone beads yes *Chromatin fibres are formed from tertiary coiling of nucleosomes yes *Chromatin fibres form loops around a scaffold of acidic protein called histone falsenon histone acidic protein *Chromosomes are formed from thght copiling of chromatin yes "In general, pregnant women should avoid x-rays if possible. The first trimester is especially important when much of the fetal development is happening. Some of the risks associated with x-rays during pregnancy involve certain cancers and leukemia, birth defects, and low birth weight. The risks of these problems occurring after an x-ray are very small and should be discussed with your doctor if you have had an x-ray or are planning to have one. " 30-year-old female undergoes a Chest and abdominal X-ray arranged by her GP and does not realise that she is 7 weeks pregnant. Increased risk to which of the following are found in those children exposed in utero to such X irradiation *Acute lymphoblastic Leukaemia yes *Intra-uterine growth retardation yes *Mental retardation ?? *Diabetes false *Cerebral gliomas yes

Metabolic acidosis: With repeated vomiting With ingestion of ammonium chloride With hypokalemia.. PG stimulates alpha adrenergic receptors Pg stimulates the isolated myometrial muscle fibres in vitro Cervix !!!!!!!!!!!!!!! Microglandular hyperplasia is premalignant. Microglandular hyperplasia is caused by COC Microglandular hyperplaisia can co exist with CIN Cervical polyp is premalignant. All types of dysplasia may be present in microscopi examination.. The following are characters of hypoxic cell death: Apoptosis Phagocytosis Pyknosis Poikilocytosis Release of phospholipids

1.HNPCC syndrome.- is endometrial ca associated with it? 2.HCG IS DETECTABLE WITHIN 48 HRS OF FERTILISATION 3.THF IS PRECURSOR OF FOLIC ACID 4.hpl reaches the same concentration in fetal and maternal blood at term

mooth endoplasmic reticulum The smooth endoplasmic reticulum (SER) has functions in several metabolic processes, including synthesis of lipids and steroids, metabolism of carbohydrates, regulation of calcium concentration, drug detoxification, attachment of receptors on cell membrane proteins, and steroid metabolism. During menstruation, endometrial haemostasis depends upon a) a platelet and fibrin plug formation after the first 20 hours of bleeding b) oestrogen mediated vasoconstriction c) prostaglandin mediated spiral vessel constriction d) production of plasminogen activating factor by the glandular epithelial cells human milk containscontains more calories then breast mil is radiation exposure of dental x-ray equivalent to 1/5 that from a transatlantic flight tumor necrotic factor alpha raised in parturition can u rely on chromosome length to idintify them. human milk contains more water ring inversion of Y MRI involves the administration of radio labelled compound Proton is injected to improve contrast during MRI Regarding data a The coefficient of variation expresses the Standard deviation as a proportion of the mean b Standard error of mean indicates how close a sample is to the population mean c A probability of one means that an event has one in ten chance of happening d Logarithmictransfoormation is a method of normalizing data that are not normally distributed e ANOVA is used for normally distributed data

The prevalence rate of a disease has the following features: a. it is dependent on the incidence of the disease yes b. it is dependent on the duration of illness no c. it measures all the current cases in the community yes d. it can be estimated from a cross-sectional study yes e. it can be used to determine the health needs of a community yes 10.An aetiological factor must satisfy the following before one can say that it is causally related to a disease: a. exposure to the factor must precede the development of the disease yes b. elimination of the factor decreases the risk of the disease yes c. the factor is found more frequently among the diseased than non-diseased yes d. the factor is not found among persons without the disease yes e. the factor is found in all cases with the disease yes 11.A recently discovered treatment for retinoblastoma significantly extends the lifespan of the patient, but does not prevent the disease or lead to its cure. Given this scenario, which of the following statements about retinoblastoma is (are) true? a. its incidence will increase. false b. its prevalence will increase. Yes. Incidence is no of new cases over a period of time.and prevalence is no of cases affected at a given time. c. its incidence will decrease. true d. its prevalence will decrease. false e. both the incidence and prevalence will increase false

12.A random selection of 1200 adults agree to participate in a study of the possible effects of drug X. They are followed prospectively for a period of five years to see if there is an association between the incidence of cataract and the use of drug X. This type of study is a: a. case-control study. no b. randomized controlled clinical trial. yes c. cross-sectional study. no d. cohort study. no e. cross-over study. No??? 13.'Validity' as applied in statistics refers to: a. expresses the degree to which two things are related. b. implies that the results of a test can be reproduced. c. describes how well a study measures what it purports to measure. d. measures the strength of relationship between cause and effect. e. indicates the probability of obtaining a given result by chance alone.

14.For the data series: 2, 1, 6, 4, 2 a. the mode is 4 false b. the median is 3 yes c. the standard deviation is 4 false d. the mean is 3 false e. the mean is always identical to the median false Microglandular hyperplasia is premalignant. false Microglandular hyperplasia is caused by COC true Microglandular hyperplaisia can co exist with CIN false Cervical polyp is premalignant. false All types of dysplasia may be present in microscopi examination.. false 1.A normal dstribution of value a.is symtrical abt the mode.T b.has amedian which is greater than the mean. F c.has 75% of its value below the upper quartile. T? d.may contain -ve value. T 2.In aclinical trial randomised allocation of pt to rx group a.eleminate the investigator bias. T b.reduce the placebo effect. F c.usually controls for known confounding variables. T? d.usually ................unknown............................... T? e.is best achvd by alternate allocation of subject. F 3.iN Randomzd dbl blind trial comparing a new drug with a placebo.. a.the pt wl b taking either of two actv drug..T?bcoz the pt him self cannot take.If given to the pt then tts true.confused. d.A large trial is more likely to give a statistically sgnificant result than a small trial. T 49.concerning the analysis of clinical trial a.the 95%confidence interval indicates the range within which19 out of 20 values will lie.T .....f b.the P value illustrate how often the result shd b expected by chance.T ......ok c.the conventional level of statistical significance isis set at p <.005.F It shd b <.05 ok d.in a randomised trial there must be thr must b equal no.of recruit in each arm of the study.F ok e.A relative risk reduction of 60% is significant irrespective of the value of p. F ok 50.concerning the ability of a test to predict dse a.sensivity is the ablty to prdct thos with dse corctly.T b.sensivity is same as +ve prdctv value.F c.The confidence interval must cross 1 to prove significance.F d.An odd ratio of 1 :3 implies a risk of 33% T e.An odd ratio of 2 indicate halvng of risk.F 5. Non-parametric tests include a. ANOVA .......f b. Student's t-test .......f c. Chi-squared test ......t d. Wilcoxon signed rank test ....t e. Mann-Whitney U test.t Options for Questions 10-11 A Benzyl penicillin B Clotrimazole C Metronidazole D Teicoplanin

E Flucloxacillin F Ampicillin G Procaine Penicillin H Rifampicin I Doxycycline J Azithromycin Instructions: For each of the scenarios below, choose the single most appropriate antibiotic from the above list. Each option may be used once, more than once or not at all. Question 10 A 20 year old woman complaining of vaginal discharge has a high vaginal swab which is positive for Candida albicans Question 11 A 37 year old woman has a positive VDRL test at 14 weeks gestation Options for Questions 12-13 A Interferons B Endotoxins C Exotoxins D Immunoglobulins E Cytokines F Histamine G Prostaglandins H Interleukins I Tumour necrosis factor J Lipopolysaccharide Instructions: For each of the statements below, choose the single most appropriate option from the above list. Each option may be used once, more than once or not at all. Question 12 Are typically produced by gram positive bacteria and destroyed by heat Question 13 Toxic lipopolysaccharide molecules Options for Questions 14-15 A Staphylococcus aereus B Actinomycete israelii C Neisseria Gonorrhoea D Chlamydia trachomatis E Schistosoma haematobium F Pseudomonas aeruginosa G E. coli H Pseudomonas aeruginosa I Bacteriodes J Mycobacterium tuberculosis Instructions: For each of the scenarios below, choose the single most appropriate micro-organism from the above list. Each option may be used once, more than once or not at all. Question 14 Gram negative intra-cellular diplococcus Question 15 Gram negative diplococcus, causes arthritis, urethritis and pharyngitis

Options for Questions 16-16 A Benzyl penicillin B Nystatin C Metronidazole D Teicoplanin E Flucloxacillin F Ampicillin G Procaine Penicillin H Rifampicin I Doxycycline J Erythromycin Instructions: For each of the scenarios below, choose the single most appropriate antibiotic from the above list. Each option may be used once, more than once or not at all. Question 16 A 70 year old woman develops oral candidiasis following abdominal hysterectomy for endometrial cancer. Options for Questions 17-18 A Interferons B IgA C IgG D Immunoglobulins E Cytokines F IgE G Prostaglandins H Interleukins I IgM J IgD Instructions: For each of the statements below, choose the single most appropriate option from the above list. Each option may be used once, more than once or not at all. Question 17 Immunoglobulin present in secretions Question 18 Pentameric immunoglobulin

Options for Questions 19-20 A Benzyl penicillin B Carbenicillin C Metronidazole D Teicoplanin E Flucloxacillin F Ampicillin G Pyramethamine H Rifampicin I Doxycycline J Azythromycin Instructions: For each of the scenarios below, choose the single most appropriate antibiotic from the above list. Each option may be used once, more than once or not at all. Question 19 A 45 year old woman has an abnormal pre-operative chest X-ray and is found to have active tuberculosis Question 20 A 30 year old farmer develops toxoplasmosis at 12 weeks gestation Options for Questions 21-22 A Staphylococcus aereus B Actinomycete israelii C Neisseria Gonorrhoea D Chlamydia trachomatis E Schistosoma haematobium F Pseudomonas aeruginosa G E. coli H Pseudomonas aeruginosa I Bacteriodes J Mycobacterium tuberculosis Instructions: For each of the scenarios below, choose the single most appropriate micro-organism from the above list. Each option may be used once, more than once or not at all. Question 21 Coagulase positive gram positive bacteria which produce an exotoxin Question 22 Gram positive branching filamentous bacteria. Infection characterised by sulphur granules.

Options for Questions 23-24 A Rubella B Influenza A C Cytomegalovirus D Parvovirus B19 E Coxsackie B virus F Hepatitis A G Hepatitis B H HIV I Toxoplasmosis J Varicella zoster Instructions: Infections in pregnancy may be associated with fetal adverse effects. For each of the questions below, choose the single most appropriate micro-organism from the above list. Each option may be used once, more than once or not at all. Question 23 If maternal infection occurs in the first trimester, which organism is associated with the highest risk of fetal infection? Question 24 Which organism is typically associated with fetal hydrops Question 19 A 45 year old woman has an abnormal pre-operative chest X-ray and is found to have active tuberculosis .....H

Question 20 A 30 year old farmer develops toxoplasmosis at 12 weeks gestation ....GQuestion 21 Coagulase positive gram positive bacteria which produce an exotoxin ....A Question 22 Gram positive branching filamentous bacteria. Infection characterised by sulphur granules. ....B Question 23 If maternal infection occurs in the first trimester, which organism is associated with the highest risk of fetal infection? ......A Question 24 Which organism is typically associated with fetal hydrops ......D Vascular derived epithelial growth factor Causes vasodilatation of placental vessels Causes angiogenesis in placenta Is an angigogenic factor Increases permeability of endothelial cells Conventional USG

Increased freq gives higher resolution ................t Increased freq gives better attenuation f TVS uses higher freq compared to conventional USG t comments awaited Ring inversion of Y chromosome found the foll will be done Maternal chromosome is tested f Paternal chromosome is tested t Scan to look for male genitalia f Anomaly scan and amniocentesis done for fetus t Do karyotype of all previous children of couple f

MRCOG part 1 RECALLS SEPTEMBER 2008


Paper 1 & 2 EMQs In a country wide survey, 10 general practices were picked at random and 5% of patients were selected at random from each practice: Ta) the sample of patients is a true random sample Tb) all practices had an equal chance of selection Tc) all patients had an equal chance of selection Fd) two siblings could not have been selected Te) inferences about all patients in the country may be drawn from the sample A lady is about to deliver and you are about to give her a pudendal block.. a) ant division of ant. remi of S2-4 b) ant division of post.rami of S2-4 b)genitofemoral nerve c) post division of ant. remi of S2-4 d) post division of post. remi of S2-4 e) sensory supply of perineal nerve f) inferior rectal nerve g) ischial tuberosity h)ischial spine Q1 -wht is the root value of pudendal nerve? Q2- wht nerve supplies lower part of vagina (I think)? Q3 why do u give local skin infiltration before episiotomy? If we take the day of fertilization as Day0 then Day2 Day4 Day 8 Day 10 Day12 Day 14

Day 18 Day20 Day 22 Day 24 Day 26 Day 42 Day 56 Day 70 Q4-Which represents 4 cell stage ? Q5- conceptus implants completely? Q6-feable hrt pulse seen on ultrasound? Q7- vertebra form completely? Table: on different values given for deaths due to maternal mortality with direct and indirect causes as well as # of days from delivery to the cause of death Deaths due to placenta previa and hypovolemia x @ e.g 20 days,10 days,8days etc suicide(with previous hx of psychiatric illness) crash victim diabetic neuropathy two more causes given with values

Different numbers were given in options Q8-the number of late maternal deaths due to indirect causes? Q9-number of maternal deaths due to direct causes? Calculate Numbers needed to treat from a drug trial on osteoporosis: (Q frm EMQ book) Options

25 50 100 250 300 400 1000 others Q10-N1=10,n2=5,n3=9990,n4=9995 Q11-N1=50,N2=25,n3=9950,N4=9975 FSH Oxytocin GH Prolactin Rennin ACTH etc Q12-which is a protein with alpha and beta subunits ? Q13 which stimulates mineralocorticoid activity/relsease? Q14- which hormone is produced by posterior pituatory? Folic acid Magnesium Calcium Vit b1-B12 given Q15- which vitamin is required in 400mcgs at preconception? Q16-which vitamins absorption is hampered by oxalic acid & phytates? Insulin & Glucagon excretion Decrease decrease Increase increase Increase decrease Deacrease increase

Nochange no change Nochange increase Nochange decrease Etc Q-17-on taking a large protein meal? Folowing are risk factors for cancer Q-18- HCC ans Hep B Q19- Skin cancer-ans UV radiation Q20- Cervical/uterine cancer-ans HPV Q21- Breast cancer-ans BRCA1 mutation (Repeats from march 2008) Hormone receptors for following are located in a) protein kinase nuclease receptor b) nucleae transcription receptor c) G-linked cell membrane receptor d) G-linked golgi apparatus receptor e) Protein complex intra cytoplasmic receptor f) Multiligand receptor in cell membrane etc g) Protein Tyrosine kinase activity receptor on cell membrane Q22- Insulin Q23-PG E2 Q24-Estrogen Q25-Progesterone

Q26-A family with one kid presumed to have cystic fibrosis however cftr gene mutation was not detected. Family members willing to give samples

if needed.mother wants definite diagnosis as she is 11wks and wants to know diagnosis for her baby.parents are cousins with 1st degree relatives having cystic fibrosis. Invasive diagnostics can be applied to reach final results by? Q27-you have Guthrie spot of their son how will u diagnose him for cystic fibrosis? Q28- families in uk with cystic fibrosis how will u find f508? Options: Pcr Fish test Chromosomal linkage analysis Telomer analysis Sum hereditary crosslinkage chromosomal option (dnt remember) Screen for some known gene mutation Sweat test Saliva test screening test for following Q29-alpha thalessemia Q30-folic acid deficiency Q31-iron deficiency anemia Q32-(one more i think!) options hbelectrophorisis mcv mchc MCh serum iron paul-bennel test etc Q33-which is the commonest cause of non-dysjunction at meiosis? trisomy 21

Q-34 which defect is associated with 60-70% cleft palate and cleft palate? trisomy 13 or 18 or KF syndrome all about cystic fibrosis screening tests for a-thallesemia trait iron deficiency amemia,folic acid anemia and....EMQ RECEPTORS FOR ESTROGEN,PROGESTERONE,PROSTAGLANDIN,INSULIN ....EMQ

NERVE SUPPLYING PERIANAL SKIN,PUDENDAL NERVE ORIGIN,LA DURING EPISIOTOMY IS GIVEN FOR?????......EMQ EMQ receptors for insulin prostaglandins estogen progesteron primary syphilis in early untreated period options syphilis treponemal antibody test syphilis hemaglutination test syphilis immobilization test emq traetment of chlamedia ..................bacterial vaginosis emq pre malignant conditions emq statistics calculate n to be traetad emq diagnosis of iron defecincy anaemia folic acid .................. early untrated syphylis emq congenital adrenal h icreased ? SUBSTANCE

emq embryology 4 cells ? day vertebra formation mcq about DIGITALIS, cyclosporine .mitochondrial dna ,cd8 cells , the uterer(2 mcq) ,3 repeated statistics mcqs, EMQ 36 WS PRIMI GRAVIDA had flu like sympt , the baby was born jaundiced , hepatosplenomegaly ,skIn rash , brady cardia ,,,,choose the cuase (all viruses ,bacteria,listeria,plasmodium )??????

EMQ ...why we do local infiltration + pudendal block EMQ ...cause of post operative infection ??pseudomonas

EMQ .....WHICH vit or mineral absorbtion is affected by phytate (no iron in the choices

MCQ citric acid cycle fatty acid oxidation ???? genetic imprinting ?????stylomastoid foramen spinal cord in smooth muscle cells /regeneration possible /actin &myosin present/lengh of fibers repeated 2 mcqs u/s and radiation effect apoptosis 2 mcqs about hyperplasia pelvic splanchnic nerve Paper 1 & 2 EMQs A lady is about to deliver and you are about to give her a pudendal block.. a) ant division of ant. remi of S2-4 b) ant division of post.rami of S2-4

b)genitofemoral nerve c) post division of ant. remi of S2-4 d) post division of post. remi of S2-4 e) sensory supply of perineal nerve f) inferior rectal nerve g) ischial tuberosity h)ischial spine Q1 -wht is the root value of pudendal nerve? Q2- wht nerve supplies lower part of vagina (I think)? Q3 why do u give local skin infiltration before episiotomy? If we take the day of fertilization as Day0 then Day2 Day4 Day 8 Day 10 Day12 Day 14 Day 18 Day20 Day 22 Day 24 Day 26 Day 42 Day 56 Day 70 Q4-Which represents 4 cell stage ? Q5- conceptus implants completely? Q6-feable hrt pulse seen on ultrasound? Q7- vertebra form completely? Table: on different values given for deaths due to maternal mortality with direct and indirect causes as well as # of days from delivery to the cause of death Deaths due to placenta previa and hypovolemia x @ e.g 20 days,10 days,8days etc suicide(with previous hx of psychiatric illness) crash victim diabetic neuropathy

two more causes given with values

Different numbers were given in options Q8-the number of late maternal deaths due to indirect causes? Q9-number of maternal deaths due to direct causes? Calculate Numbers needed to treat from a drug trial on osteoporosis: (Q frm EMQ book) Options 25 50 100 250 300 400 1000 others Q10-N1=10,n2=5,n3=9990,n4=9995 Q11-N1=50,N2=25,n3=9950,N4=9975 FSH Oxytocin GH Prolactin Rennin ACTH etc Q12-which is a protein with alpha and beta subunits ? Q13 which stimulates mineralocorticoid activity/relsease? Q14- which hormone is produced by posterior pituatory? Folic acid Magnesium Calcium Vit b1-B12 given

Q15- which vitamin is required in 400mcgs at preconception? Q16-which vitamins absorption is hampered by oxalic acid & phytates? Insulin & Glucagon excretion Decrease decrease Increase increase Increase decrease Deacrease increase Nochange no change Nochange increase Nochange decrease Etc Q-17-on taking a large protein meal? Folowing are risk factors for cancer Q-18- HCC ans Hep B Q19- Skin cancer-ans UV radiation Q20- Cervical/uterine cancer-ans HPV Q21- Breast cancer-ans BRCA1 mutation (Repeats from march 2008) Hormone receptors for following are located in a) protein kinase nuclease receptor b) nucleae transcription receptor c) G-linked cell membrane receptor d) G-linked golgi apparatus receptor e) Protein complex intra cytoplasmic receptor f) Multiligand receptor in cell membrane etc g) Protein Tyrosine kinase activity receptor on cell membrane Q22- Insulin Q23-PG E2 Q24-Estrogen Q25-Progesterone

Q26-A family with one kid presumed to have cystic fibrosis however cftr gene mutation was not detected. Family members willing to give samples if needed.mother wants definite diagnosis as she is 11wks and wants to know diagnosis for her baby.parents are cousins with 1st degree relatives having cystic fibrosis. Invasive diagnostics can be applied to reach final results by? Q27-you have Guthrie spot of their son how will u diagnose him for cystic fibrosis? Q28- families in uk with cystic fibrosis how will u find f508? Options: Pcr Fish test Chromosomal linkage analysis Telomer analysis Sum hereditary crosslinkage chromosomal option (dnt remember) Screen for some known gene mutation Sweat test Saliva test EMQ 1 ST PAPER WHICH NERVE SUPPLIES PERIANAL SKIN EMQ 2 CYSTIC FIBROSIS a - invasive testing will be done ( she is preg 11 ws) which test b- their child blood is ready for guathri test ,which test you will do c- the relatives are willing to be tested , which test will reveal the 405 mutation for families in great braiten any one have suggetions more options were given for these qs karyotyping comparative genomic hybridization flourecent ???? MCQ

propranolol smooth endoplasmic reticulum anal canal smooth muscles (2 mcq ,paper1 ,2 ) primitive streak fetal circulation ??microtubules digitalis mcq ; the following are bacteria cryptococcus ,candida,histoplasmosis, mcq : the embryological origin of peritoneum,.... mcq mitosis miosis mcq : the follwing are structral abnormalities of chromosomes mcq : ?..................increase with preg mcq : drug interaction methotrexate , sulphonamides

mcq :arterial and mean blood pressure mcq : causes of alkalosis mcq : the folloing are types of cells with their secretion ? endothelial cells heparin ?mast cells igE ? plasma celle (i am not sure of the exact coupling) mcq at 20 ws gestation rbcs are made in liver lymph nodes spleen bone marrow mcq about pudendal nerve mcq ischiorectal fossa mcq ureter in abdomen mcq ureter in pelvis mcq foramen of winslow (in detailes)

mcq kallman syndrome in men mcq hcg mcq trophoplasts mcq ? all types of cells in islets of langerhans and their products mcq the following are premalignant conditions: paget disease ...(repeat mcq endotoxins or exotoxins ( i am not sure) MCQ CORPUS LUTEUM mianly endothelial clls secerets inhibin function decrease after 12 ws MCQ INHIBIN

MARCH 2008
cystic fibrosis guthic spot test receptors in apoptosis caspa arias stella reaction blood picture in PET and in preg is inflam like 1.EMQ paclitaxel carboplatin side effects 2.EMQ complex partial seiizure gum hypertrophy,acne ,facial coarsening,vit K TO BE given..?drug used 3.EMQ oestrogen progesterone receptor site 4.EMQ on vitamins-xerophthalmia..morning sickness..macrocytic anaemia. 5.MCQ rubella incubation period,specific immunity within 15 days.. 6.EMQ HEPATOCELLULAR carcinoma,..?? 7.EMQ lining epithelium of ureter,..?? 8. EMQ montoux rection.. 9.MCQ ovulatory DUB 10.MCQ concerning aneurysm- 10% due 2 inflam.,syphilitic aneu,thoracic vessels,marfan recessive

if a mother has a child with cystic fibrosis ,she is preg and will do amniocentesis a. which test to be done (every possibility mentioned) b. all relatives will volunteer to do gutic spot test ,whichtest you will carry on the blood -graph with hormones of the endometrial cycle -citric acid cycle (graph) microsatellites alleles - cd4 -nk cells -down syndrome -pulmonary embolism anal canal -vagina -ovary

phenylketonuria inhibin passive transport erythropoietin renin hcg levels syphylis rubella chlamydia hormone dependent malignancies(testicular carc._sertoli leydig-clear cellcarc.ofkidney-thyroid carc.) pigd missed abortion MCQ congenital heart emq crown \heel lenghth at birth crown/rump length 6 cm =? weeks

about 10 questions statistics plrese does anyone remember the q about ring Y chromosome q about sattellite alleles ?? passive diffussion not depend on . concenteration gradiant molecular size

2following tissue are capable of regeneration spinal cord liver epedermis myocardium bone marrow

3 double blind trial delliosion of HCG in the urine pregnancy test about the q APOPTOSIS and capsase ,q methods of disinfection and BOWIE DICK test where is source to study these subjects? q progesterone receptors q about progesterone receptors syphilis , toxoplasma , hpv , immune responses EMQ type of hpv causing benign wart type of hpv causing cancer cx MCQ down syndrome associated with duodenal atresia true associated with ambig genetalia false only maternal chromosome F

only paternal chromosome F MCQ oogonia miosis starts at puberty miosis before mitosis during S phase chromosomes are doubled the most diagnostic test to be done on amniocentesis gautic spot test what test you do with the blood sample emq about vitamins (hyperemesis) i wrote vit B6 and (xerophthalmia) q about endometrial hyperplasia q about side effect of drug (carboplatin)-(taxal) q about arias stella if its specific in pregnancy q about chlamydia q about varicella q about hpv ,wart answers 16 or 18 or 6,11 q about hcg conc in pregnancy citric acid cycle diagram hormones diagram fsh,lh,estrogen,testosteron q about:by transvaginal u/s yolk sac apear in which week many questions in genetic about pcr,fish test q about laser q in pathology about totipotent cell,tumour like mass q about voiding presure q about ca requirement in pregnancy q about test used to diagnose folic acid deficency q about thalasthmia q about cervical ectropion cells i choose columner cell - mri contraindicated in early pregnancy q about stallete instability in dna - transverse abdominus ms attached to lumbar transverse process -cut s1 lead to autonomic bladder

q about cancer which are hormone dependent -breast -prostate -thyroid q about guthre test q about cystic fibrosis and diagnosis -swet test -karyotyping-fish test q about disinfection and sterelization q about complication of contraceptive pills emqs 1lining of ureter 2.erythropoeitin and renin q from john duthie 3complications of cisplatin 4 paclitaxel 5statistics -std error calculation 7 mean in a normal distribution 8occipito frontal diametr 9hcg titre at detection by tvs 10 at 3 days aftr fertilization 11crl in scan is 60mm what is gestational age 12crl at term ? 13 parietal sutur is between 14kuffer cells are -phagocytic 15 estradiol receptors 16 progestrone receptors -we know that thes are intra nuclear but there were 2 confusin options with intranuclear plz refer this in depth , one option had intranuclear kinase or somethin 16anticonvulsant with description of phenytoin 17vitamin deficiency causin hyperemesis 18 - macrocytic anemia 19 -xerophtalmia 20 cystic fibrosis 3 emqs 24 citric acid cyle diagrm , enzymes were options complement causes 26 b thalasemia detection 27 folate def detection

29 basal cell ca - local malignant i think 30tumor like hamaroma

March 2007
Copper IUD mode of action: sperm motility microthrombi MRI: ionising radiation? no adverse effect on fetus? detect fetal cardiac abn? USG: probe: array of magnets? doppler to detect fetal heart movement? facial hair increased by: oestrogen? testerone? spirolactone? characterics: pass thro' origin? slope = 0? pass thro the mean? unaffected by changing scale? unaffected by changing dependent variable? Copper containing IUDs: F should be changed every year F have a high incidence of actinomycoces colonization than plastic devices T Cause a relative increased in ectopic pregnancy T have been implicated as a cause of fatal infection in pregnancies F do not cause menorrhagia Comments: Inert devices can be left in place until the menopause, but copper devices need renewal every 3-5 years, depending on the make, because of the gradual absorption of copper. Copper IUCDs produce local

concentrations of copper salts which apparently give some protection against bacterial contamination. Pelvic infection with actinomyces organisms is most likely with a plastic device that has been in situ for some years. While the rate of intrauterine pregnancy is reduced, that of ectopics is not. Hence, there is a relative increase in ectopic pregnancy after IUCD insertion. If an IUCD is left in place there is a slight risk of intrauterine infection, preterm labour and antepartum haemorrhage, but most pregnancies are uncomplicated and the device is delivered with the placenta. Increased menstrual loss may be caused by increased fibrinolytic activity which occurs round the IUCD. The progestogen intrauterine system (IUS) reduces menstrual flow and often dysmenorrhoea. Which of the following is/are true concerning MRI? A It involves ionizing radiation (False) B Has no recognised side effects on the foetus (True) C The pregnant mother should be turned to her left side during scanning (True) D Tissue with high hydrogen concentrations are difficult to distinguish False) E Blood vessels appear white on scanning (False) Comments: Water is a molecule composed of hydrogen and oxygen atoms. The nuclei of the hydrogen atoms are able to act as microscopic compass needles. When the body is exposed to a strong magnetic field, the nuclei of the hydrogen atoms are directed into order - stand "to attention". When submitted to pulses of radio waves, the energy content of the nuclei changes. After the pulse, a resonance wave is emitted when the nuclei return to their previous state. The small differences in the oscillations of the nuclei are detected. By advanced computer processing, it is possible to build up a three-dimensional image that reflects the chemical structure of the tissue, including differences in the water content and in movements of the water molecules. This results in a very detailed image of tissues and organs in the investigated area of the body. In this manner, pathological changes can be documented.[/color:febed2ebf4][/b:febed2ebf4]--------------------------------------------------------------------Many stats about sensitivity, positive predictive value, accuracy, even the linear regression Acute UTI in pregnancy

rarely symptomatic ?F Copper IUD mode of action: ?T sperm motility ?T microthrombi ?T intervent ovary movement MRI: ionising radiation F no adverse effect on fetus T detect fetal cardiac abn ?F ( I think it will be very small) USG: probe: array of magnets? ?F doppler to detect fetal heart movement T facial hair increased by: oestrogen?F testerone?T spirolactone F ( I think it is used for treating hirsutism) Mycobacteria ?T strict ?? *(nor sure which word they use ) aerobic ------------------------------------------------------------------48---if a distribution of results is markedly skewed to the left(sep2000) a-the mean is same as 50th centile f b-the same no of values lie on either side of the median- t c-the mode is equal to the median- f d-the students test should be used to compare this distribution with another- f e-logrithmic transformation of the results will produce a distribution close to normal-t -concerning the ablity of a test to predict disease a-sensitivity is the ablity to predict those with disease correctly T b--sensitivity is the same as positive predictive value F c-the confidence interval must cross 1 to prove significance t (NOT A MUST as CI at 1 is perfect ) d-an odds ratio of 1;3 implies the risk of 33% f e--an odds ratio of 2 indicates a halving of risk F

-----------------------------------------------------------------------------------1-SUCCESSFUL LACTATION IS a-maintained by estrogen -b-maintained by progesterone -c-initiated by prolactin surge-d-maintained by human placental lactogen e-inhibited by dopamine 2-THE RELEASE OF CATECHOLAMINES FROM THE ADRENAL MEDULLA INCREASES a-during sleep b-when the nerves to adrenal glands are stimulated c-when the blood sugar rises d- immediately following a myocardial infarction e- in the presence of pheochromocytomaT F-DURING ACUTE HAEMORRHAGE 3- In the pathogenesis of thrombosis a-prostacyclin induces platelet aggregation b-plateletes synthesis thromboxane A2-c-thromboxane A2 induces vasoconstriction-d-contact with subendothelial collagen causes platelete aggregation-e-thrombin inhibits platelete aggregation --4- CONVERSION OF GLUCOSE TO LACTIC ACID a-occurs in single enzymatic reaction b-is the only pathway for synthesis of ATP in red blood cells C-is a reversible process in skeletal muscle d- is inhibited by high cellular conc of ATPe-occurs in skeletal muscle when availability of oxygen is limited5-GLUCOCORTICOIDS a- promotes hepatic gluconeogenesisb- suppress uptake of glucose by musclecpromotes protein breakdowndpromotes fat breakdowne-increase glycolysis in adipose tissue6-Actinomycete israelii

1-Is a fungus 2- forms yellow granules in pus 3-is a commensal in mouth-4- is a commensal in vagina-5- is usually resistant to penicillin 7-Following substances increase the serum uric acid concentration( a-colchicine-b-chlorothiazide-c-allopurinol-d-probenecid-e-phenylbutazone 7--THE FOLLOWING ARE INHERITED AS AUTOSOMAL RECESSIVE CONDITIONS A. tuberous sclerosis. b-phenylketonuria-C. ahondroplasia.-D. sickle cell anaemiaE. Von Giek l disease. 8-. Genes on sex chromosomes are responsible for the inheritance of ( A. glucose-&pbosphate dehydrogenase defiency. B. defective colour vision. C. hairy ear rims. D. homocystinuria. E. Hurler syndrome. 9-in human a haploid no of chromosomes is found in) a-red blood cellsb-blastocystsc-primary oocytes-. d-the first polar bodye-spermatozoa10-osteoprosis is associated with a-an increase in uncalcified bone matrix(osteoid tissue b-prolonged oestrogen therapy

c-a normal histological bone structured-bone fracture e-irregularity of epiphyseal plates 11-THE FOLLOWING HAVE an anti-emetic effect a-hyoscine bromide b-morphine sulphate c-chlorpropamide-d-promethazine hydrochloride e-perphenazine 12CLOMIFENE CITRATE a- is an antiandrogen b-does not stimulate ovulation directly c-can produce visual disturbances-d-is generally prescribed throughout the proliferative phase of the menstrual cycle e-in the treatment of anovulation increases the risk of multiple pregnancy 13--if a distribution of results is markedly skewed to the left a-the mean is same as 50th centile b-the same no of values lie on either siDe of the median c-the mode is equal to the mediand-the students test should be used to compare this distribution with anothere-logrithmic transformation of the results will produce a distribution close to normal15. when a man has hemophilia a. 50% of his daughters would not expected to be carriers b. 25% of his sons expected to be carriers c. Good medical control of blood definition reduce the risk of this condition in his children d. His new born child is likely to require an urgent blood transfusion e. His sister has 50% probability of being a carrier 16- THE INTERSTITIAL CELL (LEYDIG) OF THE TESTIS a--secrete seminal fluid b-are stimulated by LH c-secrete androgen binding protein

d-secrete fructose e-produce testosterone T in response to LH -17--CONCERNING VIRUSES a-the core of every virus contain RNA b-they usually produce intracellelur toxin causing cell death c-antibodies are directed against capsular protein d- they can be grown in intact cells-e-interferone are synthetic antiviral substances 18-THE FOLLOWING ARE CYTOTOXIC ALKYLATING AGENTS( a-cyclophosphamide b-mercaptopurine-c-chlorambucil-d-fluorouracil-e-methotrexate19-2,3 DIPHOSPHOGLYCERATE a-is present in higher conc in maternal erythrocytes than fetal erythrocytes-b-binds more avidly to haemoglobin A than to haemoglobin E c-increases the affinity of haemoglobin for oxygen d-is a phospholipid e- is synthesised by the pentose phosphate pathway 18-HYPERKALEMIA IS A CHARACTERSTIC FINDING IN TINDALL a-primary aldosteronism b-treatment with Spironolactonec-hyperparathyroidismd-ACTH secreting tumours of bronchuse-renal failure19- CONCERNING INHERITABLE DISEASES A-huntigdon ,s chorea is transmitted by a dominant gene b-phenylketonuria is transmitted by a recessive gene c-haemophillia ia an autosomal dominant condition-d- Von Willebrand disease is a sex linked condition e-cystic fibrosis is transmitted by an x linked recessive gene--

20-Early blood borne dissemination is a characteristic feature of: a) carcinoma of the endometrium b) osteosarcoma c) basal cell carcinoma d) carcinoma of the cervixe) choriocarcinoma21- in tissue pigmentation the following are associated a-kernicterus and conjugated billirubin b-addison disease and increased cutaneous melanin c-melanosis coli and bile pigmentsdwilson disease and copper deposition in the corneae-corpus leuteum and carotenoids22-acquired diverticular disease of the colon a-is present in at least 15% of Caucasian over the age of 50 yearsb-is due to congenital abnormality of the bowel wall c-is associated with increased intraluminal pressure d-is associated with muscular thickening e-may result in intestinal obstruction 23 stored blood which is to be used for transfusion a-kept at -4 degree b-must be used in 1 week c-is tested for compliment content before transfusion -d-may be used for platelet replacement e-contains an acid anticoagulant 24- in uncomplicated homozygous beta thalasemia there is a-hypochromasia-b-a reduction in haemoglobin A c-an increase in haemoglobin F d- RED CELL SICKLING e-presence of megaloblasts in bone marrow32-The following are capable of cellular regeneration a)spinal cord b)liver parenchyma c)gut epithelium d)kidney ???

e)bone marrow 33-during the development of female reproductive system a-primordial germ cells arise in yolk sac-b-ovarian development is dependent upon oestrogen activity-c-the paramesonephric duct give rise to the cervix-d-the greater vestibular glands arise from the urogenital sinus-e-differentiation of external genitalia is dependent upon ovarian activity-36- IN CONGENITAL ADRENAL CORTICAL HYPERPLASIA a-commonest deficiency is C21 b- plasma cortisol conc is raised c-there may be excessive secretion of 17 hydroxy progesterone-d- sodium retension is characterstic e-blood catecholamine conc are increased 37-- SCHISTOSOMA HAEMATOBIUM a- is a snail b- is prevalent in china c- infestation may affect the uterine cervix d- give rise to chronic granulomatous lesions-e- infestation predisposes to carcinoma 38- KETONE BODIES . a-can be utilised by theadult brain. B-Include acetone c-are water soluble -d- are synthesised in skeletal muscles e-can be utilised during starvation 39-IRON IONS a-diffuse passively into erythropoitic cellsb-bind to transferinc-are taken up by hepatocytesd- are necessary for cytochrome synthesise-are absorbed predominantly by the ileum40-- --the conjugation of billirubin

a-takes place in hepatocytesb-is catalysed by UDP glucuronyl transferasec-is inhibited by phenobarbitoned-renders it water solublee-is impaired in acute billiary obstruction41- in radiotherapy( a-1Gray is eq to 1joule per kg b- the skin usually recieve a greater dose of radiation than underlying tissuec-the major effect of radiation energy is to damage the cytoplasm of the celld-cells in tissue which are hypoxic are more vulnerable to radiatione-radiation induced changes in tissues may take 6 weeks to develop-42--osteomalacia is characterised by a-mineralization of the periostium b-deposition of uncalcifiedbone matrix -c-normal osteobastic activity -d-increase capillary fragility e-normal calcification of bone 43-progesterone a-is a C21 compound b-synthesised by the ovary before ovulationc-increases ventilationd- raises BMRe-binds to corticotrophin binding globulin44-EPIDERMAL GROWTH FACTOR A- is mitogenic-. b- synthesis is stimulated by oestradiolc-is a steroid molecule-F ... d- is found in endometrium-e- binds to receptor on the nuclear membrane45-the following are structural aberrations of chromosomes: a. deletions -b. inversions

c. aneuploidy d. . polyploidye translocations 46-- Messenger RNA a-Synthesis is dependent on RNA polymerase b-is an exact copy of sense DNA c-Contains exons d-Is measured by Western blotting e-translation occurs in the nucleus 47-- BETA SYMPATHOMIMETIC DRUGS MAY A- caue bronchospasm b-reduce frequency of uterine contraction c-cause heart block d-reduce diastolic blood pressure e- increase blood glucose conc 48- The following statistical statements are correcta-in the normal distribution,the value of mode is 1.73 times the median b-in a distribution skew to the right,the mean lies to the left of medianc-in the series- 2;7;5;2;3;2;5;8, the mode is 2 d-students t test is designed to correct for skew distribution e-the chi squared testmay be used when data are not normally distributed50-Contraindicated in breast feeding A-POPs B-BromocriptineC-chloroqunined-?? e-warfarineF 51-Contraindicated with kidney impairment A-Dopamine B-Gentamycine C-Cephahexin D-Cis platin 52- the following are characters of hypoxic cell death:

A-Apoptosis -B-Phagocytosis C-Pyknosis -D-Poikilocytosis E-Release of phospholipids 52- the following are characters of hypoxic cell death: A-Apoptosis -B-endocytosis C-Pyknosis -D-Poikilocytosis E-koliosytosis prostagladinsa-lipooxygenase pathway b-thromboxane causes vasoconstriction c-increased in the myometruim d-? e-? linear regression: a-starts from the origin b-passes thru the meanc-values vary d-ranking is done??? following test r used to compare 2 drugs a and b a-student t test b-meta analysis c-wiloxone paired test withney u paired test e? toxic shock syndrome associated with the use of tampoons b-due to toxigenic starinsof strep c-infrequently reported outside n america d-a consequence of previous antibioitic therapy e-confined to seualy active women the following can regenerate

a-kidneyepitheluim? b-pheriheral axonsc-liver parenchyme d-bone marrow e-bone radiation sensitive a-bone b-bone marrow c-epith of gut d-skin e-? Mycobacteruim a- are alchochol acid fast b-does not form spores c-d-a facultative anaerobes d-responsible for leprosy e-pathogenic inhumans all strains the germination of tetanus spores in a wound is inhibited by a-tissue trauma b-oxygen c-inject of toxoid d-injection of antitoxin e-removal of devitalised tissue Aldosterone a-reduces Na resorption in PCT b-reduces Na absorption in descen loop of henle c-Increase Na absortion in DCT Increase K loss from the tubule e increases Na absortion in collecting tubules the following r conclusive evidence of pregnancy in uterine cureetings???? can, t remmber a-decidua compactab-Arias -stella changes in endometrail gladc-spiral arterioles d-plasma cell infiltration e chorionic villi

White cell migration from the bld vessels in areas of inflammation involves--- march 1997/2 21 a-call migration between the endothelial cells b-a pssive loss of fluid bld elements c-cell migration independeant of endothelial cell motion d-initail emigration of polymorhs neutrophils e-more polymorphs than monocytes after 2 day= the vulva is supplied by a-ant cutaneu erve of the thigh b-femoral br/o genito femoral nerve c-? d? e? glucocorticoids a-promote hepatic gluconeogenesis b-suppress uptake of glucose by muscles c-promote protein break down d-promote fat reakdown e-increase glycolysis halothane(refer sept 1997)42 a-cardic arrythmias b-explosive mixtures with air c-liver damage 53- U/S: A-Pulstile B-Increase body temp 1 degree after 15 min scanning.. C-High frequency penetrates deeper TISSUE,D-Can distinguish between 2 points closer than 0.5 mm E-Best echoes are produded by beam at right angle to the structures. 54-Disinfectant solutions may become contaminated with: 1-Enterobacter species 2-Streptococcus species 3-Escherichia coli 4-Pseudomonas aeruginosa 5-Staphyloccus pyogenes-

55-Nitric oxide a) is synthesised in the endothelium b) has a short half life -c) causes smooth muscle contraction d) increases during pregnancy e) combines with oxygen to produce L-arginine but few question got modifications eg; nitrous oxide synthesised by macrophages naloxone antiemitic effect drugs contraindicated in breast feeding drugs causing renal impairment ultasound mri statistics quit difficult all definitions rna pre implantation diagnosis FIBRINOGEN (1 2) HEPATOGLOGIN 3) FERRITIN 4) ... predominantly produced by UG system - is a kind of lukotriene - is phospholipid which reactant protein will increase in injury -------------------------------------------------------------------------------1) FIBRINOGEN 2) HEPATOGLOGIN 3) FERRITIN 4) albumin 5) ? Cell cycle -prophase I and II

-mitosis I and II Vulva nerve supply Urinary bladder - lining/embrology vagina - embrology Fetal testis IL I MCH I Function of Neutrophil polymorphs Fetal HbF PG Umblical cord embrology prostaglandins half lifa 30 min f il1 produced by macrophages whartons jelly originate from extra embryonic endodem detrusor muscle is of mesodermal origin acute phase proteins are transferrin alfa feto protein preimplantation diagnosis

possible 2 detect the sex of the embryo uses pcr 4 chromosomal deletions can be used 4 detection of single gene disoderes facial hair causes testosterone mestranol spironolactone real time bias length time bias negative predictive value fibrocystic disease of thr breast...in paper 2 , breast anatomy and breast from endocrine function (in paper 1..)...ie 3 qs we about the breast levator ani its attachnment and its coverings endometrial carcinoma adenomyosis about measuring the arterial blood pressure and its methods interleukin 2 prenatal implantation genetic diagnosis menstural cycle in paper 1 and 2

MARCH 2006
The germination of tetanus spores in a wound is inhibited by a)tissue trauma b)oxygen c)injection of anti-toxin d)injection of toxoid e)removal of devitalised tissue The following values fall within the normal range for the adult female bladder a)residual volume of 100ml b)voiding capacity of 250ml c)bladder capacity of 900ml d)intravesical pressure rise of less than 10cm H2O during early filling e)maximum urine flow rate of 60ml per second

In the small intestine, the following substances are absorbed by active processes a)water b)sodium c)vitamin K d)amino acids e)chloride White cell migration from blood vessels in areas of inflammation involves a)cell migration occurring between endothelial cells b)a passive loss of fluid blood elements c)cell migration independent of endothelial cell motion d)initial emigration of polymorphonuclear neutrophils e)more polymorphs that monocytes after 2 days Antibodies (!!) a)are soluble proteins b)are formed in the fetus before 12 weeks of intrauterine life c)have an average molecular weight of around 10000 daltons d)of the rhesus type are genetically transmitted e)are produced by the ribosomes of plasma cells In DNA a)a codon is a sequence of three bases b)all codons have an identified function c)there is a greater variety of amino acids than there are different codons d)replication can be initiated at several different points along a chromosome e)complementary pairing precedes messenger mRNA synthesis The inferior vena cava a)is formed at the level of the fifth lumbar vertebra b)commences posterior to the right external iliac artery c)receives the left ovarian vein d)receives the right ovarian vein e)pierces the central tendon of the diaphragm In the normal human pelvis a)the promontory of the sacrum is in the upper anterior border of the first sacral vertebra b)the anterior surface of the sacrum has five paired foramina

c)the joint between the two pubic bones is a synovial joint d)the acetabular fossa is wholly formed from parts of the pubic and ischial bones e)the transverse diameter of the brim is greater than the anteroposterior diameter Arginine vasopressin a)reduces the glomerular filtration rate b)controls water loss in the proximal renal tubule c)is synthesised by the posterior pituitary gland d)is released in response to a rise in plasma osmolality e)is released in response to a fall in circulating plasma volume Concerning ovarian function a)progesterone is the major steroid of the developing follicle b)granulosa cells secrete oestradiol c)oestradiol is derived from androgen precursors d)insulin-like growth factor (IGF-1) is not secreted by the ovary e)circulating inhibin concentrations are a marker of granulosa cell function Actinomyces israelii a)is a rickettsia b)forms yellow granules in pus c)is a commensal in the mouth d)is a commensal in the vagina e)is usually resistant to penicillin Halothane produces a)cardiac arrhythmias b)explosive mixtures with air c)liver damage if given repeatedly d)myometrial relaxation e)bronchial irritation The therapeutic effect of the first drug is enhanced by the second drug a)phenytoin: ethinyloestradiol b)bromocryptine: metoclopramide c)penicillin: probenicid d)ritodrine: dexamethasone e)warfarin: phenobarbitone Potassium

a)is mainly intracellular b)plasma levels vary in proportion to intracellular levels c)plasma levels are decreased in Addisons disease d)plasma levels are increased in diabetic ketoacidosis e)deficiency occurs with prolonged vomiting Concerning carbohydrates a)sucrose is a disaccharide of glucose and fructose b)cereal grains contain less than 40% starch c)cellulose is a fructose polysaccharide d)a normal diet contains less than 60g of carbohydrate daily e)dietary carbohydrate is oxidised in the body to carbon dioxide and water Steroid hormones a)all contain 20 carbon atoms b)can be produced by structures of urogenital ridge origin c)are mostly activated in the liver d)are predominantly excreted unchanged in the urine e)mainly circulate unbound to carrier proteins Ventilation is increased due to stimulation of central receptors by a)nikethamide b)hypoxia c)doxapram d)phenobarbitone e)salbutamol Tetrahydrofolic acid a)is involved in purine synthesis b)is a precursor of folic acid c)is a coenzyme in amino acid synthesis d)catalyses the conversion of glucose to glucose-6-phosphate e)activity is inhibited by Methotrexate Mitochondrial DNA a)is located in the nucleus b)inheritance is patrilineal c)is present in two copies per cell d)mutation causes cystic fibrosis e)is involved in the control of oxidative phosphorylation The following conditions may lead to hydronephrosis

a)mercury poisoning b)cervical carcinoma c)renal calculi d)renal vein thrombosis e)posterior urethral valves In uncomplicated homozygous beta thalassaemia there is a)hypochromasia b)a reduction in haemoglobin A2 c)an increase in haemoglobin F d)no depletion of iron stores e)the presence of megaloblasts in bone marrow The following statements relate to lung function in normal pregnancy a)vital capacity is increased by about 50% b)tidal volume is increased c)the subcostal angle increases d)the residual volume is reduced e)the respiratory rate is increased In normal pregnancy, uterine blood flow a)is about 50ml/minute at term b)is maintained throughout the cardiac cycle to the choriodecidual space c)is reduced by prostacyclin d)is increased during uterine contractions e)represents about 10% of the cardiac output by the end of the first trimester During normal pregnancy a)arterial pCO2 decreases b)the blood hydrogen ion concentration decreases c)plasma bicarbonate concentrations decrease d)urine pH falls e)lactic acid production is increased The anal canal a)has an upper part which is innervated by the inferior hypogastric plexus b)has a lower part which is supplied by the superior rectal artery c)drains lymph to the superficial inguinal nodes from its upper part d)has its internal sphincter innervated by the inferior rectal nerve e)has a superficial part of its external sphincter attached to the coccyx. Human placental lactogen

a)is a single chain polypeptide b)reaches the same concentration in fetal and maternal blood at term c)may be secreted by the decidua d)is detectable only after the 25th week of pregnancy e)is an insulin antagonist Human chorionic gonadotrophin !! a)is a glycoprotein b)is detectable 48 hours after fertilisation occurs c)secretion peaks at 20 weeks of gestation d)is synthesised by the corpus luteum of pregnancy e)binds to luteinizing hormone receptors Cholecalciferol (vitamin D) a)promotes the absorption b)is 25-hydroxylated in the liver c)is synthesised in the SKIN d)is 1-hydroxylated in the kidney e)is most active in the 1,25-dihydroxyl form Unconjugated bilirubin !! a)is normally present in the plasma in lower concentration than conjugated bilirubin b)circulates in the plasma bound to albumin c)is not excreted in the urine d)does not cross the blood-brain barrier e)crosses the placenta The following are capable of cellular regeneration: a)spinal cord b)liver c)epidermis d)myocardium e)bone marrow The Leydig cells of the testis !! a)secrete seminal fluid b)are stimulated by luteinising hormone c)are active in intrauterine life d)secrete fructose e)produce androstenedione Angiotensin II !!

a)is the most potent vasoconstrictor b)reduces aldosterone production c)is mainly found in the lungs d)is a decapeptide e)is produced when the extracellular fluid volume is reduced Cyproterone acetate a)is an oestrogen b)is used for the treatment of amenorrhoea c)binds to androgen receptors d)increases libido e)inhibits spermatogenesis Lignocaine used as a local anaesthetic !! a)causes tachycardia if given as a systemic injection b)has a longer lasting action than bupivicaine c)is used in combination with adrenaline for ring block d)causes vasoconstriction e)is a weak base Plasma osmolarity in the human a)is normally about 290 milliosmoles per kg in the nonpregnant state b)increases during the first trimester of pregnancy c)is closely controlled by plasma protein concentration d)is regulated by arginine vasopressin e)regulates the sensation of thirst In the statistical analysis of any group of numerical observations a)the mean is always less than the mode b)the median value always lies at the mid-point of the range c)standard deviation is always greater than the standard error of the mean d)the standard error of the mean is independent of the total number of observations e)there are the same number of observations greater than and less than the median value The pelvic splanchnic nerves a)are derived from the posterior rami of the sacral spinal nerves b)supply afferent fibres c)mix with branches of the sympathetic pelvic plexus d)supply the ascending colon with motor fibres e)supply the uterus with parasympathetic fibres

Exotoxins a)are derived from gram-negative bacteria b)have specific action c)are more toxic than endotoxins d)are neutralised by their homologous antitoxin e)can be converted to toxoid Candida albicans a)is gram positive b)is an anaerobic organism c)is associated with diabetes mellitus d)is motile e)is inhibited by oral tetracycline therapy

Transferrin a)is one third saturated with iron b)is increased in pregnancy c)binds to 10mg of iron per gram d)levels in the neonate are low e)is actively transported In Crohns disease there is a) non casesous granuloma formation b) formation of deep fissures c) formation of crypt abscesses d) the presence of mucosal polyps e) a recognised association with occurence in the vulva The rectum a) usually commences at the level of S3 b) has a mesentery in its proximal third c) drains lymph to the pre-aortic nodes d) has a parasympathetic nerve supply derived entirely form the S3 spinal segment e) is about 25cm in length The female urethra (!!) a) traverses the perineal membrane b) is lined throughout by urothelium c) has a muscle layer continuous with that of the bladder d) has an external sphincter supplied by the obturator nerve

e) corresponds developmentally to the membranous urethra in the male During menstruation, endometrial haemostasis depends upon a) a platelet and fibrin plug formation after the first 20 hours of bleeding b) oestrogen mediated vasoconstriction c) prostaglandin mediated spiral vessel constriction d) production of plasminogen activating factor by the glandular epithelial cells In HIV, seroconversion illness after a needlestick injury a)occurs within 1 to 4 weeks following exposure b)infectivity decreases after seroconversion c)the CD4 count is a useful adjunct in diagnosis d)measurement of HIV RNA viral load is most useful in diagnosis The following tumours arise in the ovary: a) nephroblastoma b) cystadenoma c) granulosa cell tumour d) neuroblastoma e) teratoma Early blood borne dissemination is a characteristic feature of: a) carcinoma of the endometrium b) osteosarcoma c) basal cell carcinoma d) carcinoma of the cervix e) choriocarcinoma Breast milk compared to cow's milk (!!) a) has more protein b) has more sodium c) contains more carbohydrate d) has more casein e) contains more calories The following are consequences of pulmonary embolism: a) pulmonary infarction b) fibrinous pleurisy c) right ventricular hypertrophy d) sudden death e) haemoptysis

In a country wide survey, 10 general practices were picked at random and 5% of patients were selected at random from each practice: a) the sample of patients is a true random sample b) all practices had an equal chance of selection c) all patients had an equal chance of selection d) two siblings could not have been selected e) inferences about all patients in the country may be drawn from the sample Nitric oxide a) is synthesised in the endothelium b) has a short half life c) causes smooth muscle contraction d) increases during pregnancy e) combines with oxygen to produce L-arginine Carbohydrates a) yield 17kj/g of energy b) are a major source of energy for the brain c) are a a major component of the diet d) give a higher energy yield compared to fats Head mesenchyme gives rise to the following skull bones a)Parietal b)Mandible c)Maxillary d)Frontal e)Ethmoid Pre-implantation genetic diagnosis a) can be used to identify single gene defects b) is used in ICSI only c) testing for trisomy 21 should be offered in a case of ovum donation with a 45 year old recipient and 23 year old donor d) is routinely used in IVF in selection of embryos e) is governed by the regulations of the Human, Embryology and Fertilisation Authority The following are gram positive organisms a)Brucella b)Listeria c)Staphylococcus d)Vibrio cholera

Endometriotic deposits a) only occur in the pelvis b) consist of deposits of endometrial stromal tissue without glands c) do not occur in postmenopausal women d) are hormone sensitive Amyloidosis a)Can be inherited b)Is associated with bronchiectasis c)Is a cause of cardiomyopathy Phenylketonuria a)can only be diagnosed in the adult b)is diagnosed by high levels of phenylpyruvate in the urine c)treatment is with diet restriction DUB (Dysfunctional uterine bleeding) a) is common at the extremes of life b) may be caused by coagulation disorders Chylomicrons a) are not normally present in the fasting state b) after hydrolysis of lipoproteins, mainly consist of phospholipid Other questions that I cannot remember in such detail were about: Insulin (*half life of 30min) Pituitary gland Anatomy (*contains pars tuberalis) Many Pathology questions (equivalents ie *ovarian dysgerminoma = seminoma?, *endodermal sinus tumour = yolk sac tumour?) Embryology (notochord, primitive streak, mesonephros derivatives) Ionising radiation (radiosensitivity ie *liver /intestinal epithelium - which is more radiosensitive; *is radiation exposure of dental x-ray equivalent to 1/5 that from a transatlantic flight) difference b/w human milk and cows milk casien lactose sodium ascorbic acid water phenyl ketoneuria

inherited disorder cause by defficiency of phenyl alanine dietry restriction of tyrosine helpful diagnosed by measuring phenyl ketone level in urine essential amino acids are not synthesized by body sufficiently all ketogenic can produce energy dont remember exact wording radiation mostly harmful in 8-12 wks gestation xrays r ionizing radiation contraindicated in pregnancy transatlantic flight hv higher dose of radiation than xray MRI repeat q from sept paper chylomicrins responsible for turbulence of plsma?dont remem metabolized in adipose tissue -true 80% 20% in liver after hydrolysis by lipoprotien mainly phospholipids r absorbed form intestinal luminal cells to ----? ovarian tumors dysgerminoma and endodermal sinus tumor hv same origin choriocharcinoma can arise cocp increase risk of ovarian cancers -this qs repeated again in other qs with different wording b cells arise from plasma cells in pregnancy humoral immunity is depressed early in pregnany tumor necrotic factor alpha raised in parturition pulmonary embolism cause s sudden death fibrinous plurisy rt ventricular hypertrohy haemoptysis repeated from sept 05 transferrin endo metrium hiv seroconversion carbohydrate preimplantation diagnosis repeat from march 2005 progesteron following rise in leuteal phase

basal body temp progesterone LH--PLZ SOME ONE ADD I DONT REMEMBER NOTOCHORD R SOLID CELLS GIVE RISE TO brain --? SKULL MESENCHYME GIVE RISE TEMPORAL BONE FRONTAL PARIETAL Q-Disinfectant solutions may become contaminated with: 1-Enterobacter species 2-Streptococcus species 3-Escherichia coli 4-Pseudomonas aeruginosa 5-Staphyloccus pyogenes Q-Examples of active transport across a membrane include the passage of: 1-Potassium ions into the neurons 2-Water into the proximal renal tubular cell 3-Glucose from the proximal renal tubular cell 4-Fatty acids into the mucosal cells of the intestine 5-Hydrogen ions from the gastric oxyntic cells. ENDOMETRIOSIS has endometrial tissue with no glands not occurs after menopause The following values fall within the normal range for the adult female bladder Fa)residual volume of 100ml Tb)voiding capacity of 250ml Fc)bladder capacity of 900ml Td)intravesical pressure rise of less than 10cm H2O during early filling Fe)maximum urine flow rate of 60ml per second

The germination of tetanus spores in a wound is inhibited by Fa)tissue trauma Tb)oxygen Fc)injection of anti-toxin Td)injection of toxoid Te)removal of devitalised tissue In DNA Ta)a codon is a sequence of three bases Tb)all codons have an identified function Fc)there is a greater variety of amino acids than there are different codons Td)replication can be initiated at several different points along a chromosome Fe)complementary pairing precedes messenger mRNA synthesis Concerning ovarian function Fa)progesterone is the major steroid of the developing follicle Tb)granulosa cells secrete oestradiol Tc)oestradiol is derived from androgen precursors Td)insulin-like growth factor (IGF-1) is not secreted by the ovary Fe)circulating inhibin concentrations are a marker of granulosa cell function Actinomyces israelii Fa)is a rickettsia Tb)forms yellow granules in pus Tc)is a commensal in the mouth Fd)is a commensal in the vagina Fe)is usually resistant to penicillin In the small intestine, the following substances are absorbed by active processes F a)water T b)sodium F c)vitamin K T d)amino acids T e)chloride White cell migration from blood vessels in areas of inflammation involves T a)cell migration occurring between endothelial cells F b)a passive loss of fluid blood elements

T c)cell migration independent of endothelial cell motion T d)initial emigration of polymorphonuclear neutrophils F e)more polymorphs that monocytes after 2 days Which are formed from intramembranous ossification? T Temporal , occipital, parietal F. Sphenoid, Ethmoid Antibodies (!!) Ta)are soluble proteins -- Ab are glycoprotein. Are they soluble? not sure [color=red:c2901df9c1][b:c2901df9c1]F[/b:c2901df9c1][/color:c2901df9 c1]b)are formed in the fetus before 12 weeks of intrauterine life Fc)have an average molecular weight of around 10000 daltons Fd)of the rhesus type are genetically transmitted Te)are produced by the ribosomes of plasma cells Correction The germination of tetanus spores in a wound is inhibited by Fa)tissue trauma Tb)oxygen [color=red:1e03073b8c][b:1e03073b8c]T[/b:1e03073b8c][/color:1e0307 3b8c]c)injection of anti-toxin Td)injection of toxoid Te)removal of devitalised tissue Insulin (*half life of 30min) F Pituitary gland Anatomy (*contains pars tuberalis)F Q-Disinfectant solutions may become contaminated with: T1-Enterobacter species F2-Streptococcus species F3-Escherichia coli T4-Pseudomonas aeruginosa F5-Staphyloccus pyogenes The inferior vena cava Ta)is formed at the level of the fifth lumbar vertebra Fb)commences posterior to the right external iliac artery Fc)receives the left ovarian vein

Td)receives the right ovarian vein Te)pierces the central tendon of the diaphragm In the normal human pelvis Ta)the promontory of the sacrum is in the upper anterior border of the first sacral vertebra Fb)the anterior surface of the sacrum has five paired foramina Fc)the joint between the two pubic bones is a synovial joint [b:715c81bcd5]d)the acetabular fossa is wholly formed from parts of the pubic and ischial bones [/b:715c81bcd5] Te)the transverse diameter of the brim is greater than the anteroposterior diameter Arginine vasopressin Fa)reduces the glomerular filtration rate Fb)controls water loss in the proximal renal tubule Fc)is synthesised by the posterior pituitary gland Td)is released in response to a rise in plasma osmolality Te)is released in response to a fall in circulating plasma volume Concerning ovarian function Fa)progesterone is the major steroid of the developing follicle Tb)granulosa cells secrete oestradiol Tc)oestradiol is derived from androgen precursors Td)insulin-like growth factor (IGF-1) is not secreted by the ovary Fe)circulating inhibin concentrations are a marker of granulosa cell function Actinomyces israelii Fa)is a rickettsia Tb)forms yellow granules in pus Tc)is a commensal in the mouth Fd)is a commensal in the vagina Fe)is usually resistant to penicillin Halothane produces Ta)cardiac arrhythmias

Fb)explosive mixtures with air Tc)liver damage if given repeatedly Td)myometrial relaxation Fe)bronchial irritation The therapeutic effect of the first drug is enhanced by the second drug Fa)phenytoin: ethinyloestradiol Fb)bromocryptine: metoclopramide Tc)penicillin: probenicid Td)ritodrine: dexamethasone Fe)warfarin: phenobarbitone Potassium Ta)is mainly intracellular Fb)plasma levels vary in proportion to intracellular levels Fc)plasma levels are decreased in Addisons disease Td)plasma levels are increased in diabetic ketoacidosis Te)deficiency occurs with prolonged vomiting Concerning carbohydrates Ta)sucrose is a disaccharide of glucose and fructose [color=red:715c81bcd5][b:715c81bcd5]F[/b:715c81bcd5][/color:715c81b cd5]b)cereal grains contain less than 40% starch [color=red:715c81bcd5][b:715c81bcd5]F[/b:715c81bcd5][/color:715c81b cd5]c)cellulose is a fructose polysaccharide Fd)a normal diet contains less than 60g of carbohydrate daily Te)dietary carbohydrate is oxidised in the body to carbon dioxide and water Steroid hormones Fa)all contain 20 carbon atoms Tb)can be produced by structures of urogenital ridge origin Fc)are mostly activated in the liver Fd)are predominantly excreted unchanged in the urine Fe)mainly circulate unbound to carrier proteins Ventilation is increased due to stimulation of central receptors by

Ta)nikethamide Tb)hypoxia Tc)doxapram Fd)phenobarbitone Fe)salbutamol Tetrahydrofolic acid Ta)is involved in purine synthesis Fb)is a precursor of folic acid Fc)is a coenzyme in amino acid synthesis Fd)catalyses the conversion of glucose to glucose-6-phosphate Te)activity is inhibited by Methotrexate Mitochondrial DNA Ta)is located in the nucleus Fb)inheritance is patrilineal Fc)is present in two copies per cell Fd)mutation causes cystic fibrosis Te)is involved in the control of oxidative phosphorylation The following conditions may lead to hydronephrosis ?Fa)mercury poisoning Tb)cervical carcinoma Tc)renal calculi ?Fd)renal vein thrombosis Te)posterior urethral valves In uncomplicated homozygous beta thalassaemia there is Ta)hypochromasia Fb)a reduction in haemoglobin A2 Tc)an increase in haemoglobin F Td)no depletion of iron stores Fe)the presence of megaloblasts in bone marrow The following statements relate to lung function in normal pregnancy Fa)vital capacity is increased by about 50% (ans from busyspr) Tb)tidal volume is increased

Tc)the subcostal angle increases Td)the residual volume is reduced Fe)the respiratory rate is increased [color=darkblue:715c81bcd5]In normal pregnancy, uterine blood flow Fa)is about 50ml/minute at term Tb)is maintained throughout the cardiac cycle to the choriodecidual space Fc)is reduced by prostacyclin Fd)is increased during uterine contractions Te)represents about 10% of the cardiac output by the end of the first trimester [/color:715c81bcd5] P.200 sep 2001 paper 2 Q 46 During normal pregnancy Ta)arterial pCO2 decreases Fb)the blood hydrogen ion concentration decreases Tc)plasma bicarbonate concentrations decrease Fd)urine pH falls ?Te)lactic acid production is increased The anal canal Ta)has an upper part which is innervated by the inferior hypogastric plexus Fb)has a lower part which is supplied by the superior rectal artery Fc)drains lymph to the superficial inguinal nodes from its upper part Fd)has its internal sphincter innervated by the inferior rectal nerve Te)has a superficial part of its external sphincter attached to the coccyx. Human placental lactogen ?T[b:715c81bcd5]a)is a single chain polypeptide [/b:715c81bcd5] Fb)reaches the same concentration in fetal and maternal blood at term Tc)may be secreted by the decidua ?F[b:715c81bcd5]d)is detectable only after the 25th week of pregnancy[/b:715c81bcd5] ?T[b:715c81bcd5]Te)is an insulin antagonist [/b:715c81bcd5]

Human chorionic gonadotrophin !! Ta)is a glycoprotein ?T[b:715c81bcd5]b)is detectable 48 hours after fertilisation occurs [/b:715c81bcd5] Fc)secretion peaks at 20 weeks of gestation Fd)is synthesised by the corpus luteum of pregnancy Te)binds to luteinizing hormone receptors Cholecalciferol (vitamin D) Ta)promotes the absorption Tb)is 25-hydroxylated in the liver Tc)is synthesised in the SKIN Td)is 1-hydroxylated in the kidney Te)is most active in the 1,25-dihydroxyl form Unconjugated bilirubin !! ?Fa)is normally present in the plasma in lower concentration than conjugated bilirubin ?Tb)circulates in the plasma bound to albumin Fc)is not excreted in the urine Fd)does not cross the blood-brain barrier ?Te)crosses the placenta The following are capable of cellular regeneration: Fa)spinal cord Tb)liver Tc)epidermis Fd)myocardium Te)bone marrow The Leydig cells of the testis !! Fa)secrete seminal fluid Tb)are stimulated by luteinising hormone Tc)are active in intrauterine life Fd)secrete fructose ?Fe)produce androstenedione

Angiotensin II !! Ta)is the most potent vasoconstrictor Fb)reduces aldosterone production Tc)is mainly found in the lungs Fd)is a decapeptide -- angiotensin I is a decapeptide Te)is produced when the extracellular fluid volume is reduced Cyproterone acetate Fa)is an oestrogen [color=blue:715c81bcd5]--is an antiandrogenic progesterone[/color:715c81bcd5] Fb)is used for the treatment of amenorrhoea [color=blue:715c81bcd5]-treat hirsutism [/color:715c81bcd5] Tc)binds to androgen receptors Fd)increases libido Te)inhibits spermatogenesis Lignocaine used as a local anaesthetic Ta)causes tachycardia if given as a systemic injection Fb)has a longer lasting action than bupivicaine Fc)is used in combination with adrenaline for ring block Fd)causes vasoconstriction Te)is a weak base Plasma osmolarity in the human Ta)is normally about 290 milliosmoles per kg in the nonpregnant state Fb)increases during the first trimester of pregnancy ?Fc)is closely controlled by plasma protein concentration Td)is regulated by arginine vasopressin Te)regulates the sensation of thirst In the statistical analysis of any group of numerical observations Fa)the mean is always less than the mode Tb)the median value always lies at the mid-point of the range Tc)standard deviation is always greater than the standard error of the mean Fd)the standard error of the mean is independent of the total number of observations

Te)there are the same number of observations greater than and less than the median value The pelvic splanchnic nerves Fa)are derived from the posterior rami of the sacral spinal nerves Fb)supply afferent fibres Tc)mix with branches of the sympathetic pelvic plexus Fd)supply the ascending colon with motor fibres Te)supply the uterus with parasympathetic fibres Exotoxins Ta)are derived from gram-negative bacteria [color=red:715c81bcd5][b:715c81bcd5]T[/b:715c81bcd5][/color:715c81 bcd5]b)have specific action Tc)are more toxic than endotoxins Td)are neutralised by their homologous antitoxin Te)can be converted to toxoid Candida albicans Ta)is gram positive Fb)is an anaerobic organism Tc)is associated with diabetes mellitus Fd)is motile Fe)is inhibited by oral tetracycline therapy T treat with miconazole T commonsal in intestine

Transferrin Ta)is one third saturated with iron c)binds to 10mg of iron per gram d)levels in the neonate are low e)is actively transported Increased in pregnancy t

In Crohns disease there is ?Ta) non casesous granuloma formation Tb) formation of deep fissures Tc) formation of crypt abscesses Td) the presence of mucosal polyps e) a recognised association with occurence in the vulva The rectum Ta) usually commences at the level of S3 Fb) has a mesentery in its proximal third [b:715c81bcd5][color=red:715c81bcd5]T[/color:715c81bcd5][/b:715c81 bcd5]c) drains lymph to the pre-aortic nodes [color=red:715c81bcd5][b:715c81bcd5]T[/b:715c81bcd5][/color:715c81 bcd5]d) has a parasympathetic nerve supply derived entirely form the S3 spinal segment [color=red:715c81bcd5][b:715c81bcd5]F[/b:715c81bcd5][/color:715c81b cd5]e) is about 25cm in length [b:715c81bcd5]The female urethra (!!) a) traverses the perineal membrane b) is lined throughout by urothelium c) has a muscle layer continuous with that of the bladder d) has an external sphincter supplied by the obturator nerve e) corresponds developmentally to the membranous urethra in the male [/b:715c81bcd5] [b:715c81bcd5]During menstruation, endometrial haemostasis depends upon a) a platelet and fibrin plug formation after the first 20 hours of bleeding b) oestrogen mediated vasoconstriction Tc) prostaglandin mediated spiral vessel constriction d) production of plasminogen activating factor by the glandular epithelial cells [/b:715c81bcd5] During menstruation endometrial hemostasis depends upon Platelet and fibrin plug formation after first 20 hrs of bleeding Production of PAF by glandular epithelial cells

Estrogen mediated hemostasis PG mediated hemostasis

In HIV, seroconversion illness after a needlestick injury Ta)occurs within 1 to 4 weeks following exposure [color=darkblue:715c81bcd5][b:715c81bcd5]?T[/b:715c81bcd5][/color:7 15c81bcd5]b)infectivity decreases after seroconversion Tc)the CD4 count is a useful adjunct in diagnosis Td)measurement of HIV RNA viral load is most useful in diagnosis HIV seroconversion Illness occurs witin 1-4 wks......T...........SEROCONVERSION OCCURS IN FIRST FEW WEEKS......FOLLOWED BY SIGNS OF ILLNESS After illness infectivity decreases .........[b:715c81bcd5][color=darkblue:715c81bcd5].F.........[/color:715c81 bcd5][/b:715c81bcd5]I THINK SO BECAUSE THOUGH ITS A LATENT PHASE AND NO OVERT SYMPTOMS OCCUR WHILE THE VIRUS MULTIPLIES, I THINK INFECTIVITY REMAINS JUST THE SAME......... The best available diagnostic test is HIV viral load assay........[color=red:715c81bcd5][b:715c81bcd5]T[/b:715c81bcd5][/col or:715c81bcd5].........FROM AN OBSTETRIC POINT OF VIEW.....VIRAL LOAD IS THE ONLY USEFUL WAY OF PREDICTING DISEASE......BECAUSE SEROLOGY IS UNRELIABLE IN ITS RESULTS DUE TO MATERNAL ANTIBODIES......WHICH MIGHT TAKE UPTO 18MONTHS TO DIE DOWN... Without any intervention during pregnancy the mother to child transmission rate is 80% .........[color=red:715c81bcd5][b:715c81bcd5]F[/b:715c81bcd5][/color:71 5c81bcd5].....ITS 15-45%......LATTER IN UNTREATED CASES...... Mode of delivery by LSCS decreases the transmission to baby......[color=red:715c81bcd5][b:715c81bcd5]T[/b:715c81bcd5][/color :715c81bcd5].....CAN BRING THE TRANSMISS DOWN TO 2% Short course of anti viral Rx during labor decreases transmission to baby.........[b:715c81bcd5][color=red:715c81bcd5]T[/color:715c81bcd5][/ b:715c81bcd5]

BF is safe ........[b:715c81bcd5][color=red:715c81bcd5]F[/color:715c81bcd5][/b:71 5c81bcd5].......MOST IMP METHOD OF TRASMISS IF OTHER PREVENTIVE MEASURES HAVE BEEN UNDERTAKEN....

The following tumours arise in the ovary: Fa) nephroblastoma Tb) cystadenoma Tc) granulosa cell tumour Fd) neuroblastoma Te) teratoma Early blood borne dissemination is a characteristic feature of: Fa) carcinoma of the endometrium [color=red:715c81bcd5]T[/color:715c81bcd5]b) osteosarcoma Fc) basal cell carcinoma Fd) carcinoma of the cervix Te) choriocarcinoma Breast milk compared to cow's milk (!!) a) has more protein Tb) has more sodium c) contains more carbohydrate d) has more casein e) contains more calories difference b/w human milk and cows milk Tlactose Tsodium ascorbic acid Twater PS:human milk contains more water The following are consequences of pulmonary embolism: Ta) pulmonary infarction Tb) fibrinous pleurisy

?Fc) right ventricular hypertrophy Td) sudden death Te) haemoptysis [color=blue:715c81bcd5]In a country wide survey, 10 general practices were picked at random and 5% of patients were selected at random from each practice: Ta) the sample of patients is a true random sample Tb) all practices had an equal chance of selection Tc) all patients had an equal chance of selection Fd) two siblings could not have been selected Te) inferences about all patients in the country may be drawn from the sample [/color:715c81bcd5] ans from onexamination Nitric oxide Ta) is synthesised in the endothelium Tb) has a short half life Fc) causes smooth muscle contraction Td)production is increases during pregnancy [b:715c81bcd5]e) combines with oxygen to produce L-arginine [/b:715c81bcd5] T generated by arginine (from onexamination) Carbohydrates a) yield 17kj/g of energy b) are a major source of energy for the brain ?Tc) are a a major component of the diet Fd) give a higher energy yield compared to fats Head mesenchyme gives rise to the following skull bones a)Parietal b)Mandible c)Maxillary d)Frontal e)Ethmoid Pre-implantation genetic diagnosis

Ta) can be used to identify single gene defects Fb) is used in ICSI only Fc) testing for trisomy 21 should be offered in a case of ovum donation with a 45 year old recipient and 23 year old donor Fd) is routinely used in IVF in selection of embryos ?T[b:715c81bcd5]e) is governed by the regulations of the Human, Embryology and Fertilisation Authority[/b:715c81bcd5] The following are gram positive organisms ?Fa)Brucella Tb)Listeria Tc)Staphylococcus ?Fd)Vibrio cholera Endometriotic deposits Fa) only occur in the pelvis ?Tb) consist of deposits of endometrial stromal tissue without glands ?Fc) do not occur in postmenopausal women ?Td) are hormone sensitive Amyloidosis Ta)Can be inherited Tb)Is associated with bronchiectasis Tc)Is a cause of cardiomyopathy (ref: Familial Amyloidosis Familial amyloidosis, or ATTR, is a rare form of inherited amyloidosis. The amyloid deposits in familial amyloidosis are composed of the protein transthyretin, or TTR, which is made in the liver. Familial amyloidosis is inherited an [color=darkblue:715c81bcd5]autosomal dominant [/color:715c81bcd5]in genetics terminology) http://www.ncbi.nlm.nih.gov/entrez/query.fcgi?cmd=Retrieve&db=PubM ed&list_uids=12472203&dopt=Abstract http://www.emedicine.com/med/topic3365.htm When the heart is involved heavily but the nerves are not, the disease is called familial amyloid cardiomyopathy (FAC).

Phenylketonuria Fa)can only be diagnosed in the adult Tb)is diagnosed by high levels of phenylpyruvate in the urine ?Tc)treatment is with diet restriction ?Tinherited disorder Fcause by defficiency of phenyl alanine ( defficiency of phenylalanine hydroxylase) Fdietry restriction of tyrosine helpful (restriction of phenylalanine) ?Fdiagnosed by measuring phenyl ketone level in urine DUB (Dysfunctional uterine bleeding) Ta) is common at the extremes of life Tb) may be caused by coagulation disorders

Insulin (*half life of 30min) T Pituitary gland Anatomy (*contains pars tuberalis) F Many Pathology questions (equivalents ie *ovarian dysgerminoma = seminoma?, *endodermal sinus tumour = yolk sac tumour?) Embryology (notochord, primitive streak, mesonephros derivatives) NOTOCHORD R SOLID structure as it forms --T GIVE RISE TO brain -F F is formed from primitive streak (ans from mrcogexam) [color=blue:715c81bcd5][b:715c81bcd5]F fuses temporally with the endoblast of yolk sac[/b:715c81bcd5][/color:715c81bcd5] Ionising radiation (?T radiosensitivity ie *liver /intestinal epithelium - which is more radiosensitive; *is radiation exposure of dental x-ray equivalent to 1/5 that from a transatlantic flight)

essential amino acids Fare not synthesized by body sufficiently ?Tall ketogenic ?Tcan produce energy dont remember exact wording [b:715c81bcd5]Are ONLY used for the synthesis or important body proteins. Canot be catabolised for energy.[/b:715c81bcd5] radiation ?Tmostly harmful in 8-12 wks gestation Txrays r ionizing radiation Tcontraindicated in pregnancy [b:715c81bcd5]transatlantic flight hv higher dose of radiation than xray [/b:715c81bcd5] Biophysics: Radiation effect more in hypoxic tissue..false Bone, peripheral nerve and muscle are senstive to radiation..false Intestine and bone marrow are sestive to radiation..true

chylomicrins Ta) are not normally present in the fasting state ?T[b:715c81bcd5]responsible for turbulence of plsma?[/b:715c81bcd5]dont remem [color=darkblue:715c81bcd5]metabolized in adipose tissue [b:715c81bcd5]true [/b:715c81bcd5]80% 20% in liver [/color:715c81bcd5] Fafter hydrolysis by lipoprotien mainly phospholipids ?F r absorbed form intestinal luminal cells to ----? (ref: Monoglycerides and free fatty acids associate with bile salts and lecithin to form micelles * The core of the micelle is composed of cholesterol and fat soluble vitamins * Monoglycerides, cholesterol and free fatty acids are absorbed by passive diffusion across the duodenal and jejunal mucosa while the bile salts

remain in the lumen and are absorbed in the terminal ileum Chylomicrons have a core of triglycerides and cholesterol and a coat of protein and phospholipids

ovarian tumors Tchoriocharcinoma can arise cocp F increase risk of ovarian cancers Pill failure: tetracyclineT Rifmpicintrue digoxin....F phenobarbitone....T Contraindicated in breast feeding: cascara.....T phenophthalien...F norethisterone.....F chloramphenicol...T sulphonamides......T

b cells Farise from plasma cells in pregnancy humoral immunity is depressed early in pregnany Ftumor necrotic factor alpha raised in parturition pulmonary embolism cause s : Tsudden death Tfibrinous plurisy

Frt ventricular hypertrohy Thaemoptysis following rise in leuteal phase Tbasal body temp Tprogesterone

SKULL MESENCHYME GIVE RISE TEMPORAL BONE FRONTAL PARIETAL Q-Examples of active transport across a membrane include the passage of: ?T1-Potassium ions into the neurons F2-Water into the proximal renal tubular cell --F ?T3-Glucose from the proximal renal tubular cell [color=red:715c81bcd5][b:715c81bcd5]F[/b:715c81bcd5][/color:715c81b cd5]4-Fatty acids into the mucosal cells of the intestine 5-Hydrogen ions from the gastric oxyntic cells. (ref: glucose absorption in small intestine n renal tubules is active,all other places is facilitated using GLUT. also Na/K pump is active. Na absorption is active in renal tubules. Monoglycerides, cholesterol and free fatty acids are absorbed by passive diffusion across the duodenal and jejunal mucosa while the bile salts remain in the lumen and are absorbed in the terminal ileum )

ENDOMETRIOSIS has endometrial tissue with no glands ?Tnot occurs after menopause[b:715c81bcd5][/b:715c81bcd5] Regarding data

a The coefficient of variation expresses the Standard deviation as a proportion of the mean b Standard error of mean indicates how close a sample is to the population mean c A probability of one means that an event has one in ten chance of happening d Logarithmictransfoormation is a method of normalizing data that are not normally distributed e ANOVA is used for normally distributed data

Indomethacin can block ovulaion Pulsatile release of LH occurs in first weel of life RBCs possess the antigens of ABO system in fetus MRI involves the administration of radio labelled compound Doppler effect is used to detect movement Proton is injected to improve contrast during MRI

ventilation and stimulation of central chemoreceptors...someone said true for hypoxia. From my knowledge, the central chemoreceptors respond to pH and CO2, not O2, rather the PERIPHERAL aortic and carotid bodies respond to O2 and thus hypoxia.

sept.2005

Sigmoid colon

Attached to pelvic wall Lies lateral to psoas major Ends at S3 Rectum Starts at S3 No appendices epiploae Middle rectal artery is the major artery suuplying it 12 cm long T pallidum Spiral shape Causes yaws in children Stained with silver impregnation HIV seroconversion Illness occurs witin 1-4 wks After illness infectivity decreases The best available diagnostic test is HIV viral load assay Without any intervention during pregnancy the mother to child transmission rate is 80% Mode of delivery by LSCS decreases the transmission to baby Short course of anti viral Rx during labor decreases transmission to baby BF is safe Percutaneous mode of infection H simplex Rabies Hepatitis B Epstein barr virus Hep A mode of infection is through Blood transfusion Shared needles Faeco oral route Insect bites Air borne

Match the disease with their respective causative org Bone marrow suppression myco bacterium avium Red cell aplasia parvovirus

The foll are pre malignant CIN 3 Vulval lichen sclerosis Endometrioses Fibro epithelial polyp Atypical endometrial hyperplasia Bacterial vaginosis Microscopic finding clue cells pH less than 4.5 best Rx with ampicillin sensitive to clindamycin may be due to gardnerella vaginalis vaginal infecton incidence of n gonorrhoea resistance to penicillin is 1% incidence of n gonorrhoea resistance to 3 rd gen ceph 5% azithromycin DOC for trichomoniasis crytococcus neoformans causes meningitis C trachomatis Strains L123 cause LGV Reticulate body is the infectious form Transferrin Increased in pregnancy One third saturated with Fe Binds to 10 mg of iron per gm Levels in neonate are low Antibodies

Are soluble proteins Formed before 12 wks in fetus ABO antigens are present in fetus Foll Hb chains are present in intrauterine life Alpha 2 beta2 Alpha 2 gamma 2 Gamma 4 Alpha 2 epsilon 2 Zeta 2 epsilon2 Pulm embolism Spiral CT scan is indicated in suspected cases DVT always causes pulm embolism X ray angiography is diagnostic Spiral CT involves dignoses based on the perfusion copared to air in lungs ECG is reliable for diagnosis MRI Involves administerind of radio contrast agents Protons give better imaging Damage to fetus is due to tissue heating Xray Contraindicated in pregnancy Low rays are used for breast imaging Conventional USG Increased freq gives higher resolution Increased freq gives better attenuation TVS uses higher freq compared to conventional USG Doppler USG measures movement Gives accurate measurement of bld flow in fetal vessels Doppler effect based on blood flow within vessels

Calcification occurs in Renal calculi Hyaline degeneration of fibroid Parathyroid adenomas Secondary bone deposits in prostratic CA Foll originate from endothelial cells Nitric oxide Protein C Fibronectin Plasminogen activator Thyroid 100-200 micro gms are trapped daily increases in size in pregnancy trapsinorganic iodine colloid is stored outside epithelial cells TBG is increased n pregnancy 5% is bound to CBG ketone bodies formed from acetone formed from acetyl CoA not formed during starvation not utilised by brain Mgso4 Potentiates action of non depolarising agents Check respiratory status Causes brisk patellar reflexes In toxicity Rx IV calcium sulphate

CIN CIN 3 does not involve breach in basement epithelial layer Arias stella reaction mimicks clear cell CA Tissue biopsy is required for diagnosis

Incidence is decreased in renal transplant patients GH Stmulated by glucose infusion Increased in sleep Opposes insulin action Causes positive nitrogen balance Decreased in pregnancy Foll form boundaries of ovarian fossa Ureter Ext iliac vn Int iliac art Int pudendal art Obliterated umb art Penem antibiotics Active in beta lactam ring Drug of choice for ps aeroginoa Vagina Derived from mullerian ducts Seperated from anal canal by anal body Bladder Pelvic splanchnic stimulation contracts the trigone Ext urethral sphincter innervated by pudendal nv Mucosa of urethra is innervated by pudendal nv Transaction of spinal cord above S2 causes autonomous bladder Afferent innervation of glans of clitoris is by ilioinguinal nv Hypo k+ is found in Prolonged vomiting In diabetis ketoacidosis Addisons disease Foll are below inguinal ligament

Ilio inguinal nv Lateral branch of genito femoral nv Subcostal nv Femoral art Inf hypogastric (pelvic) plexus Supplies pain fibres to body of ut Continues as vesical plexus Lies at base of braod lig Sciatic nv S3&4 contribute towards it Lies behind quadratus femoris Tibial branch supplies short head of biceps femoris Injection on the outer & upper quadrant of the buttock may cause damage to the nv Undescended testis More common in premature than term infants Commonly assoc with gonadal neoplasia Asoc with 1% of absent testis More common on left side Embryo Parameso duct lies medial to mesonephros Mesonephros forms urine TVS detects fetal heart beat 21 days after implantation Erthropoiesis is entirely medullary

UG sinus Divides the cloaca coronally Urachus connects bladder to umbilicus Corpus luteum Is major source of relaxin Contains endothelial cells

Hysterectomy prolonges the life period 17 hydroxy prog is the main prog secreted steroids prevent recruitment of new follicles fertilisation acrosome fuses with zona pellucida polar body has same karyotype as ovum fetilised egg moves slower than unfertilised egg in fallopian tube inner ell mass lysis deciduas during implantation histo pathologic examination of tissue of conception can diagnose CIN 3 Cx incompetence Ectopic preg Gest Trophblastic Di Ring inversion of Y chromosome found the foll will be done Maternal chromosome is tested Paternal chromosome is tested Scan to look for male genitalia Anomaly scan and amniocentesis done for fetus Do karyotype of all previous children of couple Right ureter Below sigmoid coon Supplied by vaginal art Lined throughout by urothelium Contain glands Oral absorption of drug depends More availablibility in ionised form than non ionised Increased lipid solubility increases penetration in CNS Enters intestinal cells by simle diffusion Protein binding increases excretion Lipid form increases uptake Fetal circulation

Portal vn drains in IVC Ductus arteriosus takes blood to pulm art R atria and L atria communicate thro Foramen Ovale Voiding in females Occurs above rate of 20 ml/sec When vesical press is more than 45 mm H2O Initial contraction of trigone Enzyme activity is modified by Dephosphorylation Allosteric esters Binding to plasma prot Proteolytic cleaving of inactive precursors Gene transcription Functional cyst occurs in Stein levinthal synd PID Multiple sclerosis Gest tropho di Rx with clomiphene Amniotic fluid Hypotonic compared to fetal plasma Absored in lungs Unrelated to swallowing Increased until term in fetus Contains more HCO3 compared to fetal blood Action of complement Increase in permeability Migration of polymorphs Genetics South blot detects DNA DNA is exact copy of RNA

DNA is transcribed by RNA polymerase Ribosomes contain RNA Transcription occurs at any direction along the chain DNA replication is called transformation DNA has transcription restriction enzyme Histones do not contain DNA Insulin Binding to tyrosine receptor kinase increases activity Causes glucose transporter movement to plasma membrane Increases glycogen synthetase activity Increases fat deposition Increase glucose utilisation by CNS Heparin T is 1.5 hrs LMW heparin is administered 4 hrly Given in pregnancy causes intracranial haemorrhage in fetus Longer duration of action in S/C compared to IV administration Foll have imp natural reservoir other than human Listeria Saolmonella typhimurium Brucella Crohns disease Non caseous granulomatous infection Forms deep fissures Forms crypt abscesses Assoc with mucosal polyps Recognised to occur in vulva Assoc with intestinal wall thickening Absorption in kdny Glucose occurs in loop of henle Water is actively absorbed in proximal conv tubules

Estrogen 17 beta estradiol is formed by aromatisation of testos increases LH receptors secreted by theca cells granulosa cells secrete inhibin humoral immunity is depressed in pregnancy T & B cells are derived from bone marrow teratoma usually benign can occur is assoc with germ cell tumors secretes hormones

during menstruation platelet and fibrin plug is formed after 1st 20 hrs of bleeding hemostasis is thro estr mediated vaso constriction hemostasis is thro PG mediated spiral vessel constriction During ovulation Basal vacuoles occur in mid secretory phase Gland stromal mitosis maximum in mid secretory phase PG in follicle fluid during ovulation Increase of estr and prog receptors in proliferaive phase LDL receptors are increased in proliferative phase Indomethacin inhibits ovulation Matrixmetalloproteinases(MMC) is inhibited by progestrones Cloiphene can cause deficient secretory phase Glycogen Synthesis occurs in muscle Is stored along with water Carbohydrates Yield energy ?17 J/Kg Major source of energy for brain Major component of diet

Higher energy compared with fats Foll decrease the efficacy of OCPs Isoniazid Carbamaepine Phenytoin Rifampicin Female pelvis compared to male Sacrum more curved Greater sc notch is larger Sub pubic angle is more Distance betw pubic sym to acetabulum is less than the diam of acetabulum Vascular derived epithelial growth factor Causes vasodilatation of placental vessels Causes angiogenesis in placenta Is an angigogenic factor Increases permeability of endothelial cells And always save the best for last STATISTICS Definition LBW less than 1,500 gms PMR deaths in first 1 week per 1,000 births Perinatal mortality deaths after 22wks ( more than 154 days) until 1 wk after birth Maternal mortality rate Tests used ANOVA Std error of mean means diffce betw sample mean from population mean Probablity of 1 means 1 in 10 chance for event to occur Examples of ?continuous variables are Bld glucose

Haemoglobin Gender Height Blood pressure

Sigmoid colon Attached to pelvic wall......T Lies lateral to psoas major ........F Ends at S3 .........T SIGMOID COLON...STARTS AT PELVIC BRIM.....ENDS AT S3.......IS 45 CMS LONG.....LIES ON THE PERITONEAL SURFACE OF BLADDER AND UTERUS.....SO THE PSOAS IS A DEEPER STRUCTURE.....

Rectum Starts at S3 ........T No appendices epiploae.......T Middle rectal artery is the major artery suuplying it........F......ITS THE SUPERIOR RECTAL ARTERY 12 cm long .............T

T pallidum Spiral shape........T Causes yaws in children..........T Stained with silver impregnation.........F....... THE ONLY TESTS FOR MICROSCOPY ARE DARK GROUND

IMMUNOFLOURSCENCE................OTHERWISE SEROLOGY IS THE OTHER DIAGNOSTIC AID....

HIV seroconversion Illness occurs witin 1-4 wks......T...........SEROCONVERSION OCCURS IN FIRST FEW WEEKS......FOLLOWED BY SIGNS OF ILLNESS After illness infectivity decreases ..........F.........I THINK SO BECAUSE THOUGH ITS A LATENT PHASE AND NO OVERT SYMPTOMS OCCUR WHILE THE VIRUS MULTIPLIES, I THINK INFECTIVITY REMAINS JUST THE SAME......... The best available diagnostic test is HIV viral load assay........T.........FROM AN OBSTETRIC POINT OF VIEW.....VIRAL LOAD IS THE ONLY USEFUL WAY OF PREDICTING DISEASE......BECAUSE SEROLOGY IS UNRELIABLE IN ITS RESULTS DUE TO MATERNAL ANTIBODIES......WHICH MIGHT TAKE UPTO 18MONTHS TO DIE DOWN... Without any intervention during pregnancy the mother to child transmission rate is 80% .........F.....ITS 15-45%......LATTER IN UNTREATED CASES...... Mode of delivery by LSCS decreases the transmission to baby......T.....CAN BRING THE TRANSMISS DOWN TO 2% Short course of anti viral Rx during labor decreases transmission to baby.........T BF is safe ........F.......MOST IMP METHOD OF TRASMISS IF OTHER PREVENTIVE MEASURES HAVE BEEN UNDERTAKEN....

Percutaneous mode of infection H simplex........T Rabies .......F.....BITE WITH SALIVA Hepatitis B ........F...... Epstein barr virus .......T I THINK.......ECAUSE IT SAYS YOU CAN

GET IT FROM PATIENTS THROUGH "CASUAL CONTACT"......

Hep A mode of infection is through Blood transfusion.....F Shared needles .....F Faeco oral route ......T Insect bites ......F Air borne ......F Match the disease with their respective causative org Bone marrow suppression myco bacterium avium Red cell aplasia parvovirus

The foll are pre malignant CIN 3 ........T Vulval lichen sclerosis.......F BUT NOT SURE... Endometrioses .......F Fibro epithelial polyp ............T I THINK... Atypical endometrial hyperplasia ??T

Bacterial vaginosis Microscopic finding clue cells.........T pH less than 4.5 .......F........HAS TO BE MORE THAN 5 FOR BV TO OCCUR best Rx with ampicillin ............F sensitive to clindamycin ...........F.......I DIDNT FIND IT ANYWHERE.....SO I WILL GO FOR F

may be due to gardnerella vaginalis........T.....

vaginal infecton incidence of n gonorrhoea resistance to penicillin is 1% ......T BUT NOT SURE 100% incidence of n gonorrhoea resistance to 3 rd gen ceph 5% .....? FALSE azithromycin DOC for trichomoniasis ,.......F....METRO... crytococcus neoformans causes meningitis .........T.....ESP IN AIDS PTS

C trachomatis Strains L123 cause LGV.........T Reticulate body is the infectious form .......F.....RETICULATE I THE VEGETATIVE NON INF INTRACELL. FORM.... Transferrin Increased in pregnancy ........T One third saturated with Fe .........T Binds to 10 mg of iron per gm ???.....DUNNO....DIDNT FIND THIS KIND OF DATA ANYWHERE.... Levels in neonate are low........FALSE.....

Antibodies Are soluble proteins........I THINK ITS TRUE....THEY HAVE TO BE.....BUT NO BOOK CONFIRMATION YET..... Formed before 12 wks in fetus .........TRUE.....IGM FORMS AT 11 WEEKS.... ABO antigens are present in fetus........TRUE

Foll Hb chains are present in intrauterine life Alpha 2 beta2 .......TRUE......HB A......PRESENT 8TH MONTH ONWARS Alpha 2 gamma 2 ........T......HB F......ALL THROUGH PREG AND LATER TOO... Gamma 4 ......F Alpha 2 epsilon 2........F Zeta 2 epsilon2 ........T.....HBGOWER 1.......PRESENT I THE FIRST FEW WEEKS..... ALSO THERE IS HBA2....ALPHA2 AND DELTA 2 UNITS...

Pulm embolism Spiral CT scan is indicated in suspected cases.........F DVT always causes pulm embolism ............F X ray angiography is diagnostic ...............T Spiral CT involves dignoses based on the perfusion copared to air in lungs ...............F ECG is reliable for diagnosis............F IS SPIRAL CT AND VQ SCAN THE SAME THING????SORRY FOR ASKING SUCH A BASICALLY DUMB QUESTION....

MRI Involves administerind of radio contrast agents........T I THINK.... Protons give better imaging ?????T I THINK.........BECAUSE IT DEENDS ON THE OSCILLATION OF CHARGED PARTICLES......EPS THE HYDROGEN ATOM.... Damage to fetus is due to tissue heating.......F I THINK.....BECAUSE MRI IS NOT AN IONISING RADIATION.....SO I DONT THINK THERE IS ANY HEATING.....

Xray Contraindicated in pregnancy...............F Low rays are used for breast imaging ............????......LOW COMPARED TO WHAT??7 TO 10 MRADS ARE USED..........WHICH IS MORE THAN EG THE 2-3 MRADS FOR CHEST X RAY...........I DONT THINK THATS LOW.......I WOULD SAY FALSE.....

Conventional USG Increased freq gives higher resolution Increased freq gives better attenuation TVS uses higher freq compared to conventional USG Doppler USG measures movement ........T Gives accurate measurement of bld flow in fetal vessels........T Doppler effect based on blood flow within vessels ...........T......THE SOUND OF BLOOD FLOW WITHIN VESSELS....TO BE EXACT.... FOR THE FIRST THREE STEMS......I COULDNT FIND THE ANSWER......BUT IT WAS SOMEWHERE IN THIS FORUM.........I WILL POST IT AS SOON AS I COME ACROSS IT AGAIN.......

Calcification occurs in Renal calculi ...........T Hyaline degeneration of fibroid.........T.....CALCIFICATION OCURS IN FIBROID........I DONT WHAT THE SIGNIFICANCE OF HYALINE DEGENERATION IS IN TH QUESTION Parathyroid adenomas ............T Secondary bone deposits in prostratic CA ..........T

Foll originate from endothelial cells Nitric oxide......T Protein C ............F Fibronectin ........T Plasminogen activator................T

Thyroid 100-200 micro gms are trapped daily.......T increases in size in pregnancy .........T I THINK.....ACTOVITY INCREASES........SO DOES THE SIZE......BUT IM NOT SURE.... trapsinorganic iodine .......T colloid is stored outside epithelial cells.........T TBG is increased n pregnancy ...........T. 5% is bound to CBG ...........................TO TBG I THINK WAS MEANT....F......T3 AND T4 ARE BOUND 75 AND 85 %

ketone bodies formed from acetone..............T formed from acetyl CoA ..........T ormed during starvation .....T not utilised by <a href="http://www.[obscene]a.com/tutorial/?q=brain &s=0">brain </a> T

Mgso4 Potentiates action of non depolarising agents T I THINK Check respiratory status T......PARALYSIS OF RESP MUSCLE IS THE

FIRST SIGN OF OVERDOSE Causes brisk patellar reflexes T.........ANOTHER SIGN OF MG EXCESS.... In toxicity Rx IV calcium sulphate .....F.....CA GLUCONATE...

CIN CIN 3 does not involve breach in basement epithelial layer F....THATS EXACTLY TH DISTINCTION Arias stella reaction mimicks clear cell CA..........THIS ONE IS JUT SOMEWHERE IN MY brain CELLS......DONT KNOW NOW.... Tissue biopsy is required for diagnosis ......FALSE....SMEAR CAN ALSO SHOW CIN III CHNGES....... Incidence is decreased in renal transplant patients?????......F I THINK.....COULDNT FIND INFO ANYWHERE.......BUT MY GUESS IS THAT IMMUNOSUPRESSION DURING RENAL TRANSPLANT WOULD EXACERBATE MALIGNANCY RATHER THAN SUPPRESS IT.....

GH Stmulated by glucose infusion.........FALSE.... Increased in sleep ......T Opposes insulin action F Causes positive nitrogen balance F Decreased in pregnancy F GH STIM BY LOW ENERGY CONDN HYPOGLYCaenia EXCERCISE FASTING

HIGH AMINO ACIDS HIGH PROT MEAL ARGININE INFUSION STRESS STIM GLUCAGO SLEEP GH INHIB BY GLUCOSE CORTISOL FFA MEDROXYPROGEST GH ANABOLIC HORMONE......WANTS TO BULD AND CONSTRUCT.....SO I THINK N2 BALANCE WOULD BE NEGATIVE...... AND SHOULD BE HIGH IN PREG I THINK......

Foll form boundaries of ovarian fossa Ureter T LATERAL TO OF Ext iliac vn T ABOVE OF Int iliac art T ABOVE THEN POST TO OF Int pudendal art F.....WAY OFF BELOW THE LEVATOR ANI....NOWHERE CLOSE... Obliterated umb art..T LAT TO OVARY..

Penem antibiotics Active in beta lactam ring T Drug of choice for ps aeroginoa T

Vagina Derived from mullerian ducts T Seperated from anal canal by anal body....PINEAL BODY-T Bladder Pelvic splanchnic stimulation contracts the trigone F......ALL TRUE....BUT NOTHING CONTRACTS THE TRIGONE! Ext urethral sphincter innervated by pudendal nv....F....NO PUDENDAL INVLVMT IN URETHERA......SUPP BY SOMATIC FIBS FROM S2-4 Mucosa of urethra is innervated by pudendal nv F Transaction of spinal cord above S2 causes autonomous bladder T I THINK........BECAUSE IT WOULD STILL WORK AUTONOMOUSLY THROUGH REFLEX.......BUT NO SOMATIC ACTION BECAUSE CEREBRAL CONTROL IS LOST.... Afferent innervation of glans of clitoris is by ilioinguinal nv ........F AFFERRENT OF GLANS CLIT IS THROUGH PELVIC SPLANCH......EFF IS THRU PUDENDAL N......AND BLOOD SUPPLY IS THROUGH INT PUD ARE PUD N ALSO SUPPS INF RECTAL NERVE AND A PERINEAL NERVE....

Hypo k+ is found in Prolonged vomiting T In diabetis ketoacidosis T Addisons disease F HYPERKALEMIA HERE........CORTISOL IS HYPOKALEMIC.......AND NO CORTISOL IN ADDISONS....

Foll are below inguinal ligament Ilio inguinal nv T Lateral branch of genito femoral nv T Subcostal nv F Femoral art T

Inf hypogastric (pelvic) plexus Supplies pain fibres to body of ut T Continues as vesical plexus T Lies at base of braod lig T

Sciatic nv S3&4 contribute towards it F L4 TO S3 Lies behind quadratus femoris F......IT LIES ON IRIFORMIS.....WIT NO MORE USCLES ON TOP....QF IS A VERY DEEP MUSCLE Tibial branch supplies short head of biceps femoris ......FASE......SHORT HEAD IS BY COMMON PERONEAL.....LONG HEAD ISBY TIBIAL..... Injection on the outer & upper quadrant of the buttock may cause damage to the nv ....FLASE........THIS IS THE SAFEST AREA

Undescended testis More common in premature than term infants T Commonly assoc with gonadal neoplasia T Asoc with 1% of absent testis More common on left side

Corpus luteum Is major source of relaxin T RELAXIN, INHIBIN AND ACTIVIN Contains endothelial cells T I THINK, IT HAS RICH BLOOD SUPPLY.......SO ENDOTHELIAL CELLS TOO I SUPPOSE... Hysterectomy prolonges the life period F I THINK.....HYSTERECTOMY WOULD HAVE NO EFFECT.......OR IT WOULD DECREASE THE LIFE......NO BOOK CONFIRMATION.... 17 hydroxy prog is the main prog secreted F I THINK.......PROGESTERONE IS THE MAIN steroids prevent recruitment of new follicles T STEROIDS LOWER THE LEVEL OF ESTROGEN RECEPTORS..

fertilisation acrosome fuses with zona pellucida T polar body has same karyotype as ovum T fetilised egg moves slower than unfertilised egg in fallopian tube F UNSURE.....BUT WHY SHOULD IT? inner ell mass lysis deciduas during implantation F INNER CELL MASS DOES NOTHING OUTSIDE.....IT BECOMES THE FETUS.....

histo pathologic examination of tissue of conception can diagnose CIN 3 T

Cx incompetence F Ectopic preg T Gest Trophblastic Di T

Ring inversion of Y chromosome found the foll will be done Maternal chromosome is tested F Paternal chromosome is tested T Scan to look for male genitalia T Anomaly scan and amniocentesis done for fetus ?T Do karyotype of all previous children of couple F ALL ARE DEDUCTIONS......BECAUSE Y IS RELATED WITH PATERNAL.....AND ASSOCIATED WITH EXPRESSION OF MALE PHENOTYPE.......PLEASE DISCUSS

Right ureter Below sigmoid coon F LEFT URETER Supplied by vaginal art T Lined throughout by urothelium T IF UROTHELIUM IS ANOTHER NAME FOR TRANSITIONAL EPITH Contain glands F.......BLADDER EPITH IS DEVOID OF GLANDS......I THINK URETERS' IS ALSO THE SAME....

Oral absorption of drug depends More availablibility in ionised form than non ionised F.....BECAUSE I DIDNT FIND IT ANYWHERE THAT IONISED FORM INFLUENCED ORAL ABS.....

Increased lipid solubility increases penetration in CNS T Enters intestinal cells by simle diffusion T Protein binding increases excretion F Lipid form increases uptake T

Fetal circulation Portal vn drains in IVC T Ductus arteriosus takes blood to pulm art F AWAY FROM IT R atria and L atria communicate thro Foramen Ovale T

Voiding in females Occurs above rate of 20 ml/sec T MAX URINE FLOW RATE =20-40 ML/SEC When vesical press is more than 45 mm H2O T VOID PRESS=45-70 MM Initial contraction of trigone F........TRIGONE DOES NOTHING!!!

Enzyme activity is modified by Dephosphorylation T Allosteric esters T Binding to plasma prot T Proteolytic cleaving of inactive precursors T Gene transcription ?F ALL GUESSES......ANYONE KNOWS THE ANSWERS??

Functional cyst occurs in Stein levinthal synd T PID F Multiple sclerosis F Gest tropho di F Rx with clomiphene T

Amniotic fluid Hypotonic compared to fetal plasma ?T Absored in lungs ?T Unrelated to swallowing F Increased until term in fetus F DECS TO TERM Contains more HCO3 compared to fetal blood ?T

Action of complement Increase in permeability T Migration of polymorphs T

Genetics

South blot detects DNA F ITS ONLE SEROLOGICAL TEST DNA is exact copy of RNA F DNA is transcribed by RNA polymerase T Ribosomes contain RNA T Transcription occurs at any direction along the chain F ONLY 5' TO 3' DNA replication is called transformation F DNA has transcription restriction enzyme ?F DIDNT FIND ANYTHING LIKE THAT ANYWHERE Histones do not contain DNA T

Insulin Binding to tyrosine receptor kinase increases activity T Causes glucose transporter movement to plasma membrane T Increases glycogen synthetase activity T Increases fat deposition T Increase glucose utilisation by CNS ?T

Heparin T is 1.5 hrs T LMW heparin is administered 4 hrly F 8 TO 12 Given in pregnancy causes intracranial haemorrhage in fetus F DOESNT CROSS PLAC COS ITS TOO BIG Longer duration of action in S/C compared to IV administration T

Foll have imp natural reservoir other than human

Listeria F Saolmonella typhimurium F Brucella T FROM DAIRY AIMALS

Crohns disease Non caseous granulomatous infection T Forms deep fissures T Forms crypt abscesses T Assoc with mucosal polyps F......PSEUDOPOLYPS Recognised to occur in vulva F......AT THE MOST....A FISTULA MIGHT OPEN I THE VULVA......BUT THATS NOT CROHNS OCCURING IN THE VULVA I THINK.... Assoc with intestinal wall thickening ? T......NOT A PROPER STATEMENT......THERE IS INFLAMMATION.....THICKENING IS A VERY LAYMAN TERM I THINK....

Absorption in kdny Glucose occurs in loop of henle F ACTIVE ABSPN IN PCT WITH AA Water is actively absorbed in proximal conv tubules F OPASSIVE DIFFSN IN PCT WITH NA AND CL

Estrogen 17 beta estradiol is formed by aromatisation of testos T increases LH receptors T secreted by theca cells F GRANULOSA CELLS

granulosa cells secrete inhibin F CORP LUT DOES humoral immunity is depressed in pregnancy F I THINK.....IF IT WERE TRUE, RHESUS CONFLICT WOULDNT BE AN ISSUE T & B cells are derived from bone marrow T

teratoma usually benign T can occur is assoc with germ cell tumors T secretes hormones F....NOT USUALLY I THINK

during menstruation platelet and fibrin plug is formed after 1st 20 hrs of bleeding F I THINK.....PLATELET PLUG FORMN TAKES PLACE VERY QUICK AND IN VERY SMALL TRAUMAS..... hemostasis is thro estr mediated vaso constriction F hemostasis is thro PG mediated spiral vessel constriction F PG ARE CONSIDERED TO BE THE CAUSE.....BUT I DONT KNOW IF ITS AT THE SPIRAL ARTS OR BELOW THEM.......BECAUSE THE APIRAL ARTS ARE ACTUALLY SHED....ISNT IT...

During ovulation Basal vacuoles occur in mid secretory phase T Gland stromal mitosis maximum in mid secretory phase T PG in follicle fluid during ovulation F DIDNT FIND THAT ANYWHERE

Increase of estr and prog receptors in proliferaive phase T I THINK LDL receptors are increased in proliferative phase ?F Indomethacin inhibits ovulation F Matrixmetalloproteinases(MMC) is inhibited by progestrones ???!!! WOW NO CLUE Cloiphene can cause deficient secretory phase F ITS ANTI ESTROGEN......SO IT MAY SHORTEN PROLIF PHASE....

Glycogen Synthesis occurs in muscle T Is stored along with water F ITS POLYMERISED......AND TURNS TO CRYSTALS ON STORING.....THIS IS THE REASON THERE IS NO OSMOTIC HAVOC IN CELLS WITH IT

Carbohydrates Yield energy ?17 J/Kg T Major source of energy q=brain &s=0">brain </a> F I THINK ITS GLUCOSE ACTULLY..... Major component of diet T Higher energy compared with fats F FATS HAVE MAX ENERGY

Foll decrease the efficacy of OCPs Isoniazid F Carbamaepine T Phenytoin T

Rifampicin T ALSO PHENOBARB

Female pelvis compared to male Sacrum more curved F LESS CURVED AND MORE MORE BACK TILT Greater sc notch is larger T Sub pubic angle is more T Distance betw pubic sym to acetabulum is less than the diam of acetabulum F......ACETABULUM IS VERY SMALL...AND THE DISTANCE IS GREATER

S-ar putea să vă placă și